From kls Fri Nov 13 22:47:31 1992 Newsgroups: sci.aeronautics.airliners Path: news From: Karl Swartz Subject: Welcome to sci.aeronautics.airliners! Message-ID: Approved: kls@ohare.Chicago.COM X-Original-Message-Id: Sender: kls@ohare.Chicago.COM Date: Fri, 13 Nov 92 22:45:11 PST Welcome to sci.aeronautics.airliners! This is a moderated newsgroup for the discussion of airliners. More precisely, the charter, taken from the CFV, is as follows: A moderated discussion group on airliner technology: the design, construction, performance, human factors, operation, and histories of transport-category airplanes. MODERATION POLICY ---------- ------ The moderation policy will in general be to post most articles as they are submitted, rejecting articles only if they are redundant or mostly content-free (a flood of random conjecture after a crash, for example) or inappropriate to the charter of the group. I expect to process new submissions at least once per day, except occasional weekends and major holidays. If I anticipate longer delays I'll send a note to the group and, when appropriate, arrange a backup moderator. SUBMISSIONS ----------- Submissions for the newsgroup should be sent to airliners@chicago.com If your newsreader properly supports posting to moderated newsgroups (and your mailpaths file is correctly configured) you may prefer to post articles -- this should have the same effect as sending mail to the above address, with the added benefit of preserving some of the reference information for those using threaded newsreaders. Administrative questions pertaining to the group should be sent to airliners-admin@chicago.com ARCHIVES -------- Several archives will be maintained and made available for anonymous ftp. Further details will be posted within the next few weeks. MAILING LIST ------- ---- A mailing list form of the group is also being considered for people lacking access to Usenet. As with the archives, details will be posted when they are available. CREDITS ------- Many thanks are due to Robert Dorsett, who organized and ran the vote and without whose perseverence this group would probably still be a topic for idle dinner e-mail. Thanks, too, to the 168 people who voted fror the group. (I'll refrain from any Bronx cheer to the 26 naysayers -- at least they voted!) And thanks in advance to all of you eager contributors -- I'm eagerly awaiting the first article! -- Karl Swartz |INet kls@ditka.chicago.com 1-415/854-3409 |UUCP uunet!decwrl!ditka!kls QUAYLE IS A BOZOE |Snail 2144 Sand Hill Rd., Menlo Park CA 94025, USA "I never vote for anyone. I always vote against." (W. C. Fields) From kls Tue Nov 17 03:10:31 1992 Newsgroups: sci.aeronautics.airliners Path: news From: bowen@comlab.oxford.ac.uk (Jonathan Bowen) Subject: TV programme on 777 Message-ID: Approved: kls@ohare.Chicago.COM Reply-To: Jonathan.Bowen@prg.oxford.ac.uk (Jonathan Bowen) Organization: Programming Research Group, Oxford University, UK X-Original-Message-Id: <1992Nov16.121654.13087@topaz.comlab.ox.ac.uk> Sender: kls@ohare.Chicago.COM Date: Mon, 16 Nov 1992 12:16:54 GMT Last night (Sunday 15 Nov), Channel 4 broadcast a TV programme on the production of the new Boeing 777 airplane in the Equinox series in the UK. Unfortunately I only saw the last 5 minutes of the programme. Would anyone who saw the whole programme like to provide a summary in this forum? In particular, I would be interested to hear if any mention was made of the fly-by-wire and safety aspects of the plane. -- Jonathan Bowen, Oxford University Computing Laboratory. From kls Wed Nov 18 00:26:01 1992 Newsgroups: sci.aeronautics.airliners Path: news From: cid@athena.mit.edu (Derek H Cedillo) Subject: 747 engine mounts Message-ID: Approved: kls@ohare.Chicago.COM Organization: Massachusetts Institute of Technology X-Original-Message-Id: <1992Nov16.042657.19926@athena.mit.edu> Sender: kls@ohare.Chicago.COM Date: Mon, 16 Nov 1992 04:26:57 GMT I would like to merely offer up a first topic for discussion. Boeing and the FAA have conducted fuse pin inspections on the 747 fleet. A few of these inspections found cracked pins, and quite a few found a large amount of corrosion. Inspections have been expanded to include the engine mount pylons. What Im looking for is some specs on the shear yeilding of these pins and and the stuctural integrity of the pylons. Thanks a bunch, Derek --------------------------------------------------------------------------- "There Are Two Great Tragedies In Life, One Is Not To Get Your Heart's Desire. The Other Is To Get It. -Bernard Shaw --[20968]-- From kls Wed Nov 18 00:26:05 1992 Newsgroups: sci.aeronautics.airliners Path: news From: jerry@telecom.ksu.edu (Jerry Anderson) Subject: Boeing 747-300 Message-ID: Approved: kls@ohare.Chicago.COM X-Original-Message-Id: <9211171641.AA06619@pawnee.telecom.ksu.edu> Sender: kls@ohare.Chicago.COM Date: Tue, 17 Nov 92 10:41:51 CST Is the Boeing 747-300 the largest commercial passenger aircraft in the world? Is the -300 the latest version, or are there newer, possibly larger stretched versions of the 747? I've heard Boeing, McDonnell-Douglas and Airbus all have plans in the works for Really Big Planes in the 600-700 passenger, 7500-8000 mile range. Does anyone know if these planes are really going to get built, or is this the usual "if we can pre-sell a couple of hundred, maybe we'll really build it?" Does anyone have model numbers, specs (passengers, range), or projected delivery dates? These numbers come from memory, and I have no faith in them at all: Company Model Delivery ------- ------ -------- McDonnell-Douglas MD-11 1993 Airbus AE-400 1994 Boeing 777 1995-6 -- jerry@telecom.ksu.edu Jerry Anderson Kansas State University vox: (913) 532-6936 Telecommunications fax: (913) 532-7114 Manhattan KS 66506 From kls Wed Nov 18 01:42:59 1992 Newsgroups: sci.aeronautics.airliners Path: news From: kls@ohare.Chicago.COM (Karl Swartz) Subject: Re: Boeing 747-300 References: Message-ID: Approved: kls@ohare.Chicago.COM Organization: Chicago Software Works X-Original-Message-ID: <1992Nov18.094020.12550@ohare.Chicago.COM> Sender: kls@ohare.Chicago.COM Date: Wed, 18 Nov 1992 09:40:20 GMT In article jerry@telecom.ksu.edu (Jerry Anderson) writes: >Is the Boeing 747-300 the largest commercial passenger aircraft >in the world? Is the -300 the latest version, or are there >newer, possibly larger stretched versions of the 747? The 747-300 is simply a -200 with an Extended Upper Deck, which allows greater seating. The only 747 model currently being produced is the 747-400, which shares the same fuselage dimensions as the -300 version and visually is quite similar, the most noticeable difference being winglets at the wingtips and subtle engine differences. Internally, the -400 is a *very* different aircraft, including a two-person cockpit and new, more powerful, and more fuel efficient engines. MGTOW is up to at least 870,000 from 833,000 for the -200/-300 models and range is substantially increased as well. Unless the Russians have something which I've missed, the 747-400 is easily the largest commercial passenger aircraft in terms of number of seats and payload. Its range is also the greatest of anything now in service, though the Airbus A340 will exceed it once it enters service next year. >I've heard Boeing, McDonnell-Douglas and Airbus all have plans >in the works for Really Big Planes in the 600-700 passenger, >7500-8000 mile range. Does anyone know if these planes are >really going to get built, or is this the usual "if we can >pre-sell a couple of hundred, maybe we'll really build it?" No manufacturer (well, except for Airbus) would build a new aircraft just for grins. If they have tangible demand in the form of orders they'll build it, if not, they won't. Given the current state of the airline industry major new orders aren't likely in the near future. > McDonnell-Douglas MD-11 1993 > Airbus AE-400 1994 > Boeing 777 1995-6 All three of these are much smaller than what you're thinking of. Here are the important parameters for these three plus the 747-400 for comparison. Seating is for a "typical" three-class cabin and service is the date of first service; MGTOW is in US pounds. Mfr. Type MGTOW seating service Boeing 747-400 870,000 430 1989 MacDAC MD-11 618,000 250 1991 Airbus A-340 559,000 230 1993 Boeing 777 515,000 ~220 1995 In the 600+ passenger market, Boeing has talked about both further stretches of the 747 and an entirely new aircraft, sometimes using the N650 moniker. McDonnell-Douglas has most recently talked about the MD-12 -- once yet another stretch of the MD-11 -- as a new and much larger aircraft, also in the 600+ passenger category. Airbus has said that if there is demand and/or if Boeing builds such an aircraft, Airbus will build one too. The name A600 or maybe A2000 seems vaguely familiar though I can't locate any references. -- Karl Swartz |INet kls@ditka.chicago.com 1-415/854-3409 |UUCP uunet!decwrl!ditka!kls QUAYLE IS A BOZOE |Snail 2144 Sand Hill Rd., Menlo Park CA 94025, USA "I never vote for anyone. I always vote against." (W. C. Fields) From kls Wed Nov 18 10:21:47 1992 Newsgroups: sci.aeronautics.airliners Path: news From: sdl@linus.mitre.org Subject: hydraulic problems with DC-10's?? Message-ID: Approved: kls@ohare.Chicago.COM X-Original-Message-Id: <9211181502.AA03545@rigel.mitre.org> Sender: kls@ohare.Chicago.COM Date: Wed, 18 Nov 92 10:02:08 -0500 Great new newsgroup! Perhaps now I can get a more definitive answer to the following: Some of my pilot friends have accused the DC-10 as having a particularly bad history of hydraulic problems (which have contributed to a few crashes). But do the statistics really support the notion that the DC-10 really has a significantly worse safety record than other wide-body jets, or is this just a myth? Also, have all these hydraulic problems been corrected, or does the DC-10 still suffer from hydraulic problems even today? From kls Wed Nov 18 22:58:13 1992 Newsgroups: sci.aeronautics.airliners Path: news From: lomasm@t9.cs.man.ac.uk (Martin Lomas) Subject: Re: TV programme on 777 References: Message-ID: Approved: kls@ohare.Chicago.COM X-Original-Message-Id: Sender: kls@ohare.Chicago.COM Date: 18 Nov 92 19:39:39 GMT In bowen@comlab.oxford.ac.uk (Jonathan Bowen) writes: >Last night (Sunday 15 Nov), Channel 4 broadcast a TV programme on the >production of the new Boeing 777 airplane in the Equinox series in the >UK. Unfortunately I only saw the last 5 minutes of the programme. Would >anyone who saw the whole programme like to provide a summary in this >forum? In particular, I would be interested to hear if any mention was >made of the fly-by-wire and safety aspects of the plane. >-- >Jonathan Bowen, >Oxford University Computing Laboratory. OK, here goes (and without the aid of a video recorder! :-): New Boeing 777 and its design. The program concentrated on the general aspects of how the whole thing is put together - ie: need, management, tools used, some financial. Hard technical details were scant. Boeing need a plane that is bigger than their 757 and 767 yet smaller than the 747. With latest design, the 777 will be smaller than the 747 yet carry nearly the same number of passengers. Pressure from customers and competition from Airbus and McDonnald Douglas. Large order from American Airlines and others prompted the design start. Production aircraft by 1995(?). Large mainframe cluster (IBM!) being used for all drawings (CAD) work with stress analysis to let engineers reduce component weight ('safely') where possible. A full size mockup to test whether all components will fit together will not be needed (as made for previous planes) due to computer design checks. (Component clashes checked and highlighted.) International manufacturing: Electronics from UK, rudder from Australia, various (large) sections from Europe, and all assembled at Boeing's now being built plant in America. Some design decisions shown: Use of Aluminium-Lithium alloy -- strong and light but cracks when drilled. However, the cracks don't propagate and so are safe. Reluctantly rejected due to engineers' fears of cracks and possible confusion over what parts must be crack free and where cracks are 'ok'. Safety: Issue of doors mentioned -- trade many doors against plane too heavy, so compromise. Doors must open even with quarter inch ice sealing them shut. Good demonstration showing their intended design works. (Big freezer, idiot in there sprays on water, big party next day to see the door break open.) Cost compromises: None made where the plane's flyability is concerned, possible compromises for such as crash survivabilty and other cases. Concentrate efforts to AVOID crashes. Anyway, bad for business if your plane falls out of the sky! Fly by wire briefly explained. Safety issues NOT covered. Only advantages of better fuel economy and smoother flight mentioned. Implied weight savings due to easier mechanics. Intended fly-by-wire system currently being tested on a 757 with dual control systems (mechanical and the FBW). Hundreds of real flights being performed. Some mention given to making the controls similar to existing planes so the pilots can be easily trained for the plane. Control ergonomics reviewed by test pilots to great detail (focus in on 'that knob doesn't click nicely when switching between settings -- make it click better...'). Two engines only on the plane -- one adequate for flight across the Atlantic. Engine reliabilty relied upon. Customer engineers allowed to review the new engines for servicing and to make mods. Customers allowed into (some) of the Boeing meetings to discuss the 777's design and to suggest mods. 'Open management' strategy. Main thrust of the program was the huge cost and complexity of the task of producing a new aircraft to tight schedules. Management style/issues covered much more than the technical issues. Good documentary very much in the 'Skyscraper' style that this film company first produced. Any other critics out there? Martin. From kls Wed Nov 18 22:58:17 1992 Newsgroups: sci.aeronautics.airliners Path: news From: kls@ohare.Chicago.COM (Karl Swartz) Subject: Re: hydraulic problems with DC-10's?? References: Message-ID: Approved: kls@ohare.Chicago.COM Organization: Chicago Software Works X-Original-Message-ID: <1992Nov18.205005.13733@ohare.Chicago.COM> Sender: kls@ohare.Chicago.COM Date: Wed, 18 Nov 1992 20:50:05 GMT In article sdl@linus.mitre.org writes: >Some of my pilot friends have accused the DC-10 as having a >particularly bad history of hydraulic problems (which have >contributed to a few crashes). Two, at least. AA 191 at Chicago/O'Hare on May 25, 1979, and UA 232 at Sioux City, Iowa on July 19, 1989. The Turkish Airlines crash near Paris on March 3, 1974 helped set up UA 232 though I'm never seen any reference to the hydraulics as being contributory to that crash. >But do the statistics really support the notion that the DC-10 really >has a significantly worse safety record than other wide-body jets, or >is this just a myth? After the A320 crash at Strasbourg early this year I saw something that said the A320 had overtaken the DC-10 as having the worst safety record of any large jetliner and that both were an order of magnitude worse than the third-place contender. I really wish I could find the reference, but alas, I can't. I don't recall the metrics used, and one could certainly debate the statistical validity given the small samples involved. Another view is to look at the number of airworthiness directives (AD) issued by the FAA in the US for the DC-10 versus its contemporaries. As of January 1, 1982, the DC-10 had 148, far ahead of Boeing's 747 with 57 and Lockheed's L-1011 with 51. The FAA clearly found a lot more to worry about in the DC-10. Probably the best general interest discussion of the DC-10 and all its problems is in The Sporty Game, by John Newhouse (Alfred A. Knopf, New York, 1982). Chapter 5 in particular goes into great detail, though it of course predates the Sioux City crash. >Also, have all these hydraulic problems been corrected, or does the >DC-10 still suffer from hydraulic problems even today? The DC-10 (and the MD-11) haven't "suffered" from them recently, though the problems still exist. Fundamentally, the DC-10's hydraulic system shortcomings as compared to the 747 and L-1011 are that there are only three, instead of four, giving less redundancy, and they tend to be routed together so that something which affect one probably will affect all three. Boeing and Lockheed (and I believe Airbus) used three hydraulic systems in any one area of the plane, providing the mandated redundancy, but used four overall so that a problem which caused the failure of all three systems in one part of the aircraft would still leave control in other areas via the intact fourth system. They also chose to route the three systems independently, again to minimize the possibility of complete failure. As mentioned, I don't believe the Paris crash involved the hydraulics, but it did lead McDonnell-Douglas to relocate the control cables and hydraulics from under the cabin floor, where the cables had been severed by a collapsing floor, to the top of the cabin. This simply left them vulnerable to a different failure mode -- demonstrated 15 years later when the fan on UA 232's #2 engine disintigrated and sent shrapnel through the top of the aft fuselage, including all three of those hydraulic lines. Clearly a more reasonable solution would have been to move only *some* of the lines after the Paris crash, but this was not done and has not been done since, though some check valves were added to minimize the impact. The vulnerable location of the hydraulics at the leading edge of the wing, instead of a mid-wing and/or trailing edge location as used by other manufacturers, precipitated the Chicago crash, and the lack of any mechanical locking mechanism to prevent uncommanded flap retrac- tion also played a significant part. (The largest blame was placed on American Airlines for improper maintenance practices, though the airframe certainly received its share of blame.) While I believe the locking mechanism was later added, nothing was done about the routing of the hydraulics. Indeed, American requested a modification kit to move the hydraulics and was willing to pay for it, but McDonnell- Douglas refused. I'll leave you to draw your own conclusions, but I certainly do not feel comfortable flying on the DC-10, even though I've flown on them many times. I fly SFO-ORD somewhat often and do my best to catch one of the 747 flights United offers or a 757. -- Karl Swartz |INet kls@ditka.chicago.com 1-415/854-3409 |UUCP uunet!decwrl!ditka!kls |Snail 2144 Sand Hill Rd., Menlo Park CA 94025, USA Send sci.aeronautics.airliners submissions to airliners@chicago.com From kls Wed Nov 18 22:58:17 1992 Newsgroups: sci.aeronautics.airliners Path: news From: Helen Trillian Rose Subject: Re: Boeing 747-300 References: Message-ID: Approved: kls@ohare.Chicago.COM X-Original-Message-Id: <199211182226.AA03203@rocza.eff.org> Sender: kls@ohare.Chicago.COM Date: Wed, 18 Nov 1992 17:26:36 -0500 Karl> == Karl Swartz [....] Karl> Mfr. Type MGTOW seating service Karl> Boeing 747-400 870,000 430 1989 Karl> MacDAC MD-11 618,000 250 1991 Karl> Airbus A-340 559,000 230 1993 Karl> Boeing 777 515,000 ~220 1995 I hadn't thought the B777 was going to be in between the 757 (~200) and 767 (~250) passengers. I thought it was going to fill the niche between the 767 and the 747 -- about the size of the old 747SP in number of seats. The 777 is a widebody as wide as the 747's (3-4-3 seating in economy) and was meant to fill the market left wide open by the L-1011 and DC-10 trijets (one big reason why it has optional folding wingtips: to fit into a DC-10 gate). Karl> In the 600+ passenger market, Boeing has talked about both Karl> further stretches of the 747 and an entirely new aircraft, Karl> sometimes using the N650 moniker. McDonnell-Douglas has most Karl> recently talked about the MD-12 -- once yet another stretch of Karl> the MD-11 -- as a new and much larger aircraft, also in the 600+ Karl> passenger category. McDonnell Douglas has put a hold on the MD-12 for lack of financing (lets face it, would *you* get into bed with a company that produced the DC-10?). Karl> Airbus has said that if there is demand and/or if Boeing builds Karl> such an aircraft, Airbus will build one too. The name A600 or Karl> maybe A2000 seems vaguely familiar though I can't locate any Karl> references. Only Airbus would create a plane just to compete with Boeing. Just imagine, Europe, your tax money is going to support the egos of a bunch of political types. And inciting fear in any airliner fan. What a worthwhile task. --Helen -- Helen Trillian Rose Electronic Frontier Foundation email eff@eff.org for EFF Info Kapor Enterprises, Inc. Flames to: Systems and Networks Administration women-not-to-be-messed-with@eff.org From kls Wed Nov 18 23:21:58 1992 Newsgroups: sci.aeronautics.airliners Path: news From: kls@ohare.Chicago.COM (Karl Swartz) Subject: Re: Boeing 747-300 References: Message-ID: Approved: kls@ohare.Chicago.COM Organization: Chicago Software Works X-Original-Message-ID: <1992Nov19.072012.14608@ohare.Chicago.COM> Sender: kls@ohare.Chicago.COM Date: Thu, 19 Nov 1992 07:20:12 GMT In article Helen Trillian Rose writes: Karl> == Karl Swartz Karl> Mfr. Type MGTOW seating service Karl> Boeing 747-400 870,000 430 1989 Karl> MacDAC MD-11 618,000 250 1991 Karl> Airbus A-340 559,000 230 1993 Karl> Boeing 777 515,000 ~220 1995 >I hadn't thought the B777 was going to be in between the 757 (~200) and >767 (~250) passengers. I thought it was going to fill the niche between >the 767 and the 747 -- about the size of the old 747SP in number of >seats. Sorry, that was a typo. The correct number should be 290. Note, though, that I said a *three* class configuration, since the topic was long-range aircraft. A 757 is in the 185 to 195 range with only two classes; 250 is the right ballpark for a two class 767-300. The smaller 767-200 is just over 200 with two classes, not much bigger than the 757. With three classes, a 767-200 is aroung 165 to 170 while the 767-300 is about 210 to 220. In the case of the 777, the only numbers I have handy are for United's two class configuration, which has 38+325 for a total of 363 seats. I tried to extrapolate from that number and the ratio of seats on United's two and three class 767-200s. >The 777 ... was meant to fill the market left wide open by the L-1011 >and DC-10 trijets (one big reason why it has optional folding wingtips: >to fit into a DC-10 gate). True, though it ended up significantly larger than either. (Using the United configs again, 363 seats vs. 298 on a DC-10-30 with a below-deck galley.) As for the foldings wingtips, nobody has yet ordered them. I wonder just what they intend to do with all those not-quite-big-enough gates?! >McDonnell Douglas has put a hold on the MD-12 for lack of financing >(lets face it, would *you* get into bed with a company that produced the >DC-10?). I doubt the DC-10 has much to do with it, since the MD-11 has been selling well enough. They simply found themselves in a Catch-22: they couldn't raise the cash without any firm orders and couldn't get any firm orders without a reasonable expectation of the financing falling into place. Their poor financial condition of course means they can't finance it themselves as Boeing could, which greatly complicates the whole matter. Actually, even if they had the financing they probably couldn't get the orders given the current economic situation, and this is exactly what they've said in their announcement of delaying the MD-12. >Only Airbus would create a plane just to compete with Boeing. Hmmm ... seems to me McDonnell-Douglas with the DC-10 and Lockheed with the L-1011 were pretty bullheaded about going ahead simply to compete with each other, knowing full well that with the orders split neither one could really succeed! 8-) -- Karl Swartz |INet kls@ditka.chicago.com 1-415/854-3409 |UUCP uunet!decwrl!ditka!kls |Snail 2144 Sand Hill Rd., Menlo Park CA 94025, USA Send sci.aeronautics.airliners submissions to airliners@chicago.com From kls Thu Nov 19 03:44:49 1992 Newsgroups: sci.aeronautics.airliners Path: news From: drinkard@bcstec.ca.boeing.com (Terrell D. Drinkard) Subject: Re: Boeing 747-300 References: Message-ID: Approved: kls@ohare.Chicago.COM Organization: Boeing X-Original-Message-Id: <9211190615.AA02738@bcstec.ca.boeing.com> Sender: kls@ohare.Chicago.COM Date: Wed, 18 Nov 92 22:15:02 PST In article jerry@telecom.ksu.edu (Jerry And erson) writes: >Is the Boeing 747-300 the largest commercial passenger aircraft >in the world? Is the -300 the latest version, or are there >newer, possibly larger stretched versions of the 747? No, actually, I believe the 747-400 is currently the largest production passenger aircraft in the world. Wingspan of 211' and max takeoff weight of 870,000lb, I believe. Not that I do Everett products... :-) >I've heard Boeing, McDonnell-Douglas and Airbus all have plans >in the works for Really Big Planes in the 600-700 passenger, >7500-8000 mile range. Does anyone know if these planes are >really going to get built, or is this the usual "if we can >pre-sell a couple of hundred, maybe we'll really build it?" >Does anyone have model numbers, specs (passengers, range), or >projected delivery dates? The Boeing and the Airbus offerings in this market seem to both hover around 600 seats and 7,000 mile range. Takeoff weights in the million pound plus range. The anticipated market, as described by John Hayhurst, Director of New Large Airplane Division, is only a couple hundred airplanes TOTAL. From my knothole, it looks like a prestige fight. But there are a lot of interesting questions that must be answered before anyone will build one of these monsters. > These numbers come from memory, and >I have no faith in them at all: Rightly so. :-) > Company Model Delivery > ------- ------ -------- > McDonnell-Douglas MD-11 1993 I think you'll find that the MD-11 has been delivering since 1990. Very nice airplane overall. My writer flew on one to Europe, he liked it better than the 767. :-) It is probably the MD-12 that you have in mind. It is on hold until somebody comes up with $2Begabucks to finance the development. > Airbus AE-400 1994 You may be thinking of the A-340, which should certainly be delivering by then. I think it is the A-350 that is the number being kicked about for their UHCA (Ultra High Capacity Aircraft). > Boeing 777 1995-6 Pretty close. March '95 sticks in my mind for some reason, but I'm not that familiar with the 777's schedule. My wife probably knows... :-) Terry drinkard@bcstec.ca.boeing.com From kls Thu Nov 19 21:46:59 1992 Newsgroups: sci.aeronautics.airliners Path: news From: Pete Mellor Subject: Re: TV programme on 777 Message-ID: Approved: kls@ohare.Chicago.COM X-Original-Message-Id: <3891.9211191222@csrsun8.cs.city.ac.uk> Sender: kls@ohare.Chicago.COM Date: Thu, 19 Nov 92 12:22:53 GMT Jonathan, > Would anyone who saw the whole programme like to provide a > brief summary in this forum? (A slightly more temperate response to your question! :-) The programme concentrated mainly on the design of the airframe, and the use of CAD systems to do this. It described how the structural calculations and spatial arrangement of components could be handled using 3-D movable graphics. The design system was intended to be "paperless", with electronic transfer of designs between engineers' workstations, instead of blue-prints being dropped in in-trays. Examples of the sort of problems they were shown tackling were "What weight of metal can we drill out of this structure and still leave it strong enough to bear the stress?", and "When the kitchen door opens, does it hit the knees of the first-cklass passengers?". There was quite a bit of time devoted to the design of the doors, with a management requirement to have them all identical to cut production costs, and the design problems this entailed. The poor old designer took three months to solve this one first time round, under pressure from what the manager described as a "management ploy". ("Well, if *you* can't do it, which consultant do you suggest we bring in to solve it for you?") On the second version of the design, the problem of door uniformity was solved in a few weeks, and by the third iteration it was down to a few days. It definitely did have the feel of a "Boeing commercial" about it, with lots of "gee-whizz" shots of designers manipulating computer graphics, and anecdotes of the "Yes, of course we had problems, but just look how we learned to overcome them!" variety. (See the door problem above.) It was an interesting programme, but I was disappointed to find only one passing reference to the flight control systems, having originally watched it in the hope of learning about Boeing's approach to fly-by-computer. Pete ------------------------------------------------------------------------------- From kls Thu Nov 19 21:47:01 1992 Newsgroups: sci.aeronautics.airliners Path: news From: hoyme@src.honeywell.com (Ken Hoyme) Subject: Re: Boeing 747-300 References: Message-ID: Approved: kls@ohare.Chicago.COM Organization: Honeywell Systems & Research Center, Mpls. MN, USA. X-Original-Message-Id: Sender: kls@ohare.Chicago.COM Date: Thu, 19 Nov 1992 15:35:39 GMT (I posted this yesterday, but our mailer had problems with finding where to send for this moderated newsgroup. I have been told this has been fixed. I see that other follow-ups have occured as well, but there is some information in here that wasn't covered. Rather than editing this, I am sending it on as originally written.) In article kls@ohare.Chicago.COM (Karl Swartz) writes: > In article jerry@telecom.ksu.edu (Jerry Anderson) writes: > Unless the Russians have something which I've missed, the 747-400 is > easily the largest commercial passenger aircraft in terms of number of > seats and payload. Its range is also the greatest of anything now in > service, though the Airbus A340 will exceed it once it enters service > next year. According to the "Commercial Airliners of the World" section of the 21-27 October 1992 issue of Flight International, the largest Russian transport is the Ilyushin II-86 Camber with a maximum seating of 350. I noticed that the max. seating estimates for the other airplanes were for sardine configurations, so I have to assume that this is not a 3-class estimate. (Ex: 747-400 with max seating of 660?? That's cramped!) >> McDonnell-Douglas MD-11 1993 >> Airbus AE-400 1994 >> Boeing 777 1995-6 > All three of these are much smaller than what you're thinking of. > Here are the important parameters for these three plus the 747-400 > for comparison. Seating is for a "typical" three-class cabin and > service is the date of first service; MGTOW is in US pounds. > Mfr. Type MGTOW seating service > Boeing 747-400 870,000 430 1989 > MacDAC MD-11 618,000 250 1991 > Airbus A-340 559,000 230 1993 > Boeing 777 515,000 ~220 1995 My data for the 777-200 is 3-class seating of 320, with a stretch version planned with 3-class seating in the 360-390 range. United ordered the 320 seat version according to AvWeek Oct. 22, 1990. According to AvWeek Nov. 4, 1991, the A-340-300 will have a 3-class seating configuration of 295, and the A340-200 will be shorter with 262 seats. > In the 600+ passenger market, Boeing has talked about both further > stretches of the 747 and an entirely new aircraft, sometimes using > the N650 moniker. McDonnell-Douglas has most recently talked about > the MD-12 -- once yet another stretch of the MD-11 -- as a new and > much larger aircraft, also in the 600+ passenger category. Airbus > has said that if there is demand and/or if Boeing builds such an > aircraft, Airbus will build one too. The name A600 or maybe A2000 > seems vaguely familiar though I can't locate any references. Boeing is considering three configurations for their "New Large Airplane (NLA)" A 747 stretch, a double deck 747 and a totally new double decker. 3 class seats range from 484-612. See AvWeek Jan 6, 1992 for a description of these options. An Oct. 28, 1991 AvWeek article covers Airbus's studies on large airplane configurations. That article confusingly talks about the ASX-700, but shows an artists concept with an A2000 on the tail. 600 3-class seats in a double deck configuration. I have also heard that Boeing will build theirs if Airbus launches. I suspect both are eying the market and hoping to delay the investment as long as possible, given the current economic climate. If one decides the go-ahead, the other will have to launch defensively to prevent the other from capturing the market. I hope this won't be another fiasco like the DC-10/L-1011 developments, where each captured enough of the market to keep the other from making any money. Lockheed got out of the business, and some have questioned whether MDAC has ever really recovered from that. Ken --- Ken Hoyme Honeywell Systems and Research Center (612)951-7354 3660 Technology Dr., Minneapolis, MN 55418 Internet: hoyme@src.honeywell.com From kls Thu Nov 19 21:47:01 1992 Newsgroups: sci.aeronautics.airliners Path: news From: bentson@CS.ColoState.EDU (Randolph Bentson) Subject: Re: TV programme on 777 References: Message-ID: Approved: kls@ohare.Chicago.COM Organization: Colorado State University, Computer Science Department X-Original-Message-ID: Sender: kls@ohare.Chicago.COM Date: Thu, 19 Nov 1992 16:30:31 GMT In article lomasm@t9.cs.man.ac.uk (Martin Lomas) writes: >... >Fly by wire briefly explained. >... >Any other critics out there? > > >Martin. > -NOT FROM THE SHOW- Boeing is _very_ reluctant to use fly-by-wire. Management trusts computer solutions no more than members of this forum. I got the impression that this system has a pilot override as part of it's basic design. (A sort of "do what I say, not what you think I want" mode.) One interesting feature is the networking of non-flight related computers. Planes will have an internet that will also link link to ground stations, satellites, and other planes. They will have the ability to do significant book-work while in the air. Flight crews will be able to order maintenance and consumables, weather and traffic information can be exchanged, etc. It's likely there will be a network for passangers-- a big step forward from airphone. Boeing recently moved a great number of folks (on the order of 5000) that were scattered south of Seattle to a new facility in Everett, Washington (about 50 miles north). They did it on the week-end so as not to disrupt anybody's work. They figure this will enhance communication among folks working. -- Randolph Bentson Colorado State University bentson@CS.ColoState.Edu Computer Science Department 303/491-5792 Ft. Collins, CO 80523 From kls Thu Nov 19 22:46:21 1992 Newsgroups: sci.aeronautics.airliners Path: news From: Christopher Davis Subject: Re: hydraulic problems with DC-10's?? Message-ID: Approved: kls@ohare.Chicago.COM X-Original-Message-Id: <199211200503.AA03970@loiosh.eff.org> Sender: kls@ohare.Chicago.COM Date: Fri, 20 Nov 1992 00:03:12 -0500 Karl> == Karl Swartz Karl> Probably the best general interest discussion of the DC-10 and Karl> all its problems is in The Sporty Game, by John Newhouse (Alfred Karl> A. Knopf, New York, 1982). Chapter 5 in particular goes into Karl> great detail, though it of course predates the Sioux City crash. Other good books on the subject include Moira Johnston's _The Last Nine Minutes_ (which, being 1976, only discusses the Turkish Airlines Ermenonville crash and the Windsor "incident" which foreshadowed it) and John Nance's _Blind Trust_ (1986, also predating Sioux City). (The latter discusses many other air safety issues and incidents, including the Air Florida crash in Washington, DC in 1982.) _The Sporty Game_ tends to show its age in other areas as well; the dire predictions of market failure for the 747, 757, and 767 have not quite been borne out by intervening events :) From kls Fri Nov 20 02:04:38 1992 Newsgroups: sci.aeronautics.airliners Path: news From: Chijioke.Anyanwu@brunel.ac.uk (Chijioke D Anyanwu) Subject: Re: TV programme on 777 References: Message-ID: Approved: kls@ohare.Chicago.COM Organization: Brunel University, West London, UK X-Original-Message-Id: <2666.9211200818@gassendi.brunel.ac.uk> X-Original-Message-ID: Sender: kls@ohare.Chicago.COM Date: Fri, 20 Nov 1992 08:17:55 GMT In article , lomasm@t9.cs.man.ac.uk (Martin Lomas) gives an excellent summary of last Sunday's Equinox on the 777 - I definitely couldn't have done better. He says >Customers allowed into (some) of the Boeing meetings to discuss the >777's design and to suggest mods. 'Open management' strategy. Specifically Lord King: BA will be a launch customer as will Emirates which is already advertising the fact. One thing which he omitted and which I found quite interesting was the amount of attention paid to the design of the toilet. Apparently, banging toilet seats tend to cause some passengers some amount of distress (thoughts of bombs going off?) and so a virtually noiseless toilet seat and cover were designed. Although as Martin pointed out FBW safety issues were not covered (something which I had rather expected in view of all the controversy surrounding the A320), the programme really was a fascinating insight into modern aircraft design and development. Chijioke. From kls Sat Nov 21 03:55:53 1992 Newsgroups: sci.aeronautics.airliners Path: news From: Mohamed Ishaq Subject: Info on the El-Al Plane crash Message-ID: Approved: kls@ohare.Chicago.COM Reply-To: Mohamed Ishaq X-Original-Message-Id: Sender: kls@ohare.Chicago.COM Date: Fri, 20 Nov 1992 12:50:48 -0600 (CST) I am looking for information on the El-Al 747-200 plane crash in the Netherlands on Oct 4, 1992. I am planning on writing a report on the application of non-destructive testing to engine mount fuse pins and am analyzing the Boeing SB related to this crash and the China Airlines one earlier this year. I found your address on the user-net : sci.aeronautics.airliners. Once I complete my report I will be more than happy to post it the network. Thanks Mohamed Ishaq - mishaq@ccwf.cc.utexas.edu P.O. Box 8171 Austin, TX 78713-8171 Tel: 512-472-9290 P.S. I kind of need this info ASAP From kls Sat Nov 21 03:55:56 1992 Newsgroups: sci.aeronautics.airliners Path: news From: Robert Dorsett Subject: Re: TV programme on 777 Message-ID: Approved: kls@ohare.Chicago.COM Reply-To: rdd@rascal.ics.utexas.edu X-Original-Message-Id: Sender: kls@ohare.Chicago.COM Date: Fri, 20 Nov 92 15:46:39 CST In article bentson@CS.ColoState.EDU (Randolph Bentson) writes: >Boeing is _very_ reluctant to use fly-by-wire. Management >trusts computer solutions no more than members of this forum. I >got the impression that this system has a pilot override as part >of it's basic design. (A sort of "do what I say, not what you >think I want" mode.) As I understand it, the FBW system is the only way the pilots can signal the actuators. Boeing is simply providing a "conventional" control law and interface, with "protections" that can be over-ridden by the pilot, if necessary. Redundancy/backup is at the hardware level, not in alternate select modes. So, rather than a simple joystick, Boeing's "simulating" a conventional interface, with feedback, in the cockpit cab: each control column inter- connected with the other, each providing tactile feedback. The FBW is there, one way or the other. On the other hand, I do think it's a positive step that Boeing's not "re- writing" the book by offering *artificial* control laws, as Airbus is doing. Thus, to override the protections, the pilots just need to push or pull *harder,* or click an overrride button: they don't have to deal with or anticipate the effects of *four* distinct control law modes, and the many permutations within each mode, depending upon system status, as is the case with the A3[2-4]0. --- Robert Dorsett rdd@cactus.org ...cs.utexas.edu!cactus.org!rdd From kls Sat Nov 21 03:55:57 1992 Newsgroups: sci.aeronautics.airliners Path: news From: sfg2483@uxa.cso.uiuc.edu (sfg2483 ) Subject: Manufacturer responsibility? Message-ID: Approved: kls@ohare.Chicago.COM Organization: University of Illinois at Urbana X-Original-Message-Id: Sender: kls@ohare.Chicago.COM Date: Sat, 21 Nov 1992 04:24:47 GMT Does anyone know if a manufacturer (e.g., Boeing) is responsible financially for the results of an airplane crash if the crash is proven to be caused by a faulty part it made? (E.g., a bad design of pins in the 747). sfg2483@uxa.cso.uiuc.edu From kls Sat Nov 21 03:55:59 1992 Newsgroups: sci.aeronautics.airliners Path: news From: rdd@rascal.ics.utexas.edu (Robert Dorsett) Subject: The DC-10 Case Message-ID: Approved: kls@ohare.Chicago.COM X-Original-Message-Id: <9211210611.AA25114@rascal.ics.utexas.edu> Sender: kls@ohare.Chicago.COM Date: Sat, 21 Nov 92 00:11:55 CST Newsgroups: sci.aeronautics,rec.travel.air From: rdd@cactus.org (Robert Dorsett) Subject: The DC-10 Case (non-review) Date: Fri, 9 Oct 1992 06:16:16 GMT Ran across this. It looks like a nice little anthology, covering many aspects of the DC-10. Probably worth it for the NTSB reports alone ($20 each from NTIS). I haven't read the more "thematic" articles, though, and no endorsement is meant or implied. -------------------- Title: The DC-10 Case Subtitle: A study in applied ethics, technology, and society. Editors: John H. Fielder and Douglas Birsch Publisher: State University of New York Press Date: 1992 Pages: 346 ISBN: 0-7914-1087-0 (hardcover) 0-7914-1088-9 (paper) Illustrated. CONTENTS: Preface Introduction Ethical Analysis of Case Studies/John H. Fielder HISTORY AND EARLY WARNINGS 1. Regulatory and Institutional Framework 2. High Risks, Sinking Fortunes/John Newhouse 3. Floors, Doors, Latches and Locks/John Fielder 4. The 1970 Ground Testing Incident/Paul Eddy, Elaine Potter, Bruce Page 5. National Transportation Safety Board Report on the Windsor Incident 6. The Applegate Memorandum/Paul Eddy, Elaine Potter, Bruce Page 7. Fat, Dumb and Happy: The Failure of the FAA/Paul Eddy, Elaine Potter, Bruce Page 8. Compliance with Service Bulletin SB 52-37 9. Conclusions of the US Senate Oversight Hearings and Investigation of the DC-10 Aircraft THE 1974 PARIS CRASH 10. French Government Report on the 1974 Paris Crash 11. Engineers Who Kill: Professional Ethics and the Paramountcy of Public Safety/Kenneth Kipnis 12. Whistleblowing, Ethical Obligation, and the DC-10/Douglas Birsch 13. What is Hamlet to McDonnel Douglas or McDonnell Douglas to Hamlet?: DC-10/Peter French Commentary/Homer Stewell 14. Statement of John C. Brizendine, President, Douglas Aircraft Company, McDonnell Douglas Corporation THE 1979 CHICAGO CRASH 15. National Transportation Safety Board Report on the 1979 Chicago Crash 16. The DC-10: A Special Report/McDonnell Douglas 17. Two Models of Professional Responsibility/Martin Curd and Larry May THE 1989 SIOUX CITY CRASH 18. National Transportation Safety Board Report on the 1989 Sioux City Crash 19. The 1989 Sioux City Crash/John Fielder 20. Statement of Ralph Nader 21. Aviation Safety: Management Improvement Needed in FAA's Airworthiness Directive Program 22. The FAA, the Carriers, and Safety/Charles Perrow 23. International Airline Passengers Association Critique of the DC-10 24. Moral Responsibility for Engineers/Kenneth D. Alpern Commentary/Andrew Oldenquist Commentary/Samuel C. Florman Select Bibliography IEEE Code of Ethics Index Back Cover: "Designed as a textbook for courses in ethics, this book privdes the material needed to understand the accidents in which more than 700 people were killed-- accidents that many believe were the result of unethical actions and inactions by individuals, organizations, and government agencies. An introduction to ethical analysis and discussions of the ethical responsibilities involved are also provided. The case study offers material for a sustained inquiry into every level of ethical responsiblity reflecting the rich complexity of actual events. "_The DC-10 Case_ presents these issues through a collection of original and published articles, excerpts from official accident reports, congressional hearings, and other writings on the DC-10. The authors allow the readers to examine the ethical issues of airline safety as they actually occur, taking account of the circumstances in which they arise. "John H. Fielder is is Professor and Douglas Birsch is Assistant Professor of Philosophy at Villanova University." --- Robert Dorsett rdd@cactus.org ...cs.utexas.edu!cactus.org!rdd From kls Sun Nov 22 14:47:54 1992 Newsgroups: sci.aeronautics.airliners Path: news From: Pete Mellor Subject: Re: Airline Software-safety database (RISKS-14.08) Message-ID: Approved: kls@ohare.Chicago.COM X-Original-Message-Id: <4664.9211221721@csrsun8.cs.city.ac.uk> Sender: kls@ohare.Chicago.COM Date: Sun, 22 Nov 92 17:21:22 GMT Dave "Van Damme" Ratner writes in RISKS-14.08: > I am posting this for Robert Ratner, Ratner Associates Inc, which does > international consulting in air-traffic control and aviation safety issues. > He is looking for a public-accessible data base on software-related incidents > in this area. Email correspondence can be sent to me at ratner@cs.ucla.edu. > Thanks. Dave "Van Damme" Ratner ratner@cs.ucla.edu In my experience, all major manufacturers of software keep databases of incidents reported by users of their software and the faults ("bugs") which give rise to those incidents. I know for a fact that IBM, ICL, DEC, Unisys (or whatever it is now), and Sun all do this. Such a database is essential to their efforts to improve the quality of their software by identifying and fixing bugs, and to reduce their maintenance workload by informing customers about known problems so that repeated reports are suppressed. The interesting phrase is "public-accessible". If you are a customer of a large manufacturer of system or application software, you will almost certainly have access to the *relevant* parts of the database (those which concern the products you have bought). This will be provided either on-line, or as printed or micro-fiche extracts, updated on a regular basis. The other interesting phrase is "in this area" (i.e., of air-traffic control and aviation safety). The users of safety critical on-board avionics software are the companies that buy the aircraft. They are provided with regular information about all sorts of design glitches in the aircraft they have bought, including those in the software. Such information is provided in the form of "OEBs" (Operating Engineering Bulletins), which are distributed to the flight crews. Information about software faults in safety-critical avionics systems *must*, therefore, be kept on a database somewhere. These databases are public in the sense that any pilot on that type of aircraft would have access, but Joe Public (as far as I know) does not. Incidents in flight must (or should) be reported via offical channels by the crews. These reports drive the manufacturers' quality improvement programmes. After the fault which caused an incident has been diagnosed, it may result in an OEB or similar, and in a modification. Databases of such incident reports are not generally widely accessible. Published reports sometimes appear, however. In addition, there are channels for anonymous reporting of incidents. In the UK, "CHIRP" is such a forum. In the US, I believe the FAA used to run such a scheme, but it was compromised when the guarantee of anonymity was removed. For further information I suggest you contact ALPA. Given the increasing use of safety-critical software, a central database for each major application area would be highly desirable, to say the least. Obviously, sensitive issues of commercial confidentiality are involved. In particular, it may be difficult to obtain corresponding figures for the operating time so as to be able to estimate reliability, and it may be difficult to correlate incidents with faults, and so determine which incidents are due to software. I stand to be corrected if anyone *does* know of an official channel for public access to flight incident and system fault reports. Regarding ATC incidents, again I am certain that these are recorded, but access is not likely to be easy. Peter Mellor, Centre for Software Reliability, City University, Northampton Sq., London EC1V 0HB, Tel: +44(0)71-477-8422, JANET: p.mellor@city.ac.uk ----------------------------------------------------------------------------- From kls Mon Nov 23 04:38:46 1992 Newsgroups: sci.aeronautics.airliners Path: news From: Dave Williamson Subject: Re: hydraulic problems with DC-10's?? Message-ID: Approved: kls@ohare.Chicago.COM Reply-To: <199211200503.AA03970@loiosh.eff.org> Organization: AT&T BL0512310 X-Original-Message-Id: <9211222144.AA09019@conch> Sender: kls@ohare.Chicago.COM Date: Sun, 22 Nov 92 16:44:33 EST sdl@linus.mitre.org writes > Perhaps now I can get a more definitive answer to the following: > Some of my pilot friends have accused the DC-10 as having a > particularly bad history of hydraulic problems (which have contributed > to a few crashes). But do the statistics really support the notion > ... I don't have anything statistical (others have covered that quite admirably) but I had a similar experience. Being a pilot myself (just small stuff), I tend to meet a lot of airline pilots. A former instructor of mine introduced me to a friend of his who was currently flying for a major airline. This guy referred to the DC-10 as the "Death-Cruiser 10" and said he wouldn't fly in it under any circumstances, especially not as a pilot. My personal opinion is that flying is still safer than other forms of transportation, DC-10 accidents notwithstanding. While I prefer flying in a B7[456]7 (when I can't fly myself), I have no problem getting into a DC-10 if that is what is at the gate. -------------------------------------------------------------------------- >| David M. Williamson, d.m.williamson@att.com >| Commercial, ASEL, IA --> --> --> | --- ====== --- --- --- --- --- --- >| Proud part owner of Archer N7185F >| -------------------------------------------------------------------------- From kls Tue Nov 24 00:34:06 1992 Newsgroups: sci.aeronautics.airliners Path: news From: weiss@mott.SEAS.UCLA.EDU (Michael Weiss) Subject: Re: Boeing 747-300 References: Message-ID: Approved: kls@ohare.Chicago.COM Organization: SEASnet, University of California, Los Angeles X-Original-Message-Id: <8731@lee.SEAS.UCLA.EDU> Sender: kls@ohare.Chicago.COM Date: 24 Nov 92 02:47:07 GMT In article jerry@telecom.ksu.edu (Jerry Anderson) writes: >Is the Boeing 747-300 the largest commercial passenger aircraft >in the world? Is the -300 the latest version, or are there >newer, possibly larger stretched versions of the 747? Well, Boeing no longer makes the -300; I personally saw the last -300 being built in mid-September of 1991. The -300 has been replaced by the -400, which has few fuselage changes. The upper deck is the same size. Basically, the only "major" change is the addition of upper-surface winglets. In my aero classes, we were taught that winglets are supposed to reduce the trailing vortices and downwash from the wings. However, according to my cousin, who used to work for Lockheed's Skunk Works, the winglets have a cost in drag that is roughly equivalent to the gain, and therefore is more a marketing ploy than anything else. Go figure. >I've heard Boeing, McDonnell-Douglas and Airbus all have plans >in the works for Really Big Planes in the 600-700 passenger, >7500-8000 mile range. Does anyone know if these planes are >really going to get built, or is this the usual "if we can >pre-sell a couple of hundred, maybe we'll really build it?" Airbus is truly working on the plane, and hopes to knock Boeing out of the 747 sales. However, Boeing has a history of being the best aircraft in the world in terms of maintenance; Airbus apparently makes planes that are almost as difficult to repair and inspect as McDonnell-Douglas. McDonnell-Douglas has basically dropped out of that race, to my knowledge, apparently because they require such a large amount of capital. >Does anyone have model numbers, specs (passengers, range), or >projected delivery dates? These numbers come from memory, and >I have no faith in them at all: > Company Model Delivery > ------- ------ -------- > McDonnell-Douglas MD-11 1993 Try 1992. The MD-11 was certified in October of 1991. I saw about a dozen of them at DFW this summer. They are pretty much a DC-10 with upper and lower winglets. > Airbus AE-400 1994 Sounds about right. Well, they were talking about '93, but that likely means 1994. Supposedly, it will compete directly with the 747. > Boeing 777 1995-6 I would guess sooner, simply based upon the information I have heard. I could very well be wrong, though. -- \ | | | | | | | | | | | | | | | | | | | | | | | | | | | | | | | | | | | | | | / - Michael weiss@seas.ucla.edu | School of Engineering & Applied Science - - Weiss izzydp5@oac.ucla.edu | University of California, Los Angeles - / | | | | | | | | | | | | | | | | | | | | | | | | | | | | | | | | | | | | | | \ From kls Tue Nov 24 03:54:40 1992 Newsgroups: sci.aeronautics.airliners Path: news From: gjacobs@qualcomm.com (Gary Jacobs) Subject: Emergency Oxygen Masks Message-ID: Approved: kls@ohare.Chicago.COM Summary: How often is testing done and what is failure rate? Organization: Qualcomm, Inc., San Diego, CA X-Original-Message-Id: Sender: kls@ohare.Chicago.COM Date: Tue, 24 Nov 1992 07:43:50 GMT Having just been involved in a flight where the oxygen masks were dropped, I'm curious what the test criteria are and how often is it done? I'd also like to know what the failure rate is? On my flight, America West on a Boeing 737, an oil seal supposedly failed on the APU for the air conditioning system. This sent oily smoke in the cabin. I assume the cockpit crew decided that breathing that air wasn't good for the passengers and dropped, or should I say tried to drop, the emergency oxygen masks. I'd say that 1/4 of the overhead doors did not open until they were pried open by a passenger and then a lot of the masks did not seem to supply oxygen even after following the "pull hard to start the flow of oxygen" instructions. I looked at my mask which didn't work and could not tell what the "pull hard" did to start the oxygen. Gary Jacobs gjacobs@qualcomm.com From kls Tue Nov 24 03:54:42 1992 Newsgroups: sci.aeronautics.airliners Path: news From: kls@ohare.Chicago.COM (Karl Swartz) Subject: Re: Boeing 747-300 References: Message-ID: Approved: kls@ohare.Chicago.COM Organization: Chicago Software Works X-Original-Message-ID: <1992Nov24.110151.26562@ohare.Chicago.COM> Sender: kls@ohare.Chicago.COM Date: Tue, 24 Nov 1992 11:01:51 GMT In article drinkard@bcstec.ca.boeing.com (Terrell D. Drinkard) writes: >In article jerry@telecom.ksu.edu (Jerry And >>I've heard Boeing, McDonnell-Douglas and Airbus all have plans >>in the works for Really Big Planes in the 600-700 passenger, >>7500-8000 mile range. >The Boeing and the Airbus offerings in this market seem to both hover >around 600 seats and 7,000 mile range. Takeoff weights in the million >pound plus range. The anticipated market, as described by John Hayhurst, >Director of New Large Airplane Division, is only a couple hundred airplanes >TOTAL. From my knothole, it looks like a prestige fight. That's an *awful* lot of cash to dump down a hole simply for bragging rights. I suppose that's part of "being sporty" but there's also a real market there -- the Pacific Rim, which is where nearly all the growth is in the airline industry and which requires those kind of range figures. The load potential is there too, if not now then well well within the next 10 to 20 years. One aircraft that could punch a major hole in this market would be the next generation supersonic transport. (HSCT? I can't pick the right acronym out of my bowl of alphabet soup today ...) *If* built, and at least for now that is a very big if, this too would be aimed directly at the Pacific Rim market. The studies I've seen for this bird seem to be aiming at the mainstream market and not just a very tiny high- priced market like the Concorde. -- Karl Swartz |INet kls@ditka.chicago.com 1-415/854-3409 |UUCP uunet!decwrl!ditka!kls |Snail 2144 Sand Hill Rd., Menlo Park CA 94025, USA Send sci.aeronautics.airliners submissions to airliners@chicago.com From kls Tue Nov 24 03:54:43 1992 Newsgroups: sci.aeronautics.airliners Path: news From: kls@ohare.Chicago.COM (Karl Swartz) Subject: Re: Boeing 747-300 References: Message-ID: Approved: kls@ohare.Chicago.COM Organization: Chicago Software Works X-Original-Message-ID: <1992Nov24.113158.26665@ohare.Chicago.COM> Sender: kls@ohare.Chicago.COM Date: Tue, 24 Nov 1992 11:31:58 GMT In article weiss@mott.SEAS.UCLA.EDU (Michael Weiss) writes: >Well, Boeing no longer makes the -300; I personally saw the last -300 being >built in mid-September of 1991. The -300 has been replaced by the -400 ... [ Moderator's note: When I started reading this I thought "oh no, yet another answer to the same question ... time to play moderator and turn on the squelch." But there were some interesting points further down. If you think we're beating a dead horse or would somehow like to see the thread split, please send suggestions to me at airliners-admin@chicago.com. ] >In my aero classes, we were taught that winglets are supposed to reduce the >trailing vortices and downwash from the wings. However, according to my >cousin, who used to work for Lockheed's Skunk Works, the winglets have a cost >in drag that is roughly equivalent to the gain, and therefore is more a >marketing ploy than anything else. I've seen this comment before, either on sci.aeronautics or perhaps on rec.aviation. Would anyone care to provide a more scientific discourse on the subject for the benefit of the rest of us? >Airbus is truly working on the plane, and hopes to knock Boeing out of the >747 sales. However, Boeing has a history of being the best aircraft in the >world in terms of maintenance; Airbus apparently makes planes that are almost >as difficult to repair and inspect as McDonnell-Douglas. I've heard some comments about Airbus maintenance being exorbitantly expensive. In particular, one comment I heard was that they are very unforgiving about substitution of equivalent parts and gold-plate the prices of Genuine Airbus Parts. I have not previously encountered negative comments regarding McDonnell-Douglas products in this context, however, and in fact have heard that the DC-10 is rather well-liked because it's somewhat like a big Chevy V-8 -- solid, and easy to fix when it breaks. (Problems with the design of the hydraulics notwithstanding.) Seems to me that Lockheed, the L-1011 in particular but perhaps the Electra in its time as well, tended toward somewhat more finicky products that compensated by giving better performance. Again, any more concrete comments on the subject would be welcomed. >McDonnell-Douglas has basically dropped out of that race, to my knowledge, >apparently because they require such a large amount of capital. With regard to the MD-12, MacDAC seems to remain in the race nearly as much as Boeing and Airbus, though their ability to carry through with an actual aircraft is certainly less certain given their finances. In any case all three are paper planes until the airlines get themselves into better financial shape. > Airbus AE-400 1994 > >Sounds about right. Well, they were talking about '93, but that >likely means 1994. Supposedly, it will compete directly with the 747. That would be the A-340, which is well along in its test program and looks likely to make its scheduled first delivery (to Lufthansa) in the first quarter of 1993. It competes with the 747 in the sense that it is a long-range aircraft, in fact exceeding the range of the 747-400 by a little bit, but it's somewhat smaller, on the order of two-thirds the size. In that sense it competes more closely with the MD-12. >> Boeing 777 1995-6 > >I would guess sooner, simply based upon the information I have >heard. I could very well be wrong, though. It is indeed due the first half of 1995. This seemed an inordinately long gestation, but at launch time Boeing still had its hands full with the 747-400, and was also painfully aware of the delays in the 747-400 program due to an over-ambitious schedule. They simply did not have the resources to commit to an earlier delivery. The extra time has not at all been leisurely, however. Boeing is using many new design techniques with the 777, for example doing all the mockups in computers. (Actually a mockup *was* built of the nose section, but more as a check on the computer models rather than of the fit of the parts.) There is also an aggressive commitment to delivering an aircraft that's ready for service from day one, without a substantial period of teething problems in operation. This is an area of some controversy in that they are striving for ETOPS rating at initial delivery. -- Karl Swartz |INet kls@ditka.chicago.com 1-415/854-3409 |UUCP uunet!decwrl!ditka!kls |Snail 2144 Sand Hill Rd., Menlo Park CA 94025, USA Send sci.aeronautics.airliners submissions to airliners@chicago.com From kls Tue Nov 24 03:54:43 1992 Newsgroups: sci.aeronautics.airliners Path: news From: kls@ohare.Chicago.COM (Karl Swartz) Subject: The Sporty Game -- Boeing 757 References: Message-ID: Approved: kls@ohare.Chicago.COM Organization: Chicago Software Works X-Original-Message-ID: <1992Nov24.115042.26779@ohare.Chicago.COM> Sender: kls@ohare.Chicago.COM Date: Tue, 24 Nov 1992 11:50:42 GMT In article Christopher Davis writes: >Karl> == Karl Swartz > Karl> Probably the best general interest discussion of the DC-10 and > Karl> all its problems is in The Sporty Game, by John Newhouse (Alfred > Karl> A. Knopf, New York, 1982). >_The Sporty Game_ tends to show its age in other areas as well; the dire >predictions of market failure for the 747, 757, and 767 have not quite >been borne out by intervening events :) The 757 was doing rather weakly for quite a while, however. The huge orders in the past few years from American, United, and United Parcel have contributed mightily to the 757's success. I recently re-read this book and one thing I found fascinating was the discussion of the 757 and how it was the wrong aircraft -- it should have had about 30 fewer seats, which is what everyone wanted. Everone except British Airways, that is, and Boeing wanted desperately to sell to BA in the hopes of keeping the UK out of Airbus. They won the battle, as it were, but lost the war. In more ways than one, since not only did the UK link up with Airbus (despite BA's purchase of the 757) but Boeing was left without a well-positioned replacement for the 727 ... and Airbus *did* develop one, in the form of the A-320. All of this was particularly interesting as I was reading it right as United announced their A-320 order, rejecting Boeing's offerings as either too big (the 757) or inadequate on a variety of counts (the 737-400) for the intended job of replacing the 727-200. Fascinating to see how decisions made 15 years ago are still so clearly relected in today's market. I'm *still* surprised that Boeing hasn't made much noise (maybe none) about plugging this obvious hole by offering a 757-100 or whatever -- a shortened 757 like the original proposal and a real replacement for the 727-200. Even with United it never seemed to come up, instead all the discussion focussing on a massively stretched and pulled and re- designed 737-600. True, a 757 is more expensive (~ $45 million versus $30 - 35 million) but the changes embodied in the 737-600 would surely have added tremendously to the price. -- Karl Swartz |INet kls@ditka.chicago.com 1-415/854-3409 |UUCP uunet!decwrl!ditka!kls |Snail 2144 Sand Hill Rd., Menlo Park CA 94025, USA Send sci.aeronautics.airliners submissions to airliners@chicago.com From kls Tue Nov 24 16:00:40 1992 Newsgroups: sci.aeronautics.airliners Path: news From: rdd@cactus.org (Robert Dorsett) Subject: Seeking pointers on switch design. Message-ID: Approved: kls@ohare.Chicago.COM X-Original-Message-Id: <9211241115.AA16994@cactus.org> Sender: kls@ohare.Chicago.COM Date: Tue, 24 Nov 92 05:15:42 CST I'm looking for pointers to articles on the human-factors ramifications of switch design. I've noticed an interesting difference between Airbus and Boeing switch philosophy. Boeing seems to build the "on" state into the switch. It might be a white bar, indicating a closed circuit or open valve on a placarded systems a subdued "on" function description, with an "engage" bar, etc. But the philosophy seems to be: "default" state == off (dark indicator), pilot action to turn it on (white indicator), operational state = on (white indicator) until pilot turns it off again or an abnormal state occurs (colored indicator, annunciator). This doesn't violate the "dark cockpit" philosophy, since only one color (white) is used for selects, and abnormal states are clearly detectable. Airbus (in the A320, and presumably the A340 and A330), on the other hand, seems to use smart-logic to default to an "on" state which is completely dark. The switches, when pressed, then show an *abnormal* state, like turning a fuel pump off. Nearly all of the switches also have a "failure" state-flag, showing an amber or red fault message. There are also systems with "mixed" switch formats. For instance, since a fuel pump state is normally on, a switch, when pressed, turns it off and indicates an off state. But crossfeed valve switches, when pressed, show an "ON," followed by "OPEN," state, which seems more "positive." So the Airbus philosophy seems to be: initialize switch states at boot time (on, no indicator), pilot action to turn it off (illuminated, abnormal state), operational state = dark until pilot triggers a disconnect. Seems to me that Boeing's the correct approach: a thou-shalt-not, drilled into me at an early point, was never to use double-negatives to prompt user actions ("Do you not want to save the file? Y/N") . An action should ideally be expressed in *positive* terms. And the interface should be consistent across all systems and within systems. On the other hand, Airbus' design can be rationalized in that if the computers do *all* routine management, as they do, then bringing the pilots in the loop at initial start-up is an invitation for error: in this model, pilot involve- ment is an *abnormal* event, and signs of that involvement should be highlighted. This raises interesting implications of the pilots being out of the loop TOO long, perhaps never dealing with a system or mentally "reviewing" that system for several flights, as would be the case with more "hands-on" initialization and management. This could be the reason behind Airbus's pre-flight "walk-through," in which each switch illuminates in sequence, requiring the pilot to depress it to extinguish the light. Comments? References? --- Robert Dorsett rdd@cactus.org ...cs.utexas.edu!cactus.org!rdd From kls Tue Nov 24 16:00:41 1992 Newsgroups: sci.aeronautics.airliners Path: news From: rdd@cactus.org (Robert Dorsett) Subject: Re: Boeing 747-300 (PLEASE let this be the end of it! :-)) References: Message-ID: Approved: kls@ohare.Chicago.COM Organization: Capital Area Central Texas UNIX Society, Austin, Tx X-Original-Message-Id: <9211241245.AA19191@cactus.org> Sender: kls@ohare.Chicago.COM Date: Tue, 24 Nov 92 06:45:43 CST Michael Weiss wrote: >Airbus is truly working on the plane, and hopes to knock Boeing out of the >747 sales. Airbus is not working on the plane. It has no launch customers, and has not committed resources to it. It is in a very preliminary concept stage, which is being heavily hyped, as part of the marketing effort. It's a major step: the failure OR success of the airplane can easily damage the consortium, if they figure the market wrong. >However, Boeing has a history of being the best aircraft in the >world in terms of maintenance; Airbus apparently makes planes that are almost >as difficult to repair and inspect as McDonnell-Douglas. I've heard that Airbus maintenance is highly modular, highly automated, and very structured, with no more problems than what one might expect when client airlines switch from one vendor's accounting/maintenance practices to another. This takes time, it takes a lot of training, and it is a standard- ization nightmare: but there's little to suggest one manufacturer's program is better than the other's, on its own merits. >> McDonnell-Douglas MD-11 1993 > >Try 1992. The MD-11 was certified in October of 1991. By the end of 1991, MDC had made in excess of 30 deliveries of the MD-11. First deliveries tend to follow certification VERY quickly. >> Airbus AE-400 1994 > >Sounds about right. Well, they were talking about '93, but that likely means >1994. Supposedly, it will compete directly with the 747. The A340 should receive certification next month, with deliveries in January 1993. The A330 was rolled out last month, and should be certified by mid-1993. These are the only two new active programs Airbus is working on, apart from the A320 derivatives. >> Boeing 777 1995-6 First flight by June 1, 1994, deliveries in 1995, based on the July 1, 1992 Flight International. --- Robert Dorsett rdd@cactus.org ...cs.utexas.edu!cactus.org!rdd From kls Tue Nov 24 16:00:41 1992 Newsgroups: sci.aeronautics.airliners Path: news From: weiss@curtiss.SEAS.UCLA.EDU (Michael Weiss) Subject: Re: hydraulic problems with DC-10's?? References: Message-ID: Approved: kls@ohare.Chicago.COM Organization: SEASnet, University of California, Los Angeles X-Original-Message-Id: <8733@lee.SEAS.UCLA.EDU> Sender: kls@ohare.Chicago.COM Date: 24 Nov 92 17:40:41 GMT In article kls@ohare.Chicago.COM (Karl Swartz) writes: >In article sdl@linus.mitre.org writes: >>Some of my pilot friends have accused the DC-10 as having a >>particularly bad history of hydraulic problems (which have >>contributed to a few crashes). >Two, at least. AA 191 at Chicago/O'Hare on May 25, 1979, and UA 232 >at Sioux City, Iowa on July 19, 1989. I have a hard time believing that an intact hydraulic system would have prevented AA191 from crashing. Let's face it, a wing-mounted engine falling off produces such a rediculous unbalance that even full aileron wouldn't be able to counter it. -- \ | | | | | | | | | | | | | | | | | | | | | | | | | | | | | | | | | | | | | | / - Michael weiss@seas.ucla.edu | School of Engineering & Applied Science - - Weiss izzydp5@oac.ucla.edu | University of California, Los Angeles - / | | | | | | | | | | | | | | | | | | | | | | | | | | | | | | | | | | | | | | \ From kls Tue Nov 24 16:46:33 1992 Newsgroups: sci.aeronautics.airliners Path: news From: Stephen L Nicoud Subject: Boeing reduces production rate of 757 & 767 Message-ID: Approved: kls@ohare.Chicago.COM X-Original-Message-Id: <9211242332.AA01284@moclips.boeing.com> Sender: kls@ohare.Chicago.COM Date: Tue, 24 Nov 92 15:32:37 PST 757, 767 PRODUCTION RATE REDUCTIONS ANNOUNCED Boeing Commercial Airplane Group is announcing today reductions in 757 and 767 production rates in 1993. The monthly rate of 757s, which is currently at 8.5 airplanes per month, will go to seven in June 1993, and will be reduced further to five per month in November. The reduction to seven a month had originally been planned to take effect in September. Also in November, the 767 rate will be decreased from the current five per month to four. Early indications are that the 757 and 767 reductions could result in about 2,000 fewer jobs in the Puget Sound area and approximately 500 fewer jobs at the Commercial Airplane Group's Wichita Division. "Our customers' delivery needs are changing as a result of the difficult time many of them are currently having. We are adjusting our rates to meet their needs," said Dean Thornton, president of the Commercial Airplane Group. "We remain optimistic about the long-term. This decision takes into account the cumulative recent requests of a number of our customers for changes in their delivery streams," Thornton added. From kls Tue Nov 24 16:46:36 1992 Newsgroups: sci.aeronautics.airliners Path: news From: kls@ohare.Chicago.COM (Karl Swartz) Subject: Re: hydraulic problems with DC-10's?? References: Message-ID: Approved: kls@ohare.Chicago.COM Organization: Chicago Software Works X-Original-Message-ID: <1992Nov25.004006.28130@ohare.Chicago.COM> Sender: kls@ohare.Chicago.COM Date: Wed, 25 Nov 1992 00:40:06 GMT In article weiss@curtiss.SEAS.UCLA.EDU (Michael Weiss) writes: >I have a hard time believing that an intact hydraulic system would have >prevented AA191 from crashing. Let's face it, a wing-mounted engine falling >off produces such a rediculous unbalance that even full aileron wouldn't be >able to counter it. I don't see that ailerons have much to do with it -- the biggest effect would be a substantial yaw, which would require rudder input. In any case, start believing. A United 747 (N4713U, now N4724U) operating a HNL-SYD flight on Feb. 23, 1989 lost both engines on the right side due to debris ingestion after a cargo hatch failed. They dumped fuel and limped back to Honolulu, well over an hour's flying time. Having lots of altitude and airspeed to work with is certainly quite helpful, but isn't a requirement. A few years ago a Piedmont 737-200 lost #2 immediately after takeoff from O'Hare. The pilots promptly declared an emergency, turned around, and landed several minutes later on another runway. They didn't even realize that the engine had litterally fallen off until the got off he plane and looked. Getting back to AA 191, what really killed AA 191 was a stall of the left wing after the uncommanded retraction of the flaps on that side. Even this was recoverable had the pilots known that their stall speed was suddenly higher -- alas, McDonnell-Douglas didn't bother with any redundancy for the flap retraction warning and that happened to be powered by the engine that fell off. In simulator tests after the crash, every pilot crashed when confronted with the same scenario. When given this indicator, and thus some indication of what was going on, every pilot managed to maintain control of the aircraft. An engine separation, while certainly not a normal event, should not be a fatal event, and indeed the certification process requires some consideration of an engine separation. -- Karl Swartz |INet kls@ditka.chicago.com 1-415/854-3409 |UUCP uunet!decwrl!ditka!kls |Snail 2144 Sand Hill Rd., Menlo Park CA 94025, USA Send sci.aeronautics.airliners submissions to airliners@chicago.com From kls Wed Nov 25 03:16:35 1992 Newsgroups: sci.aeronautics.airliners Path: news From: bentson@CS.ColoState.EDU (Randolph Bentson) Subject: Re: hydraulic problems with DC-10's?? References: Message-ID: Approved: kls@ohare.Chicago.COM Organization: Colorado State University, Computer Science Department X-Original-Message-ID: Sender: kls@ohare.Chicago.COM Date: Wed, 25 Nov 1992 04:54:00 GMT In article weiss@curtiss.SEAS.UCLA.EDU (Michael Weiss) writes: > >I have a hard time believing that an intact hydraulic system would have >prevented AA191 from crashing. Let's face it, a wing-mounted engine falling >off produces such a rediculous unbalance that even full aileron wouldn't be >able to counter it. The problem was that when the hydraulics failed, leading edge slat retracted on one side and that wing stalled. No indication was given in the cockpit so the flight crew didn't use the appropriate recovery mechanism. Ref: "The DC-10 Case", John H. Feilder & Douglas Birsch, eds. -- Randolph Bentson Colorado State University bentson@CS.ColoState.Edu Computer Science Department 303/491-5792 Ft. Collins, CO 80523 From kls Wed Nov 25 03:16:39 1992 Newsgroups: sci.aeronautics.airliners Path: news From: kawai@Csli.Stanford.EDU (goh kawai - n6uok) Subject: Re: Emergency Oxygen Masks References: Message-ID: Approved: kls@ohare.Chicago.COM Organization: speech research program, sri international X-Original-Message-Id: <1992Nov25.004045.6594@Csli.Stanford.EDU> Sender: kls@ohare.Chicago.COM Date: Wed, 25 Nov 1992 00:40:45 GMT Gary Jacobs (gjacobs@qualcomm.com) comments: | I'm curious what the test criteria [of oxygen masks] are and how often | is it done? I'd also like to know what the failure rate is? I was on a flight the other day (B-747) when an oxygen mask dropped by accident in the business class cabin. I was horrified to see that the plastic tubing that connects the mask to the oxygen supply was brownish-colored due to age. I have an aquarium at home, and I know how long it takes for air-tubing to turn brown. While it is quite conceivable that they use a higher grade of tubing in aircraft, that is not awfully reassuring, because low-grade or high-grade, old tubing is old tubing. I shudder to think what condition the rest of the system is in. -goh- ----------------- Speech Research Program, SRI, Menlo Park, CA 94025-3493 USA --- Goh Kawai --- work:(415)859-2231 fax:(415)859-5984 home:(415)323-7214 ----------------- internet: kawai@speech.sri.com radio: n6uok and jk1zyp From kls Wed Nov 25 03:16:42 1992 Newsgroups: sci.aeronautics.airliners Path: news From: rdd@rascal.ics.utexas.edu (Robert Dorsett) Subject: NTSB DC-10 excerpts Message-ID: Approved: kls@ohare.Chicago.COM X-Original-Message-Id: <9211250826.AA14686@rascal.ics.utexas.edu> Sender: kls@ohare.Chicago.COM Date: Wed, 25 Nov 92 02:26:16 CST It's been about two years since I last posted this, so... -------------------- Excerpts from the NTSB accident report on the Chicago O'Hare crash: Synopsis: About 1504 CDT, May 25, 1979, American Airlines Flight 191, a McDonnell-Douglas DC-10-10 aircraft, crashed into an open field just short of a trailer park about 4600' northwest of the departure end of runway 32R at Chicago-O'Hare Internat- ional Airport, Illinois. Flight 191 was taking off from Runway 32R. The weather was clear and the vis- ibility was 15 miles. During the takeoff rotation, the left engine and pylon assembly and about 3 ft of the leading edge of the left wing separated from the aircraft and fell to the runway. Flight 191 continued to climb to about 325' above the ground and then began to roll to the left. The aircraft con- tinued to roll to the left until the wings were past the vertical position, and during the roll, the aircraft's nose pitched down below the horizon. Flight 191 crashed into the open field and the wreckage scattered into an adjacent trailer park. The aircraft was destroyed in the crash and subsequent fire. Two hundred and seventy-one persons on board Flight 191 were killed; two persons on the ground were killed, and two others were injured. An old aircraft hangar, several automobiles, and a mobile home were destroyed. The National Transportation Safety Board determines that the probable cause of this accident was the asymmetrical stall and the ensuing roll of the air- craft because of the uncommanded retraction of the left wing outboard leading edge slats and the loss of stall warning and slat disagreement indication sys- tems resulting from maintenance-induced damage leading to the separation of the No. 1 engine and pylon assembly at a critical point during takeoff. The sep- aration resulted from damage by improper maintenance procedures which led to failure of the pylon structure. Contributing to the cause of the accident were the vulnerability of the design of the pylon attach points to maintenance damage; the vulnerability of the design of the leading edge slat system to the damage which produced asymmetry; deficiencies in Federal Aviation Administration surveillance and reporting sys- tems which failed to detect and prevent the use of improper maintenance proced- ures; deficiencies in the practices and communications among the operators, the manufacturer, and the FAA which failed to determine and disseminate the particulars during previous maintenance damage incidents; and the intolerance of prescribed operational procedures to this unique emergency. Findings (p. 67) 1. The engine and pylon assembly separated either at or immediately after takeoff. The flightcrew was committed to continue the takeoff. 2. The aft end of the pylon assembly began to separate in the forward flange of the aft bulkhead. 3. The structural separation of the pylon was caused by a complete failure of the forward flange of the aft bulkhead after its residual strength had been critically reduced by the fracture and subsequent service life. 4. The overload fracture and fatigue cracking on the pylon aft bulkhead's upper flange were the only preexisting damage on the bulkhead. The length of the overload fracture and fatigue cracking was about 13 inches. The fracture was caused by an upward movement of the aft end of the pylon which brought the upper flange and its fasteners into contact with the wing clevis. 5. The pylon to wing attach hardware was properly installed at all attachment points. 6. All electrical power to the No. 1 AC generator bus and No. 1 DC bus was lost after the pylon separated. The captain's flight director instrument, the stall warning system, and the slat disagreement warning light systems were rendered inoperative. Power to these buses was never restored. 7. The No. 1 hydraulic system was lost when the pylon separated. Hydraulic systems No. 2 and No. 3 operated at their full capability throughout the flight. Except for spoiler panels No. 2 and No. 4 on each wing, all flight controls were operating. 8. The hydraulic lines and followup cables of the drive actuator for the left wing's outboard leading edge slat were severed by the separation of the pylon and the left wing's outboard slats retracted during climbout. The retraction of the slats caused an asymmetric stall and subsequent loss of control of the aircraft. 9. The flightcrew could not see the wings and engines from the cockpit. Because of the loss of the slat disagreement light and the stall warning system, the flightcrew would not have received an electronic warning of either the slat asymmetry or the stall. The loss of the warning systems created a situation which afforded the flightcrew an inadequate opportunity to recognize and prevent the ensuing stall of the aircraft. 10. The flightcrew flew the aircraft in accordance with the prescribed emer- gency procedure, which called for the climbout to be flown at V2 speed. V2 was 6 KIAS below the stall speed for the left wing. The deceleration to V2 speed caused the aircraft to stall. The start of the left roll was the only warning the pilot had of the onset of the stall. 11. The pylon was damaged during maintenance performed on the accident aircraft at American Airline's Maintenance Facility at Tulsa, Oklahoma, on March 29 and 30, 1979. 12. The design of the aft bulkhead made the flange vulnerable to damage when the pylon was being separated or attached. 13. American Airlines engineering personnel developed an ECO to remove and reinstall the pylon and engine as a single unit. The ECO directed that the combined engine and pylon assembly be supported, lowered, and raised by a forklift. American Airlines engineering personnel did not perform an adequate evaluation of either the capability of the forklift to provide the required precision for the task, or the degree of difficulty involved in placing the lift properly, or the consequences of placing the lift improperly. The CO did not emphasize the precision required to place the forklift properly. 14. The FAA does not approve the carriers' maintenance procedures, and a carrier has the right to change its maintenance procedures without FAA approval. 15. American Airlines personnel removed the aft bulkhead's bolt and bushing before removing the forward bulkhead attach fittings. This permitted the forward bulkhead to act as a pivot. Any advertent or inadvertent loss of forklift support to the engine and pylon assembly would produce an upward movement at the aft bulkhead's upper flange and bring it into contact with the wing clevis. 16. American Airlines maintenance personnel did not report formally to their maintenance engineering staff either their deviation from the removal sequence contained in the ECO or the difficulties they had encountered in accomplishing the ECO's procedures. 17. American Airline's engineering personnel did not perform a thorough evaluation of all aspects of the maintenance procedures before they formulated the ECO. The engineering and supervisory personnel did not monitor the performance of the ECO to ensure either that it was being accomplished properly or if their maintenance personnel were encountering unforeseen difficulties in performing the assigned tasks. 18. The nine situations in which damage was sustained and cracks were found on the upper flange were limited to those operations wherein the engine and pylon assembly was supported by a forklift. 19. On December 19, 1978, and Feb. 22, 1979, Continental Airlines maintenance personnel damaged aft bulkhead upper flanges in a manner similar to the damage noted on the accident aircraft. The carrier classified the cause of the damage as maintenance error. Neither the air carrier nor the manufacturer interpreted the regulation to require that it further investigate or reprot the damages to the FAA. 20. The original certification's fatigue-damage assessment was in conformance with the existing requirements. 21. The design of the stall warning system lacked sufficient redundancy; there was only one stickshaker motor; and further, the design of the system did not provide for crossover information to the left and right stall warning computers from the applicable leading edge slat sensors on the opposite side of the aircraft. 22. The design of the leading edge slat system did not include positive mechanical locking devices to prevent movement of the slats by external loads following a failure of the primary controls. Certification was based upon acceptable flight characteristics with an asymmetrical leading edge slat condition. 23. At the time of DC-10 certification, the structural separation of an engine pylon was not considered. Thus, multiple failures of other systems resulting from this single event was not considered. Additional excerpts: [design requirements for slats] "The motion on the flaps on opposite sides of the plane of symmetry must be synchronized unless the aircraft has safe characteristics with the flaps retracted on one side and extended on the other." Since the left and right inboard slats are controlled by a single valve and actuated by a common drum and the left and right outboard slats receive their command from mechanically linked control valves which are "slaved" to the inboard slats by the followup cable, the synchronization requirement was satisfied. However, since the cable drum actuating mechanisms of the left and right outboard slats were independent of each other, the possibility existed that one outboard slat might fail to respond to a commanded movement. Therefore, the safe flight characteristics of the aircraft with asymmetrical outboard slats were demonstrated by test flight. These flight characteristics were investigated within an airspeed range bounded by the limiting airspeed for the takeoff slat positions--260 kts--and the stall warning speed; the flight test did not investigate these characteristics under takeoff conditions. In addition, a slat disagree warning light system was installed which, when illuminated, indicated that the slat handle and slat position disagree, or the slats are in transit, or the slats have been extended automatically. The program engineer stated that the commanded slat position is held by trapped fluid in the actuating cylinder, and that no consideration was given to an alternate locking mechanism. The slats' hydraulic lines and followup cables were routed as close as possible to primary structure for protection; however, routing them behind the wing's front spar was not considered because of interference with other systems. "The branch chief of the Reliability and Safety Engineering Organization of the Douglas Aircraft Company described the failure mode and effects analysis (FMEA) and fault analysis. The witness indicated that the FMEA was a basic working document in which rational failure modes were postulated and analyzed; vendors and subcontractors were requested to perform similar analyses on equipment they supplied to McDonnell-Douglas. Previous design and service experience was incorporated in the initial DC-10-10's FMEA's, and analyses were modified as the design progressed. The FMEA's were synthesized to make fault analyses, which were system-oriented summary documents submitted to the FAA to satisfy 14 CFR 25.1309. The FAA could have requested and could have reviewed the FMEA's. The basic regulations under which the slats were certified did not require accountability for multiple failures. The slat fault analysis submitted to the FAA listed 11 faults or failures, all of which were correctable by the flightcrew. However, one multiple failure--erroneous motion transmitted to the right-hand outboard slats and an engine failure on the appropriate side-- was considered by McDonnell-Douglas in its FMEA. The FMEA noted that the "failure increases the amount of yaw but would be critical only under the most adverse flight or takeoff conditions. The probability of both failures occurring is less than 1 x 10e-10 [a popular number with airframe manufacturers!]." [...] "The December 1, 1978 revision of 14 CFR 25.571 retitled the regulation "Damage-Tolerance and Fatigue Evaluation of Structure." The fail-safe evaluation must now include damage modes due to fatigue, corrosion, and accidental damage. According to the manufacturer, the consideration for accidental damage was limited to damage which can be inflicted during routine maintenance and aircraft servicing." [...] "Because of the designed redundancy in the aircraft's hydraulic and electrical systems, the losses of those systems powered by the No. 1 engine should not have affected the crew's ability to control the aircraft. However, as the pylon separated from the aircraft, the forward bulkhead contacted and severed four other hydraulic lines and two cables which were routed through the wing leading edge forward of the bulkhead. These hydraulic lines were the operating lines from the leading edge slat control valve, which was located inboard of the pylon, and the actuating cylinders, which extend and retract the outboard leading edge slats. Two of the lines were connected to the No. 1 hydraulic system and two were connected to the No. 3 system, thus providing the redundancy to cope with a single hydraulic system failure. The cables which were severed provided feedback of the leading edge slat position so that the control valve would be nulled when slat position agreed with position commanded by the cockpit control. The severing of the hydraulic lines in the leading edge of the left wing could have resulted in the eventual loss of No. 3 hydraulic system because of fluid depletion. However, even at the most rapid rate of leakage possible, the system would have operated throughout the flight. The extended No. 3 spoiler panel on the right wing, which was operated by the No. 3 hydraulic system, confirmed that this hydraulic system was operating. Since two of the three hydraulic systems were operative, the Safety Board concludes that, except for the No. 2 and No. 4 spoiler panels on both wings which were powered by the No. 1 hydraulic systems, all flight controls were operating. Therefore, except for the significant effect that the severing of the No. 3 hydraulic system's lines had on the left leading edge slat system, the fluid leak did not play a role in the accident. During takeoff, as with any normal takeoff, the leading edge slats were extended to provide increased aerodynamic lift on the wings . When the slats are extended and the control valve is nulled, hydraulic fluid is trapped in the actuating cylinder and operating lines. The incompressiblity of this fluid reacts against any external air loads and holds the slats extended. This is the only lock provided by the design. Thus, when the lines were severed and the trapped hydraulic fluid was lost, air loads forced the left outboard slats to retract. While other failures were not critical, the uncommanded movement of these leading edge slats had a profound effect on the aerodynamic performance and controllability of the aircraft. With the left outboard slats retracted and all others extended, the lift of the left wing was reduced and the airspeed at which that wing would stall was increased. The simulator tests showed that even with the loss of the No. 2 and No. 4 spoilers, sufficient lateral control was available from the ailerons and other spoilers to offset the asymmetric lift caused by left slat retraction at airspeeds above that at which the wing would stall. However, the stall speed for the left wing increased to 159 KIAS. [...] The Safety Board is also concerned that the designs of the flight control, hydraulic, and electrical systems in the DC-10 aircraft were such that all were affected by the pylon separation to the extent that the crew was unable to ascertain the measures needed to maintain control of the aircraft. The airworthiness regulations in effect when the DC-10 was certificated were augmented by a Special Condition, the provisions of which had to be met before the aircraft's fully powered control system would be certificated. The Special Condition required that the aircraft be capable of continued flight and of being landed safely after failure of the flight control system, including lift devices. These capabilities must be demonstrated by analysis or test, or both. However, the Special Condition, as it applied to the slat control system, was consistent with the basic airworthiness regulations in effect at the time. The basic airworthiness regulations specified requirements for wing flap asymmetry only and did not include specific consideration of other lift devices. Because the leading edge slat design did not contain any novel or unusual features, it was certificated under the basic regulation. The flap control requirements for symmetry and synchronization were applied to and satisfied by the slat system design. Since a malfunction of the slat actuating system could disrupt the operation of an outboard slat segment, a fault analysis was conducted to explore the probability and effects of both an uncommanded movement of the outboard slats and the failure of the outboard slats to respond to a commanded movement. The fault analysis concluded that the aircraft could be flown safely with this asymmetry. Other aircraft designs include positive mechanical locking devices to prevent movement of slats by external loads following a primary failure. The DC-10 design did not include such a feature nor was it deemed necessary, since compliance with the regulations was based upon analysis of those failure modes which could result in asymmetrical positioning of the leading edge devices and a demonstration that sufficient lateral control was available to compensate for the asymmetrical conditions throughout the aircraft's flight envelope. The flight tests conducted to evaluate the controllability of the aircraft were limited to a minimum airspeed compatible with stall-warning activation predicated upon the slat-retracted configuration. From kls Wed Nov 25 03:16:43 1992 Newsgroups: sci.aeronautics.airliners Path: news From: kls@ohare.Chicago.COM (Karl Swartz) Subject: MD-11 milestone dates References: Message-ID: Approved: kls@ohare.Chicago.COM Organization: Chicago Software Works X-Original-Message-ID: <1992Nov25.102758.29314@ohare.Chicago.COM> Sender: kls@ohare.Chicago.COM Date: Wed, 25 Nov 1992 10:27:58 GMT In article rdd@cactus.org (Robert Dorsett) writes: >Michael Weiss wrote: >>> McDonnell-Douglas MD-11 1993 >>Try 1992. The MD-11 was certified in October of 1991. >By the end of 1991, MDC had made in excess of 30 deliveries of the MD-11. >First deliveries tend to follow certification VERY quickly. Having received several other submissions mentioning earlier service dates I decided to look up the real dates for the MD-11 and hopefully nail the lid on the debate once and for all. date LN reg'n note ---- -- ----- ---- Jan 10 1990 447 N11MD first flight Nov 9 1990 - - type certificate issued (GE CF6-80C2 engines) Nov 29 1990 455 OH-LGA first delivery (PR event, to Finnair) Dec 7 1990 455 OH-LGA actual title transfer (at Las Vegas) Dec 18 1990 - - type certificate issued (P&W PW4460 engines) Dec 20 1990 455 OH-LGA first revenue service (Finnair, Helsinki- Tenerife charter) Dec 21 1990 453 N891DL delivery of Delta's first two MD-11s, leased 454 N892DL from Mitsui (GE engines; Delta's own are PW) Jan 24 1991 456 HL7371 first delivery of PW4460 version (to Korean) Feb 5 1991 454 N892DL first scheduled service (DL 4049 ATL-DFW-MCO) Feb 6 1991 453 N891DL first scheduled int'l (LAX-NRT; arrived LAX previous day operating DL 4039 ATL-DFW-LAX) Jun 27 1991 447 N601FE delivery of first MD-11 (to Federal Express, after refitting by Aerotest in Mojave, CA) Roughly 35 MD-11s had been delivered by the end of 1991; McDonnell- Douglas got off to a rather slow start because nearly every one of the early aircraft were different, including passenger, freight, and combi versions, some with GE engines and others with Pratt and Whitney. The Rolls-Royce version was cancelled after the demise of Air Europe. BTW, note that line number 447 is the first MD-11. 446 was the last of the last of the DC-10 line, a KC-10 if I'm not mistaken. -- Karl Swartz |INet kls@ditka.chicago.com 1-415/854-3409 |UUCP uunet!decwrl!ditka!kls |Snail 2144 Sand Hill Rd., Menlo Park CA 94025, USA Send sci.aeronautics.airliners submissions to airliners@chicago.com From kls Wed Nov 25 03:16:44 1992 Newsgroups: sci.aeronautics.airliners Path: news From: Karl Swartz Subject: Thanksgiving Message-ID: Approved: kls@ohare.Chicago.COM X-Original-Message-Id: Sender: kls@ohare.Chicago.COM Date: Wed, 25 Nov 92 3:14:35 PST Tomorrow (er, it's late, better make that today) is the day before Thanksgiving in the United States, traditionally the busiest air travel day of the year. My home-town airport (perhaps you've heard of it -- Chicago's O'Hare International, namesake of the machine I'm posting this on) usually sets a new passenger volume record every year on this day, though this year may be an exception as the half-price fare war led to several record-shattering days this past summer. (For those of you who may be unfamiliar with this custom, it entails family and friends gathering together on the Thursday before the last Friday in November to consume a turkey and stuffing, yams, cranberry sauce, pumpkin pie, and all sorts of other goodies, followed by three days to sit back, watch football, and digest all that food!) In any case, I'll be doing my part to support the travel industry this holiday, though not at O'Hare and not by air. This means I'll be away from the keyboard until Monday, and airliners posts likely won't go out until then. (I may check in once or twice before then but no promises.) To those of you in the US or celebrating this US holiday, may this be a happy and *safe* holiday for you. To the rest, well, you'll be saved the inevitable weeks of leftover turkey sandwiches. ;-) -- Karl Swartz |INet kls@ditka.chicago.com 1-415/854-3409 |UUCP uunet!decwrl!ditka!kls |Snail 2144 Sand Hill Rd., Menlo Park CA 94025, USA Send sci.aeronautics.airliners submissions to airliners@chicago.com From kls Wed Nov 25 11:07:26 1992 Newsgroups: sci.aeronautics.airliners Path: news From: rdd@cactus.org (Robert Dorsett) Subject: Re: hydraulic problems with DC-10's?? References: Message-ID: Approved: kls@ohare.Chicago.COM Organization: Capital Area Central Texas UNIX Society, Austin, Tx X-Original-Message-Id: <9211251258.AA19229@cactus.org> Sender: kls@ohare.Chicago.COM Date: Wed, 25 Nov 92 06:58:52 CST In article kls@ohare.Chicago.COM (Karl Swartz) writes: >In article weiss@curtiss.SEAS.UCLA.EDU (Michael Weiss) writes: >>I have a hard time believing that an intact hydraulic system would have >>prevented AA191 from crashing. Let's face it, a wing-mounted engine falling >>off produces such a rediculous unbalance that even full aileron wouldn't be >>able to counter it. > >I don't see that ailerons have much to do with it -- the biggest >effect would be a substantial yaw, which would require rudder input. In the NTSB report on the DC-10 crash, a considerable amount of both yaw and rudder were necessary to regain level flight, in the simulator tests-- 80% right rudder and 70% right-wing-down aileron; roll angles didn't exceed 30 degrees before recovery. Normally, given asymmetric thrust, you bank into the good engine(s): rudder's normally used to augment the ailerons as necessary to control sideslip. >Having lots of altitude and airspeed to work with is certainly quite >helpful, but isn't a requirement. A few years ago a Piedmont 737-200 >lost #2 immediately after takeoff from O'Hare. The pilots promptly >declared an emergency, turned around, and landed several minutes later >on another runway. They didn't even realize that the engine had >litterally fallen off until the got off he plane and looked. There are actually two issues at work, here: one is the *power* lost by the engine. To maintain level flight, the power required for flight must equal the power available. If the power available is less, one will start to descend; if it's a lot less, one will descend faster. The real issue is just power: it has little to do with where the failure was: losing two of three engines on a 727 at MTOW means you'll go down, too. The second issue is the moment produced by the combination of the "dead" engine (with its drag) and the "good" engines. This is generally a minimal issue, assuming the airspeed is there, and the pilot applies correct technique. Most transport aircraft can fly with all engines out on one side, although I do not know if this is an explicit regulatory requirement. As long as the inherent longitudinal stability of the airplane (contributed by the vertical stabilizer, rudder, wings, and fuselage) is sufficient to overcome the yawing moment, the airplane can be controlled. So *correcting* for a lost engine is a near-instantaneous correction, applied by the pilot, needing no altitude reserve. During the El Al discussion on sci.aero, rec.av, and rec.travel.air, there seemed to be considerable confusion between the role each factor took. --- Robert Dorsett rdd@cactus.org ...cs.utexas.edu!cactus.org!rdd From kls Sun Nov 29 14:35:26 1992 Newsgroups: sci.aeronautics.airliners Path: news From: Bob Coggeshall Subject: 757 highest thrust to weight ratio ? Message-ID: Approved: kls@ohare.Chicago.COM X-Original-Message-Id: <199211281348.AA07134@drewll.cogwheel.com> Sender: kls@ohare.Chicago.COM Date: Sat, 28 Nov 1992 21:48:00 +0800 I've heard that the 757 has a very high thrust to weight ratio. Just how high is it ? Is it the highest of any commercial jetliner ? I've also heard that there is a boeing-internal video of it during tests doing an [almost?] straight vertical climb. What are the facts here ? Thanks. //////////////////////////////////////////////////////////////// Bob Coggeshall, President Cogwheel Incorporated - Producers of Low-cost dial-up IP Routers coggs@hongkong.Cogwheel.COM //////////////////////////////////////////////////////////////// From kls Sun Nov 29 14:35:27 1992 Newsgroups: sci.aeronautics.airliners Path: news From: weiss@wright.SEAS.UCLA.EDU (Michael Weiss) Subject: Re: pair (sorry, couldn't resist the pun) References: Message-ID: Approved: kls@ohare.Chicago.COM Organization: SEASnet, University of California, Los Angeles X-Original-Message-Id: <8761@lee.SEAS.UCLA.EDU> Sender: kls@ohare.Chicago.COM Date: 29 Nov 92 08:04:20 GMT In article kls@ohare.Chicago.COM (Karl Swartz) writes: >In article weiss@mott.SEAS.UCLA.EDU (Michael Weiss) writes: >>However, Boeing has a history of being the best aircraft in the >>world in terms of maintenance; Airbus apparently makes planes that are almost >>as difficult to repair and inspect as McDonnell-Douglas. >I have not previously encountered negative comments regarding >McDonnell-Douglas products in this context, however, and in fact have >heard that the DC-10 is rather well-liked because it's somewhat like >a big Chevy V-8 -- solid, and easy to fix when it breaks. (Problems >with the design of the hydraulics notwithstanding.) All of my repair information comes from my girlfriend's brother, who works as a maintenance guy for SkyWest (a commuter airline that operates also as the Delta Connection in Los Angeles, Palm Springs, Phoenix, and Las Vegas primarily). He said that DC-10s are notorious for repairs being all-day operations, whereas Boeing's 737-300 and -400, 747-300 and -400, 757, and 767 have self-diagnostic systems that go so far as to direct the location of the repair instructions down to the page, turning the repairs into a half-day operation instead. Note that I cannot verify this information, but I see no reason to dispute it. >Seems to me that Lockheed, the L-1011 in particular but perhaps the >Electra in its time as well, tended toward somewhat more finicky >products that compensated by giving better performance. Now there's one I should ask my cousin. He was a test pilot for the L-1011 when he was first hired by Lockheed. >With regard to the MD-12, MacDAC seems to remain in the race nearly as >much as Boeing and Airbus, though their ability to carry through with >an actual aircraft is certainly less certain given their finances. In >any case all three are paper planes until the airlines get themselves >into better financial shape. My understanding is that the capital exists within Boeing, and can certainly be "created" within Airbus, but MacDAC has been losing faith quickly from its investors. At least, that's what the LA Times seems to indicate. Rumor had it that if the MD-11 did not get cert back in October '91, MacDAC was going to have to file for bankruptcy. Again, this was LA Times info. -- \ | | | | | | | | | | | | | | | | | | | | | | | | | | | | | | | | | | | | | | / - Michael weiss@seas.ucla.edu | School of Engineering & Applied Science - - Weiss izzydp5@oac.ucla.edu | University of California, Los Angeles - / | | | | | | | | | | | | | | | | | | | | | | | | | | | | | | | | | | | | | | \ From kls Sun Nov 29 14:35:28 1992 Newsgroups: sci.aeronautics.airliners Path: news From: michael@is3000.bmr.gov.au (System Support) Subject: Boeing Book Message-ID: Approved: kls@ohare.Chicago.COM Organization: Australian Geological Survey Organization X-Original-Message-Id: <9211291359.AA08442@is3000.UUCP> Sender: kls@ohare.Chicago.COM Date: Sun, 29 Nov 92 13:59:40 AUS Maybe someone can help me. I'm trying to get hold of a new book about Boeing entitled something like "The Boeing Story". Does anyone have any ideas about such a book, author? Any help would be much appreciated. Does Boeing (Seattle) have an email address? Thanking You in appreciation, Michael michael@is3000.bmr.gov.au -- From kls Sun Nov 29 14:35:28 1992 Newsgroups: sci.aeronautics.airliners Path: news From: fxm4993@hertz.njit.edu (Farhan Muhammad) Subject: Request for the informations on Concorde. Message-ID: Approved: kls@ohare.Chicago.COM Organization: New Jersey Institute of Technology, Newark, N.J. X-Original-Message-ID: <1992Nov29.192511.29500@njitgw.njit.edu> Sender: kls@ohare.Chicago.COM Date: Sun, 29 Nov 1992 19:25:11 GMT Hello Everybody. Recently, i have come to know from an article that the Concorde doesn't have the flaps. Can anyone brief me on how the plane flies, specially land without the help of flaps. P.S. can anyone inform me on the take-of and landing procedures of Concorde. My thanks are offered. Fahan Muhammad Embry-Riddle Aeronautical Univ. muhammaf@erau.db.erau.edu From kls Tue Dec 1 00:13:23 1992 Newsgroups: sci.aeronautics.airliners Path: news From: gregory@bcstec.ca.boeing.com (Greg Wright) Subject: Re: The Sporty Game -- Boeing 757 X-Submission-Date: Wed, 25 Nov 92 13:25:45 PST References: Message-ID: Approved: kls@ohare.Chicago.COM Organization: Boeing Sender: kls@ohare.Chicago.COM X-Submission-Message-Id: <9211252125.AA23119@bcstec.ca.boeing.com> Date: 01 Dec 92 00:13:23 PST In article kls@ohare.Chicago.COM (Karl Swartz) writes: > >I'm *still* surprised that Boeing hasn't made much noise (maybe none) >about plugging this obvious hole by offering a 757-100 or whatever -- >a shortened 757 like the original proposal and a real replacement for >the 727-200. Even with United it never seemed to come up, instead all >the discussion focussing on a massively stretched and pulled and re- >designed 737-600. True, a 757 is more expensive (~ $45 million versus >$30 - 35 million) but the changes embodied in the 737-600 would surely >have added tremendously to the price. > >-- >Karl Swartz |INet kls@ditka.chicago.com >1-415/854-3409 |UUCP uunet!decwrl!ditka!kls > |Snail 2144 Sand Hill Rd., Menlo Park CA 94025, USA > Send sci.aeronautics.airliners submissions to airliners@chicago.com I think that you will find that every attempt at a shortened version of one of our planes has had limited success. Take the 747SP for example. Airlines tend not to like the sorted versions very much. In $/seat or $/mile these versions are too expensive to run. There is a real problem having too much engine or wing with them. We find it is better to stretch if anything.... Greg -- ________Greg Wright____________ "I struggle to be brief | gregory@bcstec.ca.boeing.com | and become obscure." | gregory@halcyon.com | |____uunet!bcstec!gregory_______| NOT A BOEING SPOKESPERSON. From kls Tue Dec 1 00:13:24 1992 Newsgroups: sci.aeronautics.airliners,rec.travel.air Path: news From: cid@athena.mit.edu (Derek H Cedillo) Subject: A320 loses wheels and skids 200 feet X-Submission-Date: Mon, 30 Nov 1992 03:58:24 GMT References: Message-ID: Approved: kls@ohare.Chicago.COM Organization: Massachusetts Institute of Technology Sender: kls@ohare.Chicago.COM X-Submission-Message-Id: <1992Nov30.035824.22954@athena.mit.edu> Date: 01 Dec 92 00:13:24 PST A Mexicana A320 Lost its nose gear on takeoff at LAX (flt #901 @3:25PM) The aircraft was fairly new, what I was wondering, was what it takes exactly to lose a gear, especially on take off. Landing, you have a lot more stress and what not. My main concern is that with it being a relatively new aircraft, long term stress and maintainence problems shouldnt be much of a factor, thus implying either a tragic design flaw or machining flaw/one time error. I realize that ther rest of the A320's problems dont stem from things like this but mainly from control problems. Does anyone have specific info on what it would/did take to have this gear fail? Thanks Derek ------------------------------------------------------------------- "He lived a life of going-to-do, and died with nothing done" -J. Albery In other words---JUST DO IT! From kls Tue Dec 1 00:13:24 1992 Newsgroups: sci.aeronautics.airliners Path: news From: slenk@hal.EMBA.UVM.EDU (Carl A Slenk) Subject: Boeing Book X-Submission-Date: Mon, 30 Nov 1992 19:25:43 GMT References: Message-ID: Approved: kls@ohare.Chicago.COM Organization: University of Vermont, EMBA Computer Facility Sender: kls@ohare.Chicago.COM X-Submission-Message-Id: <1992Nov30.192543.16611@uvm.edu> Date: 01 Dec 92 00:13:24 PST A new book out in January of 93 looks interesting: Wide-Body: the Triumph of the 747 by Clive Irving Pub: William Morrow &co -- Carl A. Slenk | "A computer lets you make more mistakes faster slenk@hal.emba.uvm.ed | then any other invention with the possible University of Vermont | exceptions of handguns and Tequilla" - My opinions;get your own | Mitch Ratcliffe From kls Tue Dec 1 00:13:25 1992 Newsgroups: sci.aeronautics.airliners Path: news From: raveling@Unify.com (Paul Raveling) Subject: Re: pair (sorry, couldn't resist the pun) X-Submission-Date: Mon, 30 Nov 92 20:21:32 GMT References: Message-ID: Approved: kls@ohare.Chicago.COM Organization: Unify Corporation (Sacramento) Sender: kls@ohare.Chicago.COM X-Submission-Message-Id: Date: 01 Dec 92 00:13:25 PST In article , weiss@wright.SEAS.UCLA.EDU (Michael Weiss) writes: > > In article kls@ohare.Chicago.COM (Karl Swartz) writes: > >In article weiss@mott.SEAS.UCLA.EDU (Michael Weiss) writes: > >>However, Boeing has a history of being the best aircraft in the > >>world in terms of maintenance; Airbus apparently makes planes that are almost > >>as difficult to repair and inspect as McDonnell-Douglas. > >I have not previously encountered negative comments regarding > >McDonnell-Douglas products in this context, however, and in fact have > >heard that the DC-10 is rather well-liked because it's somewhat like > >a big Chevy V-8 -- solid, and easy to fix when it breaks. (Problems > >with the design of the hydraulics notwithstanding.) My dad always said that DC-10's were much easier to maintain than the Boeings that he also worked on at Western Airlines. My impression from him was that the DC-10's greatest advantage for maintenance was better access to components or areas needing maintenance. To put it another way, the hell-hole count was much lower on the DC-10. He also cited a lot of things that were simply more durable on the DC-10. This is basically an echo of your report about being solid and easy to fix. Background: My dad worked in Western's LAX shop until he retired about the time Western merged into Delta. The Boeings in question were 707's, 720's, 727's, and 737's. Farther back (1950's) he also worked for Boeing's Renton plant when they were first manufacturing 707's and KC-135's. He still did occasional minor consulting jobs for Boeing until retirement, typically to update quick-reference maintenance manuals. He also had contacts at United's LAX shop who passed on info about 747's & 767's. ------------------ Paul Raveling Raveling@Unify.com From kls Tue Dec 1 00:13:26 1992 Newsgroups: sci.aeronautics.airliners Path: news From: Christopher Davis Subject: MD-11 milestone dates X-Submission-Date: Mon, 30 Nov 1992 15:34:58 -0500 References: Message-ID: Approved: kls@ohare.Chicago.COM Sender: kls@ohare.Chicago.COM X-Submission-Message-Id: <199211302034.AA07008@loiosh.eff.org> Date: 01 Dec 92 00:13:26 PST KS> == Karl Swartz KS> Having received several other submissions mentioning earlier service KS> dates I decided to look up the real dates for the MD-11 and hopefully KS> nail the lid on the debate once and for all. KS> date LN reg'n note KS> ---- -- ----- ---- KS> Dec 21 1990 453 N891DL delivery of Delta's first two MD-11s, leased KS> 454 N892DL from Mitsui (GE engines; Delta's own are PW) Interesting note on these two; I was on a DL flight from CVG-BOS last night (Thanksgiving travel :) and was looking in the seat pocket (trying to find a DL postcard I didn't already have, actually) and noted their "supplemental" emergency information card, which covers the aisle lighting (red lights at exits, white elsewhere) now standard. The card listed all of DL's jet types, with an interesting note next to MD-11: "(except ships 891 and 892)". After looking in JP, we realized that these were the "odd couple" and that explained it. (Unrelated notes: 757 BOS-CVG, 727 CVG-CLE, 727 CLE-CVG, 757 CVG-BOS, and the outdoor viewing deck at CLE was closed (sigh) but at least the glassed-in area was open...) From kls Tue Dec 1 00:13:26 1992 Newsgroups: sci.aeronautics.airliners Path: news From: Christopher Davis Subject: Re: Boeing Book X-Submission-Date: Mon, 30 Nov 1992 15:41:12 -0500 References: Message-ID: Approved: kls@ohare.Chicago.COM Sender: kls@ohare.Chicago.COM X-Submission-Message-Id: <199211302041.AA07012@loiosh.eff.org> Date: 01 Dec 92 00:13:26 PST Michael> == System Support Michael> Maybe someone can help me. I'm trying to get hold of a new Michael> book about Boeing entitled something like "The Boeing Story". Michael> Does anyone have any ideas about such a book, author? Any help Michael> would be much appreciated. This might be the Robert Serling _Legend & Legacy_, which I hope to find the time to do a book review on, eventually. *Very* good book. (I don't have an ISBN handy, the book's at home.) Michael> Does Boeing (Seattle) have an email address? Probably not an official one, if that's what you're asking. There are a number of divisions of Boeing, and they're all under the boeing.com domain. From kls Tue Dec 1 02:10:47 1992 Newsgroups: sci.aeronautics.airliners Path: news From: drinkard@bcstec.ca.boeing.com (Terrell D. Drinkard) Subject: Re: Seeking pointers on switch design. X-Submission-Date: Tue, 1 Dec 1992 02:17:17 GMT References: Message-ID: Approved: kls@ohare.Chicago.COM Organization: Boeing Sender: kls@ohare.Chicago.COM X-Submission-Message-Id: Date: 01 Dec 92 02:10:47 PST In article rdd@cactus.org (Robert Dorsett) writes: >I'm looking for pointers to articles on the human-factors ramifications of >switch design. I've noticed an interesting difference between Airbus and >Boeing switch philosophy. > [Much interesting material deleted] > >On the other hand, Airbus' design can be rationalized in that if the computers >do *all* routine management, as they do, then bringing the pilots in the loop >at initial start-up is an invitation for error: in this model, pilot involve- >ment is an *abnormal* event, and signs of that involvement should be >highlighted. This raises interesting implications of the pilots being out of >the loop TOO long, perhaps never dealing with a system or mentally "reviewing" >that system for several flights, as would be the case with more "hands-on" >initialization and management. This could be the reason behind Airbus's >pre-flight "walk-through," in which each switch illuminates in sequence, >requiring the pilot to depress it to extinguish the light. > >Comments? References? I'd just like to address one small part of your message, the part dealing with pilots being out of the loop too long. I've read not to long ago that there is research being performed on a tweak of the flight management systems paradigm. Instead of the FMCS just flying the programmed course, the new thought is to have it tell the pilot the next step and have the pilot initiate the maneuver. This keeps the flight crew mentally engaged, one hopes. Apparently there is a history of incidents where the pilot was too far behind the airplane. I just love this industry! :-) -- Terry drinkard@bcstec.boeing.com "Anyone who thinks they can hold the company responsible for what I say has more lawyers than sense." From kls Tue Dec 1 02:10:49 1992 Newsgroups: sci.aeronautics.airliners Path: news From: drinkard@bcstec.ca.boeing.com (Terrell D. Drinkard) Subject: Re: hydraulic problems with DC-10's?? X-Submission-Date: Tue, 1 Dec 1992 02:25:46 GMT References: Message-ID: Approved: kls@ohare.Chicago.COM Organization: Boeing Sender: kls@ohare.Chicago.COM X-Submission-Message-Id: Date: 01 Dec 92 02:10:49 PST In article weiss@curtiss.SEAS.UCLA.EDU (Michael Weiss) writes: > >In article kls@ohare.Chicago.COM (Karl Swartz) writes: >>In article sdl@linus.mitre.org writes: >>>Some of my pilot friends have accused the DC-10 as having a >>>particularly bad history of hydraulic problems (which have >>>contributed to a few crashes). >>Two, at least. AA 191 at Chicago/O'Hare on May 25, 1979, and UA 232 >>at Sioux City, Iowa on July 19, 1989. > >I have a hard time believing that an intact hydraulic system would have >prevented AA191 from crashing. Let's face it, a wing-mounted engine falling >off produces such a rediculous unbalance that even full aileron wouldn't be >able to counter it. Not true. An engine departing the airplane is a planned for event, in terms of stability and control. An aileron would have no problem countering just the imbalance of thrust (and it would actually be mostly rudder), in fact, without the added drag of a windmilling engine, the problem is a bit simplified. Flight AA 191 lost the slats on the left hand wing (if memory serves) because of Douglas' failure to include mechanical lockouts on the slat actuators. They were not required to certify the airplane. But then, why build a 'just barely good enough' airplane? The #1 engine departed the wing, taking the hydraulic lines that run along the front spar with it. This cause a major loss of pressure in the slat actuators which were then pushed back into the wing by dynamic pressure. The assymetrical loss of the leading edge high lift devices was a major contributing factor in this crash. Interestingly enough, the DC-10 is often used as an illustration of how NOT to design hydraulic systems. -- Terry drinkard@bcstec.boeing.com "Anyone who thinks they can hold the company responsible for what I say has more lawyers than sense." From kls Tue Dec 1 02:10:50 1992 Newsgroups: sci.aeronautics.airliners Path: news From: duchad@rpi.edu (David Benedict Ducharme) Subject: GE aerospace X-Submission-Date: Tue, 1 Dec 1992 07:23:35 GMT Message-ID: Approved: kls@ohare.Chicago.COM Organization: Rensselaer Polytechnic Institute, Troy, NY Sender: kls@ohare.Chicago.COM X-Submission-Message-Id: Date: 01 Dec 92 02:10:50 PST I was in an argument the other day that I hope that can be cleared up here was GE aerospace sold straightout, or was it merged with MM. sold straightout meaning that they no longer have any say in those dealing anymore. thanks email is fine duchad@rpi.edu From kls Tue Dec 1 02:10:51 1992 Newsgroups: sci.aeronautics.airliners Path: news From: kls@ohare.Chicago.COM (Karl Swartz) Subject: Re: Boeing Book X-Submission-Date: Tue, 1 Dec 1992 08:45:34 GMT References: Message-ID: Approved: kls@ohare.Chicago.COM Organization: Chicago Software Works X-Submission-Message-ID: <1992Dec1.084534.12650@ohare.Chicago.COM> Sender: kls@ohare.Chicago.COM Date: 01 Dec 92 02:10:51 PST In article michael@is3000.bmr.gov.au (System Support) writes: >Maybe someone can help me. I'm trying to get hold of a new book about >Boeing entitled something like "The Boeing Story". Does anyone have >any ideas about such a book, author? As Christopher Davis pointed out, "Legend and Legacy" sounds like the book you're after. There was a review posted on rec.aviation recently; I believe Robert Dorsett may have it archived on rascal.ics.utexas.edu if you have anon ftp access. A good source for books on airliners (though not this particular one; the review mentioned B. Dalton's as having it) is "Just Plane Crazy" in Miami, the retail arm of World Transport Press which publishes both Airliners and Airliners Monthly News, and is the U.S. distributor for the JP fleet lists. The best way of reaching them is by phone, and thankfully (for those of us on the west coast of the U.S.) they are open late, until 10 pm Eastern Time. Saturdays until 6 pm and they open at 10 am. From the 48 continental states try 1-800/875-6711; elsewhere, 1-305/477-7163. They also have a 24-hour FAX line at 1-305/599-1995. By mail, write to Airliners Catalog P.O. Box 521238 Miami, FL 33152-1238 USA If you happen to be visiting Miami, they're right next to Miami International Airport at 1200 NW 72nd Ave. in the Perimeter Road Building. All that said, I have no affiliation with them other than as a satisfied customer on several occasions. Their latest catalog includes several new books which may be of interest, including Douglas DC-8: A Pictorial History Grounded: Frank Lorenzo and the Destruction of Eastern Airlines Too bad he's gone from the scene -- we'd have great fun picking on him!!! :-) Jet Airliner Production List (2nd Edition) Mine has been on order for a week; damned Christmas mail must have it trapped along the way! -- Karl Swartz |INet kls@ditka.chicago.com 1-415/854-3409 |UUCP uunet!decwrl!ditka!kls |Snail 2144 Sand Hill Rd., Menlo Park CA 94025, USA Send sci.aeronautics.airliners submissions to airliners@chicago.com From kls Tue Dec 1 13:54:48 1992 Newsgroups: sci.aeronautics.airliners Path: news From: Christopher Davis Subject: Re: Boeing Book X-Submission-Date: Tue, 1 Dec 1992 06:17:08 -0500 References: Message-ID: Approved: kls@ohare.Chicago.COM Sender: kls@ohare.Chicago.COM X-Submission-Message-Id: <199212011117.AA07785@loiosh.eff.org> Date: 01 Dec 92 13:54:48 PST ckd> == Christopher Davis > Michael> == System Support Michael> Maybe someone can help me. I'm trying to get hold of a new Michael> book about Boeing entitled something like "The Boeing Story". Michael> Does anyone have any ideas about such a book, author? Any help Michael> would be much appreciated. ckd> This might be the Robert Serling _Legend & Legacy_, which I hope ckd> to find the time to do a book review on, eventually. *Very* good ckd> book. (I don't have an ISBN handy, the book's at home.) Okay, I'm at home now...information follows in (hopefully) refer format: %A Robert J. Serling %T Legend & Legacy: The Story of Boeing and Its People %P 480 %I St. Martin's Press %C New York %D 1992 %O ISBN 0-312-05890-X %Y US$24.95 (hc) There is a photo section in the center, including some interesting "747-300" shots (a trimotor design in the same class as the L-1011/DC-10, which obviously never saw production), the 2707 SST mockup, the Boeing hydrofoil (one of which I once rode on between Victoria BC and Seattle as the _Flying Princess_), and some others. It does not include the Boeing light rail vehicle (streetcar) that Boston and San Francisco wound up with. (Interesting note: I have had the [probably extremely rare] experience of having been a passenger in Boeing vehicles below ground, at ground level, at (and slightly above) sea level on water, and of course the traditional jet cruising altitudes. Now if I could have only managed a trip to the Moon; both the Saturn first stage and the Lunar Rover...) From kls Tue Dec 1 13:54:51 1992 Newsgroups: sci.aeronautics.airliners Path: news From: palmer@icat.larc.nasa.gov (Michael T. Palmer) Subject: Flight Envelope Protection (was: TV prog. on 777) X-Submission-Date: 1 Dec 92 13:19:47 GMT References: Message-ID: Approved: kls@ohare.Chicago.COM Organization: NASA Langley Research Center, Hampton, VA USA Sender: kls@ohare.Chicago.COM X-Submission-Message-Id: Date: 01 Dec 92 13:54:51 PST Robert Dorsett writes: >As I understand it, the FBW system is the only way the pilots can signal >the actuators. Boeing is simply providing a "conventional" control law and >interface, with "protections" that can be over-ridden by the pilot, if >necessary. Redundancy/backup is at the hardware level, not in alternate >select modes. [etc] >On the other hand, I do think it's a positive step that Boeing's not "re- >writing" the book by offering *artificial* control laws, as Airbus is doing. >Thus, to override the protections, the pilots just need to push or pull >*harder,* or click an overrride button: they don't have to deal with or >anticipate the effects of *four* distinct control law modes, and the many >permutations within each mode, depending upon system status, as is the case >with the A3[2-4]0. This is correct, and highlights a very important distinction between the approaches to flight envelope protection being taken by Boeing & Airbus. The B-777 will have protections, but as you noted the crew can override them by using excess force on the control column. So, the airplane will make it more difficult to do something it thinks shouldn't be done, but will always leave the final decision to the crew. In contrast, the protection on the A320 *cannot* be overridden - you either get switched into an alternate control mode, or your inputs are ignored. This has some serious consequences. For example, in the China Airlines B-747 incident 300 nm northwest of San Francisco in 1985 (NTSB/AAR-86-03), the crew was forced to overstress (and structurally damage) the horizontal tail surfaces to recover from a roll and near-vertical dive following an automatic disconnect of the autopilot when it could no longer compensate for an asymmetric thrust condition. At the time of disconnect, full rudder was engaged to one side and the crew was unaware of this. The crew recovered control with about 10,000 ft of altitude left (from an original high-altitude cruise). It is very likely that if the aircraft had prevented the crew from initiating control commands that would lead to aircraft damage, the aircraft (and passengers) would have been lost. Unfortunately, it appears that engine manufacturers may be heading down the same path as Airbus with respect to their electronic engine controllers. I can't remember which engine it was, but I remember reading that when the controller detects a condition for which the proper action is to shut the engine down, it will do it itself AND THE CREW CANNOT OVERRIDE THIS ACTION. Now, this may seem like a good idea on paper, but remember the Eastern L-1011 out of Miami in 1983 (NTSB/AAR-84-04) with the triple engine failure because the oil seals were missing? Can you imagine the tragic result if the engines had ALL detected this condition (in flight) and shut themselves down? It seems to me that letting the crew decide to sacrifice an engine to save the airframe is probably a good idea. If nothing else, I hope I have brought up some topics that deserve discussion among readers of this newsgroup. After all, aren't we the ones in positions to influence our industry (all in our own way, of course)? -- Michael T. Palmer, M/S 152, NASA Langley Research Center, Hampton, VA 23681 Voice: 804-864-2044, FAX: 804-864-7793, Email: m.t.palmer@larc.nasa.gov PGP 2.0 Public Key now available -- Consider it an envelope for your e-mail From kls Tue Dec 1 13:54:52 1992 Newsgroups: sci.aeronautics.airliners Path: news From: palmer@icat.larc.nasa.gov (Michael T. Palmer) Subject: Re: Airline Software-safety database (RISKS-14.08) X-Submission-Date: 1 Dec 92 14:57:47 GMT References: Message-ID: Approved: kls@ohare.Chicago.COM Organization: NASA Langley Research Center, Hampton, VA USA Sender: kls@ohare.Chicago.COM X-Submission-Message-Id: Date: 01 Dec 92 13:54:52 PST Pete Mellor writes: [etc] >Incidents in flight must (or should) be reported via offical channels by the >crews. These reports drive the manufacturers' quality improvement programmes. >After the fault which caused an incident has been diagnosed, it may result in >an OEB or similar, and in a modification. [etc] >Databases of such incident reports are not generally widely accessible. >Published reports sometimes appear, however. In addition, there are channels >for anonymous reporting of incidents. In the UK, "CHIRP" is such a forum. In >the US, I believe the FAA used to run such a scheme, but it was compromised >when the guarantee of anonymity was removed. >For further information I suggest you contact ALPA. [etc] >I stand to be corrected if anyone *does* know of an official channel for >public access to flight incident and system fault reports. Okay, here goes. In the USA, NASA and the FAA have teamed up to deploy the Aviation Safety Reporting System (ASRS), which is managed by folks at NASA Ames Research Center, Moffet Field, CA. I believe the original poster misunderstood the intent and operation of the ASRS, so I will try to fill in some details. Anyone from the ASRS Office at Ames is free to jump in and correct any mistakes I make. The ASRS collects incident and accident reports for all aviation-related activities, including flight, air traffic control, and maintenance. The way it works is that whenever an individual is involved in an incident, he or she is encouraged to submit an ASRS report describing what happended, why they think it happended, and what should be done to correct the problem so it doesn't happen again. The fact that a report is submitted (receipts are kept for proof) is accepted by the FAA as a sheild from legal retribution except in cases of gross misconduct or criminal intent. There is no anonymity, per se, in the filing of the report. When the reports are collected and entered into the ASRS database, they have keywords identified to allow easier searches on related topics. At this time, they are also "de-identified." This may be what the original poster misunderstood as a guarantee of anonymity. During de-identification, all references that would lead a later reader of the report to be able to identify the exact person, place, and aircraft (by N-number) involved are removed and replaced by generic terms. This protects filers from, say, unscrupulous company or government people that seek to harrass them later for possibly unrelated reasons. Now, for the good part. This database, which is HUGE, is publicly "accessible." However, the access is controlled, and database search requests must be submitted to and approved by the ASRS office. This helps to prevent frivolous or duplicative use, which could rack up *very* large costs *quickly* due to the sheer size of the database. The actual searches are performed by Battelle, Inc. under contract to NASA Ames. The Ames (NASA) person to call for more information about the ASRS is Vince Mellone at (415) 969-3969 or (415) 604-6467. The database search requests are actually sent to: Battelle ASRS Office 625 Ellis Street, Suite 305 Mountain View, CA 94043 If you have never used the ASRS before, I suggest you give Vince a call first to find out what information you need to provide in your search request so the Battelle people can help you find what you're actually looking for. Note that the ASRS database is NOT "on-line" such that anyone could just dial in and peruse it at their leisure. So, this may not suffice for what the original poster had in mind. However, I would venture to guess that any database with enough information in it to provide a reasonable basis for design would end up being so large that access MUST be controlled somehow. I think the ASRS is a good compromise. -- Michael T. Palmer, M/S 152, NASA Langley Research Center, Hampton, VA 23681 Voice: 804-864-2044, FAX: 804-864-7793, Email: m.t.palmer@larc.nasa.gov PGP 2.0 Public Key now available -- Consider it an envelope for your e-mail From kls Tue Dec 1 13:54:52 1992 Newsgroups: sci.aeronautics.airliners Path: news From: palmer@icat.larc.nasa.gov (Michael T. Palmer) Subject: Re: GE aerospace X-Submission-Date: 1 Dec 92 15:24:29 GMT References: Message-ID: Approved: kls@ohare.Chicago.COM Organization: NASA Langley Research Center, Hampton, VA USA Sender: kls@ohare.Chicago.COM X-Submission-Message-Id: Date: 01 Dec 92 13:54:52 PST duchad@rpi.edu (David Benedict Ducharme) writes: >I was in an argument the other day that I hope that can be cleared up here >was GE aerospace sold straightout, or was it merged with MM. >sold straightout meaning that they no longer have any say in those dealing >anymore. My understanding is that it was a merger, not an outright sale, based on a letter yesterday from a friend who works for "Generous Electric Aerospace." Additionally, I believe the FTC has not yet approved the transaction, although I may have missed the announcement. Anyone with first-hand knowledge is encouraged to correct me if I'm wrong. -- Michael T. Palmer, M/S 152, NASA Langley Research Center, Hampton, VA 23681 Voice: 804-864-2044, FAX: 804-864-7793, Email: m.t.palmer@larc.nasa.gov PGP 2.0 Public Key now available -- Consider it an envelope for your e-mail From kls Tue Dec 1 13:54:53 1992 Newsgroups: sci.aeronautics.airliners Path: news From: msb@sq.sq.com (Mark Brader) Subject: Re: Boeing Book X-Submission-Date: Tue, 1 Dec 92 15:45:09 GMT References: Message-ID: Approved: kls@ohare.Chicago.COM Organization: SoftQuad Inc., Toronto, Canada X-Submission-Message-ID: <1992Dec1.154509.8633@sq.sq.com> Sender: kls@ohare.Chicago.COM Date: 01 Dec 92 13:54:53 PST > Wide-Body: the Triumph of the 747 by Clive Irving An excerpt from this forthcoming book appears in the current, i.e. December-January, issue of "AIR & SPACE / Smithsonian" magazine. -- Mark Brader, SoftQuad Inc., Toronto | "Yet Another Wonderful Novelty -- YAWN!" utzoo!sq!msb, msb@sq.com | -- Lee R. Quin This article is in the public domain. From kls Tue Dec 1 13:54:54 1992 Newsgroups: sci.aeronautics.airliners Path: news From: Karl Swartz Subject: A310 Aerobatics X-Submission-Date: Tue, 1 Dec 92 13:53:48 PST Message-ID: Approved: kls@ohare.Chicago.COM Sender: kls@ohare.Chicago.COM X-Submission-Message-Id: Date: 01 Dec 92 13:54:54 PST The latest issue of Airliners (Winter 1992) has a fascinating article on an incident with an Airbus A310. Some tantalizing details of what happened for the fans of leaving the pilot fully in control, but not enough information to trace the incident any further. In the hopes that someone might know more (PLEASE post any relavent details, and of course dicussion is welcome too), and perhaps to turn a few folks onto a good magazine, here's the article, from Tailpieces: A310 Aerobatics ---- ---------- Following an autopilot-coupled go-around, the pilot attempted to counteract the autopilot's programmed pitch-up by pushing forward on the control column. (In most circumstances pushing on the control column disengages the autopilot but automatic disconnect is inhibited in go-around mode. The autopilot should be disconnected or a mode other than go-around should be engaged through the FCU - Flight Control Unit.) As a result of the control inputs, the autopilot trimmed the stabil- izer to -12 degrees (nose up) to maintain the go-around profile, but the elevator was deflected 14 degrees (nose down). After climbing about 600 feet (to around 2,100 feet) the autopilot captured its preselected missed approach altitude and disconnected as the go-around mode was no longer engaged. In the next 30 seconds, the grossly mistrimmed A310 pitched up to 88 degrees and airspeed dropped to less than 30 kt. (The stall warning activated then canceled itself as the airspeed fell below usable computed values and the autothrottle system dropped off.) At 4,300 feet, the A310 stalled, pitching down to -42 degrees while the pilot-applied control inputs showed full up elevator. Airspeed increased to 245 kt then the aircraft bottomed out at 1,500 feet, pulled +1.7 g, then climed rapidly. The second pitch-up reached 70 degrees followed by a stall 50 secondds after the first. The nose dropped to -32 degrees and airspeed rose to 290 kt and the aircraft bottomed out at 1,800 feet. On the third pitch-up (to 74 degrees), the A310 climed to 7,000 ft then stalled again, about 60 seconds after the second stall. This time airspeed reached 300 kt in a -32 degree nose down attitude before the aircraft leveled off at 3,600 feet. The fourth pitch-up reached 9,000 feet but this time the crew's use of thrust and elevator control (and very likely retrimming the stabilizer) prevented a stall and the A310 leveled off at 130 kt. Speed then increased accompanied by another milder pitch-up to 11,500 feet where control was eventually regained. All aircraft systems operatedd in accordance with design specifica- tions. The reaction of ATC (the incident happened at Moscow) or the passengers is not recorded. -- Karl Swartz |INet kls@ditka.chicago.com 1-415/854-3409 |UUCP uunet!decwrl!ditka!kls |Snail 2144 Sand Hill Rd., Menlo Park CA 94025, USA Send sci.aeronautics.airliners submissions to airliners@chicago.com From kls Tue Dec 1 23:15:18 1992 Newsgroups: sci.aeronautics.airliners Path: news From: drinkard@bcstec.ca.boeing.com (Terrell D. Drinkard) Subject: Re: Boeing 747-300 X-Submission-Date: Mon, 30 Nov 92 18:05:53 PST References: Message-ID: Approved: kls@ohare.Chicago.COM Organization: Boeing Sender: kls@ohare.Chicago.COM X-Submission-Message-Id: <9212010205.AA28597@bcstec.ca.boeing.com> Date: 01 Dec 92 23:15:18 PST In article kls@ohare.chicago.com (Karl Swartz) writes: >In article drinkard@bcstec.ca.boeing.com (Terrell D. Drinkard) writes: >>In article jerry@telecom.ksu.edu (Jerry And > >>>I've heard Boeing, McDonnell-Douglas and Airbus all have plans >>>in the works for Really Big Planes in the 600-700 passenger, >>>7500-8000 mile range. > >>The Boeing and the Airbus offerings in this market seem to both hover >>around 600 seats and 7,000 mile range. Takeoff weights in the million >>pound plus range. The anticipated market, as described by John Hayhurst, >>Director of New Large Airplane Division, is only a couple hundred airplanes >>TOTAL. From my knothole, it looks like a prestige fight. > >That's an *awful* lot of cash to dump down a hole simply for bragging >rights. It isn't just bragging rights. Prestige has a market value. There are several airlines who are not out to make a profit. The national airlines of some oil rich countries for example, are not expected to make buck, rather to 'carry the flag'. Thai, as another example bought 747s some years ago largely because of prestige. Therefore, you have some airlines who fly the airplane in the world to make a political statement. > I suppose that's part of "being sporty" but there's also a >real market there -- the Pacific Rim, which is where nearly all the >growth is in the airline industry and which requires those kind of >range figures. The load potential is there too, if not now then well >well within the next 10 to 20 years. Most of the market you refer to can be filled with stretched 747s. This isn't difficult, and it is no doubt being looked at quite closely. However, there is a practical limit to the stretch 747, and the real question is how many airplanes above that limit can you sell? If the answer is pretty small (<100 airplanes maybe?) then it doesn't make any sense to build the monster jet. If it is an appreciable number, then it makes sense to bypass the 747 stretch and go with an all new large airplane. And, as you noted, timing is a big issue. No point in building the thing if no one needs it for two or three years after roll-out. >One aircraft that could punch a major hole in this market would be the >next generation supersonic transport. (HSCT? I can't pick the right >acronym out of my bowl of alphabet soup today ...) *If* built, and at >least for now that is a very big if, this too would be aimed directly >at the Pacific Rim market. The studies I've seen for this bird seem >to be aiming at the mainstream market and not just a very tiny high- >priced market like the Concorde. Exactly. Last word I got was ticket prices about 20% above full-price 747 coach was a target (granted that was a couple of years ago). The HSCT is my drool-job. I doubt I'll ever get to work on it, but it really fires my imagination. Don't look for one before 2010; we seem to be a bit short on engine and materials technology. -- Terry drinkard@bcstec.boeing.com "Anyone who thinks they can hold the company responsible for what I say has more lawyers than sense." From kls Tue Dec 1 23:15:20 1992 Newsgroups: sci.aeronautics.airliners Path: news From: gary@maestro.mitre.org (Gary Bisaga) Subject: Re: pair (sorry, couldn't resist the pun) X-Submission-Date: Tue, 1 Dec 1992 18:01:51 GMT References: Message-ID: Approved: kls@ohare.Chicago.COM Reply-To: gary@maestro.mitre.org (Gary Bisaga) Organization: The MITRE Corporation, McLean, Va Sender: kls@ohare.Chicago.COM X-Submission-Message-Id: <1992Dec1.180151.4587@linus.mitre.org> Date: 01 Dec 92 23:15:20 PST In article , weiss@wright.SEAS.UCLA.EDU (Michael Weiss) writes: |> |> All of my repair information comes from my girlfriend's brother, who works as |> a maintenance guy for SkyWest (a commuter airline that operates also as the |> Delta Connection in Los Angeles, Palm Springs, Phoenix, and Las Vegas |> primarily). He said that DC-10s are notorious for repairs being all-day |> operations, whereas Boeing's 737-300 and -400, 747-300 and -400, 757, and 767 |> have self-diagnostic systems that go so far as to direct the location of the |> repair instructions down to the page, turning the repairs into a half-day |> operation instead. Note that I cannot verify this information, but I see no |> reason to dispute it. Nor would I - but you're comparing apples to oranges. The other aircraft you mention probably have much more extensive electronic maintenance aids since most have much more extensive avionics in general. It wouldn't surprise me if there was also more electronic diagnosis. The MD-11, of course, is just as highly outfitted with electronics as any of those others - and the FMC design is newer as well, if I'm not mistaken - so a comparison with the MD-11 would almost certainly be different. -- Gary Bisaga (gbisaga@mitre.org) From kls Tue Dec 1 23:15:21 1992 Newsgroups: sci.aeronautics.airliners Path: news From: ncole@nyx.cs.du.edu (Noah Cole) Subject: Re: The Sporty Game -- Boeing 757 X-Submission-Date: Tue, 1 Dec 92 18:10:57 GMT References: Message-ID: Approved: kls@ohare.Chicago.COM Organization: Macalester College, St. Paul Minnesota USA Sender: kls@ohare.Chicago.COM X-Submission-Message-Id: <1992Dec1.181057.11073@mnemosyne.cs.du.edu> Date: 01 Dec 92 23:15:21 PST gregory@bcstec.ca.boeing.com (Greg Wright) writes: > I think that you will find that every attempt at a shortened version >of one of our planes has had limited success. Take the 747SP for example. >Airlines tend not to like the sorted versions very much. In $/seat or $/mile >these versions are too expensive to run. There is a real problem having >too much engine or wing with them. We find it is better to stretch if >anything.... >Greg How many airlines use 747SP's today? I have a poster from Popular Mechanics that was around the arrival of the 747-400 with a drawing of a United 747SP and it said that the 747SP set a record flying from Payne Field, Washington to South Africa. Was that SAA? Who flies the 747SP Today and on what routes? -Noah Cole -- Noah Cole "Outside is America, NCOLE@MACALSTR.EDU Macalester College and also the car park" ncole@nyx.cs.du.edu St. Paul, MN 55105 - Bono, 27 December 1989 cncole@coos.dartmouth.edu 612-696-7388 Dublin aj909@cleveland.freenet.edu From kls Tue Dec 1 23:15:22 1992 Newsgroups: sci.aeronautics.airliners Path: news From: cid@athena.mit.edu (Derek H Cedillo) Subject: Re: GE aerospace X-Submission-Date: Tue, 1 Dec 1992 23:48:26 GMT References: Message-ID: Approved: kls@ohare.Chicago.COM Organization: Massachusetts Institute of Technology Sender: kls@ohare.Chicago.COM X-Submission-Message-Id: <1992Dec1.234826.25858@athena.mit.edu> Date: 01 Dec 92 23:15:22 PST In article duchad@rpi.edu (David Benedict Ducharme) writes: >I was in an argument the other day that I hope that can be cleared up here >was GE aerospace sold straightout, or was it merged with MM. >sold straightout meaning that they no longer have any say in those dealing >anymore. According to UPI reports on the dealing the whole deal is worth $3.05 Billion In the deal GE Gets $1 billion in convertible preffered stock. I hate to see GE lose a part of itself, but it was a wise move. Does anyone know what is to become of the Ft. Worth, IN plant? I didnt see it listed in the buisness locations to be taken over by MM. Is there going to be some work done by GE somewhat autonomously still in the biz? Thanks, Derek From kls Tue Dec 1 23:15:22 1992 Newsgroups: sci.aeronautics.airliners Path: news From: Christopher Davis Subject: Quote of the Day X-Submission-Date: Tue, 1 Dec 1992 19:00:23 -0500 Message-ID: Approved: kls@ohare.Chicago.COM Sender: kls@ohare.Chicago.COM X-Submission-Message-Id: <199212020000.AA08356@eff.org> Date: 01 Dec 92 23:15:22 PST Seen in Aviation Week & Space Technology, November 23, 1992, p. 73: ``We and Boeing have completely different philosophies,'' and Airbus official said. ``Ours is based on experience--Boeing is quite different. Airbus has a tremendous heritage of technology; Boeing does not.'' I will refrain from comment other than to say that technology is not the object of commercial airliner design--safety and efficiency are. From kls Tue Dec 1 23:15:23 1992 Newsgroups: sci.aeronautics.airliners Path: news From: rdd@cactus.org (Robert Dorsett) Subject: Re: Boeing Book X-Submission-Date: Tue, 1 Dec 92 21:29:20 CST References: Message-ID: Approved: kls@ohare.Chicago.COM Organization: Capital Area Central Texas UNIX Society, Austin, Tx Sender: kls@ohare.Chicago.COM X-Submission-Message-Id: <9212020329.AA05534@cactus.org> Date: 01 Dec 92 23:15:23 PST In article you write: >In article michael@is3000.bmr.gov.au (System Support) writes: >>Maybe someone can help me. I'm trying to get hold of a new book about >>Boeing entitled something like "The Boeing Story". Does anyone have >>any ideas about such a book, author? > >As Christopher Davis pointed out, "Legend and Legacy" sounds like the >book you're after. It might also be "Boeing: Planemaker since 1916," by one Philip M. Bowers. It's an exhaustive review of all the airplanes (and variants) Boeing's produced, sort of a mini Jane's. It's a long book (over 600 pages). It appeared at Bookstop here in Austin sometime this summer; it's about $36. --- Robert Dorsett rdd@cactus.org ...cs.utexas.edu!cactus.org!rdd From kls Tue Dec 1 23:15:24 1992 Newsgroups: sci.aeronautics.airliners Path: news From: drinkard@bcstec.ca.boeing.com (Terrell D. Drinkard) Subject: Re: 757 highest thrust to weight ratio ? X-Submission-Date: Wed, 2 Dec 1992 03:32:35 GMT References: Message-ID: Approved: kls@ohare.Chicago.COM Organization: Boeing Sender: kls@ohare.Chicago.COM X-Submission-Message-Id: Date: 01 Dec 92 23:15:24 PST In article Bob Coggeshall writes: >I've heard that the 757 has a very high thrust to weight ratio. Just >how high is it ? Is it the highest of any commercial jetliner ? My back-of-the-envelope calculations would suggest about a .38 thrust to weight ratio. That is assuming the RR RB211-535E4B gives around 42,000 lb thrust (SLST) at a MTOW of 220,000 lb. I'm not aware of anyone who actually bought that configuration, though. Oh, and on their acceptance flight tests, (I got to fly one on a 737-400 once) these babies are *empty* and climb like homesick angels. I have seen VSIs pegged (6,000fpm+). >I've also heard that there is a boeing-internal video of it >during tests doing an [almost?] straight vertical climb. I've never seen it, but then I don't do any flight test stuff. >What are the facts here ? Excellent question! :-) -- Terry drinkard@bcstec.boeing.com "Anyone who thinks they can hold the company responsible for what I say has more lawyers than sense." From kls Wed Dec 2 03:49:46 1992 Newsgroups: sci.aeronautics.airliners Path: news From: fxm4993@hertz.njit.edu (Farhan Muhammad) Subject: Concorde-landing procedures X-Submission-Date: Tue, 1 Dec 1992 21:50:26 GMT Message-ID: Approved: kls@ohare.Chicago.COM Organization: New Jersey Institute of Technology, Newark, N.J. Sender: kls@ohare.Chicago.COM X-Submission-Message-ID: <1992Dec1.215026.29278@njitgw.njit.edu> Date: 02 Dec 92 03:49:46 PST Landing approach of Concord. --------------------------- Excerpt from the book "Flying Concorde". '200 feet.' The flight engineer, who cannot see the runway since he is behind and a little below the pilots, is reading the radio altimeters which are bouncing signals off the groiund to determine height to within a foot. '100 feet ... '50, 40, 30, 20, 15.' At 40 feet the autothrottles are disconnected by pressing a small button on either side of the levers. A slight backward movement of the stick slows the rate of descent a little, pitching up perhaps a degree, from 10.5 to 11.5 degrees. The pilot's eye is still 75 feet above the runway (about the same height as a 747 pilot's) and he aims at a point about 2000 feet down the runway, knowing that the main wheels, trailing below and behind, will arrive well before that point. From about 100-feet altitude we have been about to hear the 'ground effect' starting. A large wing, approaching the ground, begins at some point to squeeze the air between it and the surface, settling into a cushion of its own making. The large wing area and the high angle of attack make this effect more pronounced on Concorde than on conventional swept-wing aircraft, and seems to throw back some of the noise of air rushing into the engines. That is what it sounds like, at any rate. At fifteen feet, the throttles are closed. The immediate effect is a tendency for the nose to drop. Landing is largely a matter of countering this tendency as the aeroplane settles into its ground effect. A slow backward movement of the stick keeps the nose where it is. The rate of movement depends on the strength of the pitch-down tendency. Good landings are simply a question of getting the balance right, so that the nose stays rock-steady against the far end of the runway. Once on the ground there is a second landing to perform- the nosewheel is still a long way in the air. A nudge forward with the stick to get it on its way, followed by a backward movement to cushion its descent, and all the wheels have arrived. As soon as the main wheels are on the ground, reverse thrust is engaged. Once the nosewheel is on, power is increased in reverse to kill the speed. At this point the stick is pushed fully forward to keep the nosewheel on the ground, as the low-slung engines, producing their reverse thrust below the body line, would tend to raise it again if allowed. The elevons are still effective, though, and the nosewheel is kept firmly on the runway as braking starts. The powerful carbon discs get to work, the speed reduces, and the runway, which on touchdown didn't seem as long as it shuold be, with the eye still 35 feet in the air, lengthens again to its proper shape. '100 knots.' The two outboard throttles are pushed into reverse idle. '75 knots.' The inner follow. '40 knots.' All the engines are returned to forward idle power and the aeroplane is nearly ready to be turned off. It is easy, in any aeroplane, to think that the speed just after landing is lower than it really is, so a glance at the INS groundspeed is useful here. Once we have turned off the runway, he nose is raised to the 5-degree position again, and the two inboard engines are shut down (at this weight, at the end of a flight, two engines provide quite enough power for taxiing). From runway 31 Left it is a longish taxi round to the British Airways terminal on the other side of the circle of airport buildings. As we approach it the time is a few minutes after 3.00 pm in London. Here, in New York, we are nearing out scheduled arrival time of 10.15 - apparently an hour before we left our gate at Heathrow. Just under four hours, gate to gate: an hour less than it takes the earth to rotate through the angular distance separating London and New York. Three and a half hours' flight-time to cover three and half thousand miles - an average speed of a thousand miles an hour. Some of the disembarking passengers show signs of excitement - it has been their first supersonic flight, and it will be a while before they have sorted out the mixture of unreality and normality they have experienced. Others, the majority now, take it all for granted - they have probably used Concorde several times. And it has been normal. So it should be by now; but this normality had to be present from the first flight, in January 1976, and that took a little doing. From kls Wed Dec 2 03:49:48 1992 Newsgroups: sci.aeronautics.airliners,rec.travel.air Path: news From: rdd@cactus.org (Robert Dorsett) Subject: Re: Airbus safety X-Submission-Date: Wed, 2 Dec 92 01:30 PST References: <1992Nov26.000453.4729@cactus.org> <1992Dec01.025604.17493@news.mentorg.com> Message-ID: Approved: kls@ohare.Chicago.COM Summary: Organization: Capital Area Central Texas UNIX Society, Austin, Tx Expires: X-Submission-Message-Id: Sender: kls@ohare.Chicago.COM Date: 02 Dec 92 03:49:48 PST In article nelson_p@apollo.hp.com (Peter Nelson) writes: >>Remember that the standard definition of an airline pilot's job is 99.999% >>pute boredom, and 0.001% pure terror (I forget where this quote came from, >>and the ratios may be incorrect) - if this is anything like true, maybe >>human pilots really are on the edge of extinction ? > > _New Scientist_ had an article devoted to this about 3 issues ago. > > Basically they said that as the % of "pilot error" crashes increases > we may already be at the point where more lives would be saved by > pilotless airplanes. This is certainly a debatable contention. Airbus certainly seems to believe it: but it's also in the business of selling products "differentiated" by their style of protection. The reality of the situation is that the safety record has remained pretty much constant since the late 1970's--note: not the early 1980's, when the first automated aircraft were introduced. It has stabilized at about 1500 lives per year. What IS true is that as mechanical failures are isolated and fixed, the proportion of pilot-induced failures must, necessarily, in- crease. The problem facing the industry is how to get the death rate to zero: we don't seem to wish to recognize that, with current technology, it may not be possible, and that we may see steadily diminishing returns in our efforts to do so. The proportion of pilot-error incidents range from 60% to 95% of the total number of crashes, with Airbus generally supporting the latter figure. The problem, of course, is how one defines *pilot error*. Is "pilot error" pushing the wrong switch? Suddenly pushing, instead of pulling, on the yoke? A psychotic break? Naturally, none of these things: in all instances, "pilot error" has been a case of a broader *system* failure, the system being a combination of the pilot, his peers, the airplane, its interface, the airline, and the regulatory backdrop. In precious few cases were the pilots "asleep at the wheel," or criminally incapacitated. What is debatable is how many of these factors can be eliminated, simply by increasing automation, reducing oversight authority, or transferring responsibility for operations to ground controllers. It is EXTREMELY important to realize that we're struggling against an *ideal*: no crashes. It is also important to note that if, indeed, pilot error is *increasing*, then it's probably a result of over-automation in the cockpit, since virtually no other part of the infrastructure has changed since the late 70's. The simple, short-term solution is to reduce the degree of auto- mation, or at least bring the pilot back into the loop (not necessarily exclusive concepts!). You would have a hard time convincing me that the number of fundamental errors would not increase GREATLY with ground-based oversight, that the safety margins would not go DOWN, as people fundamentally distanced from the reality of a flight have a go/no-go say. >And moreover, the technology to do this either > already exists or is close at hand. The technology isn't close to create safe, fully autonomous aircraft. And, in lack of that, we'd need ground-based control, with a high degree of automation in-flight. The infrastructure needed to support this would be exhorbitantly expensive (and who would run it: the dispatch controller, who just sees a number on a status board, and wants to make his schedule? A government specialist?). In addition, we'd almost certainly be replacing the existing social and interface problems that currently exist in the *air*, with a new, untried set of problems on the ground. More than any other trend in aviation, this sort of talk, much of which seems to originate with Airbus, and which deliberately, blithely underrates the problems involved in reducing pilot authority, worries me that we've passed the point of negative returns. The problem, again, is not automation: to paraphrase Don Norman, it's appropriate feedback. Or, in mil-speak, the minimum capability needed to carry out the mission requirements. There is abundant evidence that, in fact, this requirement can be met with *less* automation, *better* interfaces, and keeping the pilots in the loop. However, there is also evidence that flight deck design is engineering-and marketing-driven, and that "good" human factors does not play a primary role in flight deck design, except as a rubber-stamp on a pre-existing systemic intent. There is also increasing evidence that hybrid designs: with high degree of automation, and relegating the pilot to a passive, supervisory role, out of the loop, are *not* the way to go. > They said, however, that it would be a public-relations nightmare and > felt there was no hope of selling the idea to the public. Was Bernard Ziegler the author of this article, perchance? :-) It's symptomatic of the technocratic solution: full-speed ahead with quantifiable solutions, damn the pilots. Even if we don't fully understand the consequences of the resulting environment, when these solutions have to ultimately interact with human beings, at least at some level. > People will > continue to cite those cases where coolness or quick thinking on > the part of the crew did save the airplane or at least many lives. I wouldn't. Rather, I would ask how well we understand the *totality* of in-flight incidents and actions, which are corrected by appropriate air- manship. An old, true saying, is that a good pilot is a pilot who doesn't have to show he's a good pilot. Is the capability of being able to maintain control in a thunderstorm really that relevant, when 99% of all pilots would simply have flown around the same thunderstorm? We can automate easily quantifiable issues: simple tasks. Judgement and airmanship has thus far evaded us, on all levels. Until we get a grip on it, talk of fully autonomous aircraft or ground control is nothing more than science fiction. --- Robert Dorsett rdd@cactus.org ...cs.utexas.edu!cactus.org!rdd From kls Wed Dec 2 03:49:49 1992 Newsgroups: sci.aeronautics.airliners Path: news From: kls@ohare.Chicago.COM (Karl Swartz) Subject: 747SP X-Submission-Date: Wed, 2 Dec 1992 10:22:10 GMT References: Message-ID: Approved: kls@ohare.Chicago.COM Organization: Chicago Software Works Sender: kls@ohare.Chicago.COM X-Submission-Message-ID: <1992Dec2.102210.15645@ohare.Chicago.COM> Date: 02 Dec 92 03:49:49 PST In article ncole@nyx.cs.du.edu (Noah Cole) writes: >How many airlines use 747SP's today? I have a poster from Popular Mechanics >that was around the arrival of the 747-400 with a drawing of a United >747SP and it said that the 747SP set a record flying from Payne Field, >Washington to South Africa. Was that SAA? Who flies the 747SP Today >and on what routes? I believe United still has the largest fleet, with the ten 747SP-21s which were acquired from Pan Am in February, 1986. They also got a -27 in that deal (originally owned by Braniff) but it was sold just a few months ago. (One reference I have shows Pan Am as having had a grand total of 13 747SPs -- I think they had another two ex-Braniff planes but I'm not sure.) United first used the 747SPs on the trans-Pacific routes. Currently, they are used primarily for flights to South America, from JFK and perhaps other US airports. There's also a daily SFO-JFK round-trip. They also tend to show up as fill-ins on random other 747 flights -- I have often flown on them SFO-ORD when they subbed for the 747-100 scheduled for the flight, and I've seen them in LHR as well. American had a pair of 747SP-31s (built for TWA) acquired for their DFW-NRT route. The MD-11 is now used on that route; AA last used their pair of SPs for JFK-LHR before retiring them last summer. QANTAS and South African each have a pair, not surprising how far away they are from most everything. I think QANTAS bought theirs for SYD- SFO or perhaps LAX, though I think the US routes are now all 747-400 or 767-300(ER). South African actually owns five but seems to lease them out fairly regularly. After United and South African, Air China (mainland), China Airlines (Taiwan), and Air Mauritius probably have the largest fleets with four each. I see the CAL ones at SFO on occasion. The Air Mauritius planes, three of which are leased from South African, are used on routes to LHR and probably other European cities from Mauritius which is in the Indian ocean. Beyond that, the operators are pretty random -- Oman has one and the United Arab Emirates have two, all of which are listed as being used for Royal Flight. Saudia and Korean each have a pair. Others are tucked away in various obscure places which I haven't yet stumbled upon, or have and have since forgotten. Oh yes, it was a South African 747SP delivery that held a record for longest flight (time or distance or both) for a jetliner or some such. That may still stand but I wouldn't be too surprised if a 747-400 had subsequently established a new record. -- Karl Swartz |INet kls@ditka.chicago.com 1-415/854-3409 |UUCP uunet!decwrl!ditka!kls |Snail 2144 Sand Hill Rd., Menlo Park CA 94025, USA Send sci.aeronautics.airliners submissions to airliners@chicago.com From kls Wed Dec 2 03:49:50 1992 Newsgroups: sci.aeronautics.airliners Path: news From: kls@ohare.Chicago.COM (Karl Swartz) Subject: Re: GE aerospace X-Submission-Date: Wed, 2 Dec 1992 10:33:34 GMT References: Message-ID: Approved: kls@ohare.Chicago.COM Organization: Chicago Software Works Sender: kls@ohare.Chicago.COM X-Submission-Message-ID: <1992Dec2.103334.15730@ohare.Chicago.COM> Date: 02 Dec 92 03:49:50 PST In article cid@athena.mit.edu (Derek H Cedillo) writes: >In article duchad@rpi.edu (David Benedict Ducharme) writes: >>I was in an argument the other day that I hope that can be cleared up here >>was GE aerospace sold straightout, or was it merged with MM. >According to UPI reports on the dealing the whole deal is worth $3.05 Billion >In the deal GE Gets $1 billion in convertible preffered stock. Right. GE sold GE Aerospace, based in Valley Forge, PA, and GE Government Services, based in Cherry Hill, NJ, to Martin Marietta, but got a substantial chunk of MM in return as well as two seats on the MM board. The agreement also includes provisions for a fair amount of work to continue at GE R&D. So, while it was an outright sale, GE has by no means washed their hands of the whole thing. >Does anyone know what is to become of the Ft. Worth, IN plant? >I didnt see it listed in the buisness locations to be >taken over by MM. What division is that? Note that the transaction did *not* include the GE division most relavent to this newsgroup, namely GE Aircraft Engines, based jointly in Evendale, OH (commercial) and Lynn, MA (millitary). >Is there going to be some work done by GE somewhat autonomously >still in the biz? Yes, Aircraft Engines Div. -- Karl Swartz |INet kls@ditka.chicago.com 1-415/854-3409 |UUCP uunet!decwrl!ditka!kls |Snail 2144 Sand Hill Rd., Menlo Park CA 94025, USA Send sci.aeronautics.airliners submissions to airliners@chicago.com From kls Wed Dec 2 03:49:50 1992 Newsgroups: sci.aeronautics.airliners Path: news From: cyberoid@stein.u.washington.edu (Robert Jacobson) Subject: Re: The Sporty Game -- Boeing 757 X-Submission-Date: Wed, 2 Dec 1992 10:34:15 GMT References: Message-ID: Approved: kls@ohare.Chicago.COM Organization: WORLDESIGN, Seattle X-Submission-Message-Id: <1992Dec2.103415.11330@u.washington.edu> Sender: kls@ohare.Chicago.COM Date: 02 Dec 92 03:49:50 PST As a frequent air traveler, I find the 757 to be positively the most uncomfortable aircraft now flying. One can begin with the ubiquitous TV monitors hanging from the ceilings every few rows, which cannot be dimmed or turned off even on a red-eye, and progress to the remarkable number of seats that can be squeezed into row upon row of stifled passengers. It may be a technical feat, but I know instruct my travel agent to pass on any flight requiring me to take a 757. Yeck. Bob Jacobson From kls Wed Dec 2 03:49:51 1992 Newsgroups: sci.aeronautics.airliners Path: news From: kls@ohare.Chicago.COM (Karl Swartz) Subject: Re: 757 highest thrust to weight ratio ? X-Submission-Date: Wed, 2 Dec 1992 11:27:34 GMT References: Message-ID: Approved: kls@ohare.Chicago.COM Organization: Chicago Software Works Sender: kls@ohare.Chicago.COM X-Submission-Message-ID: <1992Dec2.112734.15956@ohare.Chicago.COM> Date: 02 Dec 92 03:49:51 PST In article drinkard@bcstec.ca.boeing.com (Terrell D. Drinkard) writes: >In article Bob Coggeshall writes: >>I've heard that the 757 has a very high thrust to weight ratio. Just >>how high is it ? Is it the highest of any commercial jetliner ? >My back-of-the-envelope calculations would suggest about a .38 thrust to >weight ratio. That is assuming the RR RB211-535E4B gives around 42,000 lb >thrust (SLST) at a MTOW of 220,000 lb. I'm not aware of anyone who >actually bought that configuration, though. The various RB.211-535 variants used on the 757 range from 37,400 lb thrust up to 43,100; the PW2037 is rated at 38,250. Most airlines choose a 230,000 or 240,000 lb MGTOW, however. The only one I could find on a quick scan with only 220,000 was Northwest, which with the PW2037 works out to 0.3477. Curiosity drove me to dig out the March 16, 1992 AW&ST with the big specifications section and look up the power/weight ratios for some common transports. Where a choice was available I tried to pick the heaviest version with the highest thrust verion of a common engine type, and tossed in a couple of other interesting cases. Over 24 samples the power/weight ratios ranged from 0.2428 to 0.3477. The higher figure is the 757 in Northwest's configuration; a better comparison point is the 240,000 MGTOW 757 which comes in at 0.3187, trailing only the A320/A321, MD-90, and 737-500 (and of course the lighter 757). Not surprisingly, the top 11 were all twins. At the other end of the scale, the heaviest 737-200 Advanced, at 0.2488, was only ahead of the 727-200 Advanced. Judging by these numbers it would seem the 737-200 is a rather underpowered aircraft. Subjectively, pilots I've talked to love the 757 for its abundant power and from the cabin once I was impressed with how quickly a 757 got us out of a wind-shear situation at O'Hare. Another factor that may give an illusion of power is that most 757s are used on medium-range routes, yet they've got a 4550 mile range. Typical flights probably carry a light fuel load and thus may be well short of MGTOW. As for highest of any commercial airliner, even the lightest 757 is still well short of the Concorde's 0.3725 power/weight ratio. :-) -- Karl Swartz |INet kls@ditka.chicago.com 1-415/854-3409 |UUCP uunet!decwrl!ditka!kls |Snail 2144 Sand Hill Rd., Menlo Park CA 94025, USA Send sci.aeronautics.airliners submissions to airliners@chicago.com From kls Wed Dec 2 13:18:55 1992 Newsgroups: sci.aeronautics.airliners Path: news From: rdd@cactus.org (Robert Dorsett) Subject: Re: Flight envelope protections X-Submission-Date: Wed, 2 Dec 92 05:37:33 CST References: Message-ID: Approved: kls@ohare.Chicago.COM Sender: kls@ohare.Chicago.COM X-Submission-Message-Id: <9212021137.AA12970@cactus.org> Date: 02 Dec 92 13:18:55 PST palmer@icat.larc.nasa.gov (Michael T. Palmer) wrote: > This has some serious consequences. For example, in the China Airlines > B-747 incident 300 nm northwest of San Francisco in 1985 (NTSB/AAR-86-03), > the crew was forced to overstress (and structurally damage) ^^^^^^ That might be overstating the case a bit. :-) The NTSB report suggests they didn't have a clue how to recover from the spiral, once they entered it, lacking military aerobatic training and being completely disoriented. I don't believe the report distinguishes the tailplane's damage as being incidental or intentional. > the horizontal > tail surfaces to recover from a roll and near-vertical dive following an > automatic disconnect of the autopilot when it could no longer compensate > for an asymmetric thrust condition. At the time of disconnect, full > rudder was engaged to one side and the crew was unaware of this. The > crew recovered control with about 10,000 ft of altitude left (from an > original high-altitude cruise). It is very likely that if the aircraft > had prevented the crew from initiating control commands that would lead > to aircraft damage, the aircraft (and passengers) would have been lost. Your point's well taken, and the risks are certainly worth considering. But allow me to play devil's advocate, for a minute, without diluting your argu- ment, and suggest that the EFCS would have prevented an A3[2-4]0 from getting into the unusual attitude to begin with. The protections are both aerodynamic and input-filtering (and configuration-evaluating, and...). In the China Air incident, the flip-over was caused by a "dumb" autopilot/autothrottle design configuration oversight, following an engine abnormality. If a similar event had occurred on an A3[2-4]0, the EFCS would probably have limited both the authority of the FMS to put the airplane into the steep bank, *and* would have provided maximum corrective action, using opposing controls, to keep the airplane in the prescribed operating envelope. But let's suppose some other kind of fault flips the airplane over: rotor, wake turbulence, transient EFCS bug (REALLY unlikely). I would have less confidence in the system than in a 747, but there are saving graces in the system design. During the flip-over itself, the system would have reverted to Alternate Law when one of these conditions were met: Pitch > 50 degrees nosePup or < 30 degrees nosePdown. Bank > 125 degrees. AOA > 30 degrees or < P10 degrees. Speed > 460 knots or < 60 knots. Mach > 0.91 or < 0.1. There would not have been protections or auto-trim; there would have been full-authority direct law in roll, without yaw-damper services. It is not clear whether "device-saving" protections would have been in place (likely, no doubt, considering the extensive use of composites in the tail surfaces). (don't forget: you have to remember all this when the shiny side's the wrong way up :-)) I also wonder how well the four accelerometers the EFCS uses would have held up to all this. No matter: they're durable. A320 simulators use pretty much the same EFCS code as the actual airplane. Since programming errors often show up in 90-degree increments (tan 90!), I suspect it would be interesting to turn off the motion system and take the thing up for a spin, so to speak... :-) More grist for the mill: In an unnamed regulatory agency's commentary on a paper that Pete Mellor and I are cooking up, there was a note that in the case of even a "run-away" surface (actuator OR software malfunction), the remaining devices/ governing software would function to provide a "virtual" effect, providing handling qualities that would mask the abnormality. I was aware that a "make-up" feature existed, but the precise wording raises the question of how much loading, exactly, the run-away surface might introduce, or how violent an oscillation the system could be trying to cover up. I find this *quite* disquieting, especially since, in the FAA's Special Conditions for the A320's certification in the United States, the point was clearly made that the FAA does *not* believe the pilots have a right to be warned of failures of this sort: This is from the Federal Register 54:17, January 27, 1989, pages 3989 and 3996: P. 3996: paragraph 2(a)2(i), the item under discussion: active controls, basic criteria, with the system in failure conditions: "(i) Warnings must be provided to annunciate the existence of failure conditions which affect the structural capability of the airplane and for which the associated reduction in airworthiness can be minimized by suitable flight limitations. Failure conditions which affect the structural capability of the airplane and for which there is no suitable compensating flight limitation need not be annunciated to the flightcrew, but must be detected before the next flight." P. 3989, the oh-so-enlightening, explanatory commentary: "The second commenter believes that the flightcrew must be aware of any failure conditions which affect the structural capability of the airplane, whether or not a compensating procedure exists. The FAA does not concur with this comment. It is not necessary for the flight crew to be aware of a failure in the active control system during the flight on which the failure occurs if there is no available corrective action; however, the airplane should not be exposed to the failure condition for an extended period of time. The flightcrew must therefore be alerted to the failure condition prior to the next flight." This is from the FAA, the agency in charge of establishing airworthiness and certification practices in the United States! In reality, the A320 likely *does* provide enough feedback: but the FAA, apparently unnecessarily, has certainly opened the door for the practice to be introduced in subsequent types. > Unfortunately, it appears that engine manufacturers may be heading down > the same path as Airbus with respect to their electronic engine controllers. Beyond "dumb" smartness, Pete Mellor has uncovered reason to believe the engine controllers do not use dissimilar software. On the A320, there are two FADECS per engine: a common-cause-of-failure logic fault could con- ceivably take out both controllers. It's not clear whether this could happen in tandem, based on environmental conditions, or serially, which could intro- duce a short timing delay in which the input parameters could be "corrected." > If nothing else, I hope I have brought up some topics that deserve > discussion among readers of this newsgroup. After all, aren't we the > ones in positions to influence our industry (all in our own way, of > course)? Especially in software, of particular relevance to the net. A lot (if not most) of the people writing this code--4M on the A320, 10M+ on the A330 and A340--are *not* aero engineers: just programmers, ostensibly with CS backgrounds (a more frightening thought I can't imagine! :-)), performing under strictly governed, structured, controlled environments: to specif- ication. Airbus even mentioned the "CS" types it brought in from "outside" to buttress a comment on its quality-control practices, in an article, as if to make the point that mere engineers weren't writing this stuff: the "pros" are doing it. :-) Yeah, we know what we're doing, SURE... :-) Computers on the brain... Alphabet soup: AOA Angle of Attack CS Computer Science EFCS Electronic Flight Control System FADEC Full-Authority Digital Engine Control FMS Flight Management System M Megabyte NTSB National Transportation Safety Board --- Robert Dorsett rdd@cactus.org ...cs.utexas.edu!cactus.org!rdd From kls Wed Dec 2 13:18:56 1992 Newsgroups: sci.aeronautics.airliners Path: news From: ulfwbadg@w228zrz.zrz.tu-berlin.de (Ulf Weber) Subject: abandoning the APU on four engine / long range aircraft X-Submission-Date: Wed, 2 Dec 92 17:15:35 +0100 Message-ID: Approved: kls@ohare.Chicago.COM Sender: kls@ohare.Chicago.COM X-Submission-Message-Id: <9212021615.AA05495@w228zrz.zrz.tu-berlin.de> Date: 02 Dec 92 13:18:56 PST Schoenen Guten Tag, who can help me with my cost/benefit analysis about "abandoning the APU on four engine / long range aircraft"? I need info about the FAA requirements (EROPS ?), the cost of installing GPUs (Ground Power Units, 400 Hz), ASUs (Air Starter Units, if possible mobile (mounted on pulling tractors?) in order to delay engine start as long as possible) and last but not least the technical feasibility of making the engines do what the APU has traditionally been doing. So. what do you think what an engine for a "zero-APU-747-400" would look like? 'Hope to hear from you, Ciao Ulf. From kls Wed Dec 2 13:18:57 1992 Newsgroups: sci.aeronautics.airliners Path: news From: Christopher Davis Subject: Re: The Sporty Game -- Boeing 757 X-Submission-Date: Wed, 2 Dec 1992 13:52:26 -0500 References: Message-ID: Approved: kls@ohare.Chicago.COM Sender: kls@ohare.Chicago.COM X-Submission-Message-Id: <199212021852.AA09487@loiosh.eff.org> Date: 02 Dec 92 13:18:57 PST RJ> == Robert Jacobson RJ> As a frequent air traveler, I find the 757 to be positively the RJ> most uncomfortable aircraft now flying. One can begin with the RJ> ubiquitous TV monitors hanging from the ceilings every few rows, RJ> which cannot be dimmed or turned off even on a red-eye, and RJ> progress to the remarkable number of seats that can be squeezed RJ> into row upon row of stifled passengers. It may be a technical RJ> feat, but I know instruct my travel agent to pass on any flight RJ> requiring me to take a 757. Yeck. What airlines are you flying? The 757's monitors *are* turned off on most of the flights I've seen (except for the coast-to-coast nonstops, where they're on for the movie and/or the "video magazine"). The seat pitch seems to be better than the predecessor 727-200s usually have (especially if you get the right seats; the NW 757s have four large doors per side instead of 3 large doors and two overwing exits, making for a number of seats with immense legroom). The 757 isn't a widebody, but (to me anyway) it has many of the best features of widebodies (it only lacks the second aisle for use in getting around meal carts :). (What aircraft *are* you flying on, if you're not on 757s?) From kls Wed Dec 2 13:18:58 1992 Newsgroups: sci.aeronautics.airliners Path: news From: ncole@nyx.cs.du.edu (Noah Cole) Subject: Re: 747SP X-Submission-Date: Wed, 2 Dec 92 19:48:13 GMT References: Message-ID: Approved: kls@ohare.Chicago.COM Organization: Macalester College, St. Paul Minnesota USA Sender: kls@ohare.Chicago.COM X-Submission-Message-Id: <1992Dec2.194813.2984@mnemosyne.cs.du.edu> Date: 02 Dec 92 13:18:58 PST kls@ohare.Chicago.COM (Karl Swartz) writes: >In article ncole@nyx.cs.du.edu (Noah Cole) writes: >>How many airlines use 747SP's today? I have a poster from Popular Mechanics >>that was around the arrival of the 747-400 with a drawing of a United >>747SP and it said that the 747SP set a record flying from Payne Field, >>Washington to South Africa. Was that SAA? Who flies the 747SP Today >>and on what routes? (Info on ownership of 747SP deleted- it was very interesting though. Thanks) >Oh yes, it was a South African 747SP delivery that held a record for >longest flight (time or distance or both) for a jetliner or some such. >That may still stand but I wouldn't be too surprised if a 747-400 had >subsequently established a new record. I thin k that the record was set by a Qantas 747-400 from LHR to SYD in the summer of 1989 with the poassengers being the British Cricket Team and the only article in the cargo was a cricket ball. -Noah Cole -- Noah Cole "Outside is America, NCOLE@MACALSTR.EDU Macalester College and also the car park" ncole@nyx.cs.du.edu St. Paul, MN 55105 - Bono, 27 December 1989 cncole@coos.dartmouth.edu 612-696-7388 Dublin aj909@cleveland.freenet.edu From kls Wed Dec 2 13:18:59 1992 Newsgroups: sci.aeronautics.airliners Path: news From: hfunk@src.honeywell.com (Harry Funk) Subject: Re: maintenance (was Re: pair (sorry, couldn't resist the pun)) X-Submission-Date: Wed, 2 Dec 1992 20:02:42 GMT References: Message-ID: Approved: kls@ohare.Chicago.COM Reply-To: hfunk@src.honeywell.com (Harry Funk) Organization: Honeywell Systems & Research Center Sender: kls@ohare.Chicago.COM X-Submission-Message-Id: <1992Dec2.200242.26692@src.honeywell.com> Date: 02 Dec 92 13:18:59 PST In article gary@maestro.mitre.org (Gary Bisaga) writes: >In article , weiss@wright.SEAS.UCLA.EDU (Michael Weiss) writes: >|> >|> primarily). He said that DC-10s are notorious for repairs being all-day >|> operations, whereas Boeing's 737-300 and -400, 747-300 and -400, 757, and 767 >|> have self-diagnostic systems that go so far as to direct the location of the >|> repair instructions down to the page, turning the repairs into a half-day >Nor would I - but you're comparing apples to oranges. The other aircraft you >mention probably have much more extensive electronic maintenance aids since >most have much more extensive avionics in general. It wouldn't surprise me >if there was also more electronic diagnosis. >The MD-11, of course, is just as highly outfitted with electronics as any >of those others - and the FMC design is newer as well, if I'm not mistaken - >so a comparison with the MD-11 would almost certainly be different. I agree. There are several factors involved here: how often does stuff break or require scheduled maintenance, how well is the fault isolated by on-board or off-board maintenance aiding systems, how well does the documentation aid you in further isolating and subsequently rectifying the fault, how difficult does the design of the aircraft make it to perform the directed operations. The MD-11 has a Centralized Fault Display System (CFDS), which serves as a gateway to the BIT information provided by other avionics. ARINC report 604 describes the characteristics of a CFDS system. The 747-400 (but not older versions of the 747, nor the 757/767) has a Central Maintenance Computer, which takes the individual reports provided by the avionics, merges them to form a consistent picture, which it provides to the maintenance tech by means of a fault message. The message is associated with a entry point into the (paper) maintenance documentation fault tree. Airbus has a similar system. B777 will have a similar system. These last three are all (I think) ARINC Report 624 (On-board Maintenance System) systems. The more advanced systems, such as those found on the B747-400, A320/340 and B777, are quite difficult to build, since a change in the design of any reporting subsystem may affect the design/operation of the CMC. During flight test, when things change rapidly, the CMC changes as the sum of all the subsystem changes (best case - worst case is that the changes interact.) "I am not, have never been, and hope never to be, a spokesperson for Honeywell." H. -- Harry A. Funk Principal Research Scientist Voice: (612)-782-7396 Honeywell Systems and Research Center FAX: (612)-782-7438 3660 Technology Dr. MS:MN65-2500 Inet: hfunk@src.honeywell.com Minneapolis, MN 55418 Bang: !srcsip!funk From kls Wed Dec 2 13:18:59 1992 Newsgroups: sci.aeronautics.airliners,rec.travel.air Path: news From: philip@rainbow.mentorg.com (Philip Peake) Subject: Re: Airbus safety X-Submission-Date: Wed, 02 Dec 1992 20:08:19 GMT References: <1992Nov26.000453.4729@cactus.org> <1992Dec01.025604.17493@news.mentorg.com> Message-ID: Approved: kls@ohare.Chicago.COM Organization: Mentor Graphics Sender: kls@ohare.Chicago.COM X-Submission-Message-Id: <1992Dec02.200819.19590@news.mentorg.com> Date: 02 Dec 92 13:18:59 PST In article , rdd@cactus.org (Robert Dorsett) writes: |> We can automate easily quantifiable issues: simple tasks. Judgement and |> airmanship has thus far evaded us, on all levels. Until we get a grip on |> it, talk of fully autonomous aircraft or ground control is nothing more |> than science fiction. Robert, in general, you write extremely well argued and researched cases, but occasionally you spoil the whole effect by little "throw away" comments such as the above - history, even modern history is littered with comments from people writing off things as "science fiction", "can't be done", "will never replace the current ...." etc who have had to eat their words shortly after. Anyway, getting back to the A320 ... I suppose that I should now admit to not actually being a strong a supporter of this machine as might have been assumed from past postings (I have flown on them too many times :-) My concernes are not based so much on the ergonomic design, so much as the engineering of the computerised systems, and the numerous "cover-ups", which are apparently inspired by the French government - if you read French, you might be interested in a series of articles in "Science et Vie", which is a sort of "Scientific American". There, the concerns expressed are simply that: - There have been many documented occurences of sudden altitiude changes which were uncorrectable by the pilot. - Airbus Industrie REFUSES to let independent experts audit their software, that say that the control system can only be examined as a "black box", which either performs correctly, or it doesn't - anyone at all familiar with software engineering will recognise this as being close to garbage. - Refusal by AI to acknowlege that there may be problems at all. - Attempts by members of the French government to abort independent investigations (including that of Science et Vie). This is getting away from our discussion about pilot contributions to "incidents", but if you want to knock the A320, there are much better grounds for doing so than ergonomics - without the more serious design problems, there would probably have been many fewer "accidents", and hence less reason to blame the ergonomics. Besides "cosmetic" issues like tactile feedback, and some layout issues, the 767 is pretty close to an A320 - as you have said (I think - sorry if I misquote you) the 767 is just more conventional in cockpit design - its a pity its automatic landing system can be as good as the best pilot on a good day, and a rough as the worst on a bad day ... usually more towards the latter ... Philip From kls Wed Dec 2 13:19:00 1992 Newsgroups: sci.aeronautics.airliners Path: news From: Stephen L Nicoud Subject: AVIATION DAILY - 12/1/92 - International Lease Finance Corp X-Submission-Date: Wed, 2 Dec 92 12:13:26 PST Message-ID: Approved: kls@ohare.Chicago.COM Sender: kls@ohare.Chicago.COM X-Submission-Message-Id: <9212022013.AA05689@moclips.boeing.com> Date: 02 Dec 92 13:19:00 PST Spotted on a Boeing Electronic Bulletin Board: 12/2/92 3. AVIATION DAILY - International Lease Finance Corp. President Steven Udvar-Hazy yesterday confirmed the company will place, probably by mid-December, orders and options for at least 80 aircraft and possibly as many as 100. He said he hoped the orders, which will involve most big engine manufacturers, "will be a spark plug to give confidence back to the airline industry." The orders also should provide a big boost to Boeing and Airbus, but not Douglas. Udvar-Hazy said Douglas "made vanilla-type proposals, whereas Boeing and Airbus have been more appreciative of ILFC steadiness in keeping their production lines going." -- Stephen L Nicoud bcstec!bcsaic!stephen Boeing Computer Services Research and Technology Bellevue, Washington USA "I ask unanimous consent to revise and extend my remarks." From kls Thu Dec 3 00:40:06 1992 Newsgroups: sci.aeronautics.airliners Path: news From: ashabana@agsm.ucla.edu (Ahmed Shabana) Subject: Re: Boeing 747-300 X-Submission-Date: 2 Dec 92 18:35:23 GMT References: Message-ID: Approved: kls@ohare.Chicago.COM Sender: kls@ohare.Chicago.COM X-Submission-Message-Id: Date: 03 Dec 92 00:40:06 PST drinkard@bcstec.ca.boeing.com (Terrell D. Drinkard) writes: >In article kls@ohare.chicago.com (Karl Swartz) writes: >>In article drinkard@bcstec.ca.boeing.com (Terrell D. Drinkard) writes: >>>In article jerry@telecom.ksu.edu (Jerry And >> >>>>I've heard Boeing, McDonnell-Douglas and Airbus all have plans >>>>in the works for Really Big Planes in the 600-700 passenger, >>>>7500-8000 mile range. >> >>>The Boeing and the Airbus offerings in this market seem to both hover >>>around 600 seats and 7,000 mile range. Takeoff weights in the million >>>pound plus range. The anticipated market, as described by John Hayhurst, >>>Director of New Large Airplane Division, is only a couple hundred airplanes >>>TOTAL. From my knothole, it looks like a prestige fight. >> >>That's an *awful* lot of cash to dump down a hole simply for bragging >>rights. Terry ( drinkard@bcstec.boeing.com) writes: >It isn't just bragging rights. Prestige has a market value. There are >several airlines who are not out to make a profit. The national airlines >of some oil rich countries for example, are not expected to make buck, >rather to 'carry the flag'. Thai, as another example bought 747s some >years ago largely because of prestige. Therefore, you have some airlines >who fly the airplane in the world to make a >political statement. While I agree with you that some airlines buy planes for prestige ( I think the cake goes to Iran Air during the Shah days when they had several Concordes on order), Thai is a world class airline that is profit oriented. They have a very close relationship with KLM and are one of the most successfull airlines in the fast growing Asian market. Moreover they have at least 9 747's in their fleet ( 6 -200, 3 -400) and an order for 5 more -400. Such commitement to an aircraft type stems from simple economics and not prestige. Ahmed -- _____________________________________________________________________ Ahmed A. Shabana |" Bill, now you can get rid of THAT watch" Anderson Graduate School of Mgmt.| Omega watches commercial aired on U.C.L.A. | Election day referring to Clinton's Timex From kls Thu Dec 3 00:40:08 1992 Newsgroups: sci.aeronautics.airliners Path: news From: davidm@questor.rational.com (David Moore) Subject: Re: 747SP X-Submission-Date: Wed, 2 Dec 1992 21:18:56 GMT References: Message-ID: Approved: kls@ohare.Chicago.COM Organization: Rational Sender: kls@ohare.Chicago.COM X-Submission-Message-Id: Date: 03 Dec 92 00:40:08 PST kls@ohare.Chicago.COM (Karl Swartz) writes: >Oh yes, it was a South African 747SP delivery that held a record for >longest flight (time or distance or both) for a jetliner or some such. >That may still stand but I wouldn't be too surprised if a 747-400 had >subsequently established a new record. QANTAS was crowing about their non-stop 747-400 delivery flight from London to Sydney when I flew with them a couple of years back. I think they were claiming records, but I do not remember for sure. Certainly, it is a long flight. To do it, they had to get a specially formulated (higher energy/kilo) fuel and the crew's luggage flew separately. Naturally, the flight magazine was silent on the question, but I imagine the crew also flew in their under-wear to further reduce weight. I have no idea why they were delivering via London. From kls Thu Dec 3 00:40:09 1992 Newsgroups: sci.aeronautics.airliners Path: news From: yarvin@CS.YALE.EDU (Norman Yarvin) Subject: Re: GE aerospace X-Submission-Date: 2 Dec 1992 17:24:15 -0500 References: Message-ID: Approved: kls@ohare.Chicago.COM Organization: Yale Computer Science Department Sender: kls@ohare.Chicago.COM X-Submission-Message-Id: <1fjd2fINN46n@CATHY.NA.CS.YALE.EDU> Date: 03 Dec 92 00:40:09 PST cid@athena.mit.edu (Derek H Cedillo) writes: >I hate to see GE lose a part of itself, but it was a wise move. A relative of mine works in a satellite manufacturing plant in New Jersey which is part of the deal. (He refers to that plant, and I don't know how much else, as "Astro".) It was bought by GE from RCA two or three years ago. He says that when GE took charge, they made many changes for the worse. For instance they brought in a computerized procurement system which was inappropriate for the satellite business -- it would have been better suited to a light bulb factory. Since Martin Marietta has much more interest in satellites than GE did, he has hopes that the new transition will not be as bad. -- Norman Yarvin yarvin@cs.yale.edu From kls Thu Dec 3 00:40:11 1992 Newsgroups: sci.aeronautics.airliners,rec.travel.air Path: news From: cid@athena.mit.edu (Derek H Cedillo) Subject: GE Ft. Wayne Plant (was: Re: GE Aerospace) X-Submission-Date: Thu, 3 Dec 1992 00:22:03 GMT References: Message-ID: Approved: kls@ohare.Chicago.COM Followup-To: rec.travel.air Organization: Massachusetts Institute of Technology Sender: kls@ohare.Chicago.COM X-Submission-Message-Id: <1992Dec3.002203.1831@athena.mit.edu> Date: 03 Dec 92 00:40:11 PST [all kinds of talk about what the Ft. Wayne Plant was] I'm sorry, but for the life of me I cannot find any info about Ft. Wayne being an AE plant. With my other GE stuff still on loan, I cant prove that it was Aerospace. But, As of 10/91 it was not listed as an AE plant. (this info is a listing of AE plants accross the country) the list includes all AE plants inluding the tiny satalite plants that make mini parts and the test center at Edwards AFB here is a listing: Lynn, MA (major production and engineering) Hooksett, NH Rutland, VA (both sat. parts prodution plants) Wilmington, NC Peebles OH Evendale, OH (Major production and engineering) Madisonville, KY Strother, KS Alburquerque, NM Ontario, CA (california, not canada) Edwards AFB, Mojave, CA and Seattle, WA and international in Singapore and Bromont, Canada Is this innacurate? It was produced by GE itself, as a recuiting brochure. So I dont think there would be editing errors. Some of the info other people have been laying out is from the mid 80's perhaps did it change hands? (again this is from 10/91) Thanks alot. (sorry if I am beating a dead horse, but this is really bugging me) Derek ------------------------------------------------------------------- "He lived a life of going-to-do, and died with nothing done" -J. Albery In other words---JUST DO IT! From kls Thu Dec 3 00:40:13 1992 Newsgroups: sci.aeronautics.airliners,rec.travel.air Path: news From: cid@athena.mit.edu (Derek H Cedillo) Subject: Re: Airbus safety X-Submission-Date: Thu, 3 Dec 1992 01:15:24 GMT References: Message-ID: Approved: kls@ohare.Chicago.COM Organization: Massachusetts Institute of Technology Sender: kls@ohare.Chicago.COM X-Submission-Message-Id: <1992Dec3.011524.2917@athena.mit.edu> Date: 03 Dec 92 00:40:13 PST >Besides "cosmetic" issues like tactile feedback, and some layout issues, >the 767 is pretty close to an A320 - as you have said (I think - >sorry if I misquote you) Im sorry, but I dont think cockpit layout and feedback are "cosmetic". These are pretty important issues. Actually, these are the main gripes about teh A320. If the cockpit is non-convential, its not a cosmetic issue, but rather a problematic issue. And think about steering your car without knowing how much of a turn will take you until your there. A prime example of lack of feedback is a digital stereo. Think of how many times you over shoot the radio station your looking for because you get no feel of how fast the thing is going through stations (other than visually). Or perhaps if you put the radio knob where the lights knob was and vice versa. As these two things are not life or death threatening, imagine if they were something for steering and braking. Oh well, maybe some dumb analogies but i hope you get the picture. Feedback and layout are not "cosmetic". Derek ------------------------------------------------------------------- "He lived a life of going-to-do, and died with nothing done" -J. Albery In other words---JUST DO IT! From kls Thu Dec 3 00:40:14 1992 Newsgroups: sci.aeronautics.airliners Path: news From: drinkard@bcstec.ca.boeing.com (Terrell D. Drinkard) Subject: Re: The Sporty Game -- Boeing 757 X-Submission-Date: Thu, 3 Dec 1992 02:27:00 GMT References: Message-ID: Approved: kls@ohare.Chicago.COM Organization: Boeing Sender: kls@ohare.Chicago.COM X-Submission-Message-Id: Date: 03 Dec 92 00:40:14 PST In article cyberoid@stein.u.washington.edu (Robert Jacobson) writes: > >As a frequent air traveler, I find the 757 to be positively the most >uncomfortable aircraft now flying. One can begin with the ubiquitous >TV monitors hanging from the ceilings every few rows, which cannot be >dimmed or turned off even on a red-eye, and progress to the remarkable >number of seats that can be squeezed into row upon row of stifled >passengers. It may be a technical feat, but I know instruct my travel >agent to pass on any flight requiring me to take a 757. Yeck. I hasten to point out that it is no fault of the airplane, or of the manufacturer for that matter, that you have been overcrowded. The interior of the airplane is determined by the operator. All the interiors. Interiors, sometimes called 'payloads', fall in the category of 'BFE', or Buyer Furnished Equipment. That means that when Delta stuffs 38 rows (or whatever the precise number happens to be) into a 757, you get a 30" seat pitch (or a 29" or a 28"!). Don't blame the manufacturer or the airplane. As another illustration, take a look at American's 727-200s. Originally designed as a 150+ seat transport, they use it as a 129 seater. Nice 34" seat pitch over almost the entire airplane (the first class has it even better). But the American ticket typically costs more. You pays your money and you takes your pick. -- Terry drinkard@bcstec.boeing.com "Anyone who thinks they can hold the company responsible for what I say has more lawyers than sense." From kls Thu Dec 3 00:40:16 1992 Newsgroups: sci.aeronautics.airliners Path: news From: weiss@turing.SEAS.UCLA.EDU (Michael Weiss) Subject: Re: hydraulic problems with DC-10's?? X-Submission-Date: 3 Dec 92 04:50:46 GMT References: Message-ID: Approved: kls@ohare.Chicago.COM Organization: SEASnet, University of California, Los Angeles Sender: kls@ohare.Chicago.COM X-Submission-Message-Id: <8793@lee.SEAS.UCLA.EDU> Date: 03 Dec 92 00:40:16 PST In article drinkard@bcstec.ca.boeing.com (Terrell D. Drinkard) writes: >In article weiss@curtiss.SEAS.UCLA.EDU (Michael Weiss) writes: >>I have a hard time believing that an intact hydraulic system would have >>prevented AA191 from crashing. Let's face it, a wing-mounted engine falling >>off produces such a rediculous unbalance that even full aileron wouldn't be >>able to counter it. >Not true. An engine departing the airplane is a planned for event, in >terms of stability and control. An aileron would have no problem >countering just the imbalance of thrust (and it would actually be mostly >rudder), in fact, without the added drag of a windmilling engine, the >problem is a bit simplified. After the third post with this answer, I figure it's time to clarify my statement. I am referring to the unbalance of WEIGHT, not THRUST. Nonetheless I suppose we should go on... >Flight AA 191 lost the slats on the left hand >wing (if memory serves) because of Douglas' failure to include mechanical >lockouts on the slat actuators. They were not required to certify the >airplane. Which doesn't disprove my theory. As it is, though, the loss of the slats (which, according to all my aero classes, only lowers the stall speed but does NOT increase the coefficient of lift!) was enough to stall the wing, more than "countering" the loss of weight on the wing. -- \ | | | | | | | | | | | | | | | | | | | | | | | | | | | | | | | | | | | | | | / - Michael weiss@seas.ucla.edu | School of Engineering & Applied Science - - Weiss izzydp5@oac.ucla.edu | University of California, Los Angeles - / | | | | | | | | | | | | | | | | | | | | | | | | | | | | | | | | | | | | | | \ From kls Thu Dec 3 00:40:17 1992 Newsgroups: sci.aeronautics.airliners Path: news From: geoff@peck.com (Geoff Peck) Subject: Re: Boeing Book X-Submission-Date: Thu, 3 Dec 1992 06:35:32 GMT References: Message-ID: Approved: kls@ohare.Chicago.COM Organization: Geoffrey G. Peck, Consultant, San Jose CA Sender: kls@ohare.Chicago.COM X-Submission-Message-Id: <1992Dec3.063532.13163@peck.com> Date: 03 Dec 92 00:40:17 PST In article Christopher Davis writes: > This might be the Robert Serling _Legend & Legacy_, which I hope to find > the time to do a book review on, eventually. *Very* good book. OK, I'll bite. Here's a review of "Legend and Legacy" which I wrote for rec.aviation in mid-September: From: geoff@peck.com (Geoff Peck) Subject: Book Review: Legend and Legacy Message-ID: <1992Sep20.180822.7903@peck.com> Date: Sun, 20 Sep 1992 18:08:22 GMT I just finished the book "Legend and Legacy: The Story of Boeing and Its People" by Robert J. Serling (St. Martins Press, New York, 1992). It's an incredibly well-written corporate biography of Boeing at its seventy- fifth anniversary, and is superb reading for Boeing fans, aviation fans, and those interested in corporate management, philosophy, and development. Besides, the book is chock-full of reliably-documented anecdotes, incredibly funny deeds, and priceless one-line quotes from a huge variety of Boeing folks. One example, which seems to come up on the net with some regularity, is the question of the Famous 707 Barrel Roll. I've included as an attachment below a slightly generous excerpt from the book which describes this famous incident to whet your appetite. The book, of course, details many of the projects that the immensely complex and diverse organization has worked on -- from commercial aircraft to cruise missiles to the lunar roving module to military aircraft to bedroom furniture (no, I'm not kidding) to the never- produced passenger SST to helicopters and light rail vehicles to completely overhauling NASA's project management structure. The variety of projects, and the number of innovative experiments, will truly boggle one's mind. I was most impressed by the personal and corporate integrity of the people who are Boeing -- this book really almost justifies the well-worn phrase "if it doesn't say Boeing, I'm not going." A minor word of caution on this book -- once you start reading it, you'll find it incredibly hard to put down, much like a Robert Ludlum novel. You may find yourself staying up all night trying to finish its generous 460-plus pages in one sitting. You're also likely to exclaim, after reading a particularly favorite chapter (I only had about 10 or 12 such chapters) "goodness -- that was much too short -- there should be a whole _book_ on this subject!" The book, as one might expect, does see Boeing through the rose-colored glasses of memory. It's well-researched -- Serling spent quite a bit of time interviewing Boeing employees and searching company documents. But one doesn't read this kind of book expecting a critical exposee of mis-deeds ... nor would one really expect much of that in a corporate biography of a company as principled as Boeing. Now, what might be the ideal reading environment for Legend and Legacy? Well, I read most of it ... while flying inside Boeing jetliners. (And I'm still sitting inside a 737, bouncing around in light, occasionally moderate, turbulence, as I type this article!) Although the order of flying the Boeings didn't quite match their presentation in the book, on this trip I flew in 757's, a 737, a 727, and even a 707. Well, sort of. The 707 was actually UAL flight 707 from Newark to Denver, operating today as a 757. :-) Ah, well. I still do very fondly remember the 707 (and the 320B, and the 727-100, the 727-200, and the 737-100, and the 737-200, and ...). Alas, no 747 or 767 on this trip, but I certainly have enough passenger-hours in those types. If it doesn't say Boeing, _I'm_ not going! For those amused by historical significance, my 727 flight ended by landing on runway 22 at New York's LaGuardia airport. I can't recommend this book highly enough! It should be available at most general booksellers (I bought mine at a B. Dalton in a mall) in hard cover at $24.95. Geoff --- [Excerpt from _Legend_and_Legacy regarding the 707 roll] [This occurred] when the same Aircraft Industries Association group and representatives of the International Air Transport Association held joint meetings in Seattle. The Gold Cup hydroplane races were being held at the same time, and Allen [Boeing's CEO] invited the industry dignitaries to watch the events from three yachts Boeing had chartered for the occasion. As a special treat he also told Tex Johnston to stage a flyby in the Dash-80 on the day of the races so everyone could see the airplane of the future. Tex never did anything halfway. PR director Carl Cleveland had told him to come over Lake Washington, where the boat races were being held, at a prearranged time. When that moment arrived, the Dash-80 was in the middle of a routine test flight over the Olympic Peninsula and Johnston said to copilot Jim Gannett, "I'm gonna roll this bird over the Gold Cup course." "They're liable to fire you," Gannett warned. "Maybe, but I don't think so." The Dash-80 was doing 450mph when Tex brought it over Lake Washington at only 300 feet, put the jet into a 35-degree climb and proceeded to do a complete 360-degree barrel roll. Then he reversed course, came back over the lake and repeated the maneuver -- again in full view of 300,000 awed spectators, some vastly impressed industry officials, and a very unhappy William McPherson Allen. After the second roll, Allen turned to Larry Bell of Bell Aircraft, one of his guests. "Larry, give me about the of those heart pills you've been taking. I need them worse than you do." Bell laughed. "Bill, I think he just sold your airplane." Allen ignored him and said to Carl Cleveland, "I don't think we should have anything in the newspapers about this." Poor Carl pointed out it was going to be a little difficult to keep something out of the papers that 300,000 people had just witnessed. Allen didn't pursue that matter further, but at eight o'clock Monday morning, Johnston appeared in his office where not only Allen but Wells, Beall, Schairer, and Martin were waiting. Allen's first question was directed not at Tex but at Schairer. "Did you tell him to do it?" Schairer never had a chance to reply, for Johnson immediately said, "No, he didn't." Allen turned to Tex. "What made you do it?" "I was selling the airplane," the test pilot answered. He went on to explain that the Dash-80 was never in any danger, that an airplane doesn't recognize altitude if the forces imposed on its structure do not exceed one g (the force of gravity). Allen, still openly upset, was silent for a moment. "You know that," he finally said, "and now we know that. But the public doesn't know it. Don't ever do it again." Tex promised to behave. What he didn't tell Allen, however, was that he had rolled the prototype near Mt. Rainier on a previous flight and his fellow test pilots agreed with him that the maneuver, while spectacular, never endangered the airplane because the roll was slow and carefully controlled. They heard later of one incident in which an Air Force pilot actually did a _full_loop_ with a KC-135, the 707's military designation, and got away with it, although both outboard pods were torn off. Maynard Pennell didn't get mad at Tex, either. "It was an unnecessary sales job," Pennell commented, "but it really was a spectacular way to demonstrate the airplane. With a skilled pilot, the risk was minimal." Johnston believed Allen had forgiven him. In fact, he was invited to Allen's home for dinner the same day of the on-the-carpet session, and the first person to greet him was Eastern's crusty Eddie Rickenbacker. He grabbed Tex's Stetson hat, pulled it down over the pilot's ears, and chortled, "You slow-rolling son of a bitch -- why didn't you let me know you were gonna pull that? I would have been riding the jump seat!" Allen overheard this and smiled when Rickenbacker added, "Damn, Bill, _that's_ the way to get attention with a new airplane." Mef Allen said the roll was the talk of that evening, most of it expressions of approval and admiration that a huge commercial jet could be rolled safely. But while Tex left the house convinced he had been exonerated, there is considerable evidence that it took a long time before Allen really forgave him. Many months went by and Johnston was attending the annual management lawn party at the Allen home. He poked Allen in the chest with a finger and asked, "Bill, are you finally willing to admit that slow-rolling the Dash-80 was the greatest thing that ever happened to that program?" Allen gave him a look that would have frozen boiling water. "No," he said icily. In a 1977 speech to the Washington State Historical Society, Allen said he thought at first the rolls might have been unintentional and that he summoned Johnston to his office merely to ask if something had gone wrong with the controls. He didn't get angry, he insisted, until the test pilot admitted he rolled the $16 million prototype deliberately. "It has taken nearly twenty-two years for me to reach the point where I can discuss the event with a modicum of humor," he told the audience. ----- There are at least two other interesting stories about rolling an airplane in the book -- but for those, you get to go read the book! Okay, okay. One more short excerpt: ----- There was Mark Miller, for example, who was an absolute terror when it came to spelling. He actually got angry at anyone who would misspell a word. One day he was reviewing a Minuteman progress presentation being delivered on a chart by Bob Edelman of engineering. The word _relieve_ appeared on the chart, but it was spelled _releive_. "Dammit, Edelman," Miller scolded, "_i_ always goes before _e_." "You're absolutely right," Edelman agreed. He took out a grease pencil and at the bottom of the chart, he changed _Boeing_ to _Boieng_." ----- Go buy the book! G ----- End Included Message ----- From kls Thu Dec 3 03:18:15 1992 Newsgroups: sci.aeronautics.airliners Path: news From: jaap@cs.ruu.nl (Jaap Romers) Subject: Re: AVIATION DAILY - 12/1/92 - International Lease Finance Corp X-Submission-Date: Thu, 3 Dec 1992 10:47:47 GMT References: Message-ID: Approved: kls@ohare.Chicago.COM Organization: Utrecht University, Dept. of Computer Science Sender: kls@ohare.Chicago.COM X-Submission-Message-ID: <1992Dec3.104747.23121@cs.ruu.nl> Date: 03 Dec 92 03:18:15 PST In stephen@boeing.com (Stephen L Nicoud) writes: >Spotted on a Boeing Electronic Bulletin Board: > >[stuff deleted] > Is it connected to the Internet, can we telnet to that BBS ?? jaap From kls Thu Dec 3 05:01:33 1992 Newsgroups: sci.aeronautics.airliners Path: news From: rdd@rascal.ics.utexas.edu (Robert Dorsett) Subject: Night landings and the 727 in the 1960's X-Submission-Date: Thu, 3 Dec 92 06:14:38 CST Message-ID: Approved: kls@ohare.Chicago.COM Sender: kls@ohare.Chicago.COM X-Submission-Message-Id: <9212031214.AA21446@rascal.ics.utexas.edu> Date: 03 Dec 92 05:01:33 PST Those interested in the night-landing problem described in Dave Rogers' excerpt from _Legend & Legacy_ would probably find the following of interest; it describes the original research. The descriptions of the experimental apparatus, and the simulator test regime, are fascinating. Kraft, Conrad L. "A psychophysical contribution to air safety: simulator studies of visual illusions in night visual approaches." In _Psychology: from research to practice_, Herbert L. Pick, Jr., ed. (New York: Plenum Press), 1978. Pages 363-385. ISBN 0-306-31132-1. -- Robert Dorsett Internet: rdd@rascal.ics.utexas.edu UUCP: ...cs.utexas.edu!rascal.ics.utexas.edu!rdd From kls Fri Dec 4 22:30:29 1992 Newsgroups: sci.aeronautics.airliners Path: news From: David.B.Horvath@rave.larc.nasa.gov, CDP Subject: Re: GE aerospace X-Submission-Date: Tue, 1 Dec 1992 15:08:32 EST References: Message-ID: Approved: kls@ohare.Chicago.COM Organization: Hidden - I don't speak for them... Sender: kls@ohare.Chicago.COM X-Submission-Message-Id: <92336.150832MBADBH@rohvm1.rohmhaas.com> Date: 04 Dec 92 22:30:29 PST I was layed off by GE Aerospace, Astro-space division (satelites) last year and have kept up with the goings-on in the company. I've also grabbed information via the Dow Jones News Service: Martin Marietta Corporation is acquiring General Electric Company's Aerospace operations. These will be *merged* in with MM's current operations. MM plans on closing the transaction by March 1, 1993. Before the arrangement was announced, GE signed a pledge not to hire any (soon to be) former GE employees for a period of 2 years (the GE term is "the employees are Frozen in their jobs"). MM will probably close the GE Aerospace HQ in Valley Forge, PA (suburban Philadelphia) as the MM HQ is in Bethesda, MD. GE is getting $1B in convertable preferred stock, 2 seats on an expanded MM board, and about $2B in cash. MM is financing somewhere around $500M of the cash and issuing the new stock to GE. Secretary of Defense Dick Cheney was briefed and was supportive. The FTC has not passed on the transaction yet. GE Aerospace consists of the GE units and units acquired from RCA a few years ago. Jack Welch bought RCA for $600M (cash - no debt); what will he buy with $2B? GE has (essentially) no debt and the stock is trading rather high for a massive stock-repurchase to take place. The sale includes GE Aerospace, GE Government Services, Knoll Atomic Power Labs (Schy., NY) and GE MAO. - David Horvath (BTW: GE also sold GE Consulting Services to Keane, Inc recently. #$%^, that's 2 parts of GE that I used to work for that got sold). ========================================================================= -- Michael T. Palmer, M/S 152, NASA Langley Research Center, Hampton, VA 23681 Voice: 804-864-2044, FAX: 804-864-7793, Email: m.t.palmer@larc.nasa.gov PGP 2.0 Public Key now available -- Consider it an envelope for your e-mail From kls Fri Dec 4 22:30:31 1992 Newsgroups: sci.aeronautics.airliners,rec.travel.air Path: news From: nelson_p@apollo.hp.com (Peter Nelson) Subject: Re: Airbus safety X-Submission-Date: Thu, 3 Dec 1992 21:37:01 GMT References: <1992Dec01.025604.17493@news.mentorg.com> Message-ID: Approved: kls@ohare.Chicago.COM Organization: Hewlett-Packard Corporation, Chelmsford, MA Sender: kls@ohare.Chicago.COM X-Submission-Message-Id: Date: 04 Dec 92 22:30:31 PST In article rdd@cactus.org (Robert Dorsett) writes: >In article nelson_p@apollo.hp.com (Peter Nelson) writes: >> _New Scientist_ had an article devoted to this about 3 issues ago. >> >> Basically they said that as the % of "pilot error" crashes increases >> we may already be at the point where more lives would be saved by >> pilotless airplanes. > >This is certainly a debatable contention. Airbus certainly seems to believe >it: but it's also in the business of selling products "differentiated" by >their style of protection. Actually they cited Airbus as a good example of the *problem*. They said that most "human error" crashes have resulted from poor "situational awareness" and that this resuts from the way Airbus-like "glass cock- pits" take the pilot out of the loop. >The reality of the situation is that the safety record has remained pretty >much constant since the late 1970's--note: not the early 1980's, when the >first automated aircraft were introduced. It has stabilized at about 1500 >lives per year. Considering that RevenuePassengerMiles have been climbing steadily since that time this is not "stabilization"; it's steady improvement! >You would have a hard time convincing me that the number of fundamental errors >would not increase GREATLY with ground-based oversight, that the safety >margins would not go DOWN, as people fundamentally distanced from the reality >of a flight have a go/no-go say. I agree. But the article was discussing taking humans out of the loop altogether, not replacing pilots with ground-based controllers. The problem that exists now is that the pilot is partially out of the loop -- he still has the authority to fly the plane into a mountain, but he can't maintain the situational awareness to avoid it. According to the article the period just prior to landing (and to a lesser extent at t.o.) overburdens the pilot with vast amounts of system management tasks, so he loses a sense of where he (or the plane) really is. >>And moreover, the technology to do this either >> already exists or is close at hand. > >The technology isn't close to create safe, fully autonomous aircraft. The article seemed to feel that it's closer than many people think. >Was Bernard Ziegler the author of this article, perchance? :-) Julian Moxon. October 17 issue, pg 22. >> People will >> continue to cite those cases where coolness or quick thinking on >> the part of the crew did save the airplane or at least many lives. > >I wouldn't. Rather, I would ask how well we understand the *totality* of >in-flight incidents and actions, which are corrected by appropriate air- >manship. An old, true saying, is that a good pilot is a pilot who doesn't >have to show he's a good pilot. Is the capability of being able to maintain >control in a thunderstorm really that relevant, when 99% of all pilots would >simply have flown around the same thunderstorm? Except the examples I cited (DC 10 fan blade, 747 door) were not avoidable by pilot action. ---peter From kls Fri Dec 4 22:30:32 1992 Newsgroups: sci.aeronautics.airliners Path: news From: drinkard@bcstec.ca.boeing.com (Terrell D. Drinkard) Subject: Re: 747SP X-Submission-Date: Fri, 4 Dec 1992 01:45:27 GMT References: Message-ID: Approved: kls@ohare.Chicago.COM Organization: Boeing Sender: kls@ohare.Chicago.COM X-Submission-Message-Id: Date: 04 Dec 92 22:30:32 PST In article davidm@questor.rational.com (David Moore) writes: >I have no idea why they were delivering via London. Possibly to reproduce the MacRobertson Race of 1932 or '33. That went from London to Sydney, as I recall. The Boeing 247 and the Douglas DC-2 came in second and third behind a special build DeHavilland racer. That was a remarkable period of change in the airplane industry. However, that race tooks a few weeks (all of this is from memory - I don't have any references on it, or if I do I don't know where they are :-). Just a theory... -- Terry drinkard@bcstec.boeing.com "Anyone who thinks they can hold the company responsible for what I say has more lawyers than sense." From kls Fri Dec 4 22:30:32 1992 Newsgroups: sci.aeronautics.airliners Path: news From: drinkard@bcstec.ca.boeing.com (Terrell D. Drinkard) Subject: Re: hydraulic problems with DC-10's?? X-Submission-Date: Fri, 4 Dec 1992 01:55:18 GMT References: Message-ID: Approved: kls@ohare.Chicago.COM Organization: Boeing Sender: kls@ohare.Chicago.COM X-Submission-Message-Id: Date: 04 Dec 92 22:30:32 PST In article weiss@turing.SEAS.UCLA.EDU (Michael Weiss) writes: > >After the third post with this answer, I figure it's time to clarify my >statement. I am referring to the unbalance of WEIGHT, not THRUST. Nonetheless >I suppose we should go on... It doesn't matter, the loss of weight on that wing was actually a short-term plus in that incident (effectively generated a right-hand (positive) roll moment to help counter the loss of lift on the left wing). The adverse affect (aside from the obvious one of damage occuring during the departure of the engine and strut) is the increase in wing-root bending moment. >>Flight AA 191 lost the slats on the left hand >>wing (if memory serves) because of Douglas' failure to include mechanical >>lockouts on the slat actuators. They were not required to certify the >>airplane. > >Which doesn't disprove my theory. As it is, though, the loss of the slats >(which, according to all my aero classes, only lowers the stall speed but does >NOT increase the coefficient of lift!) was enough to stall the wing, more than >"countering" the loss of weight on the wing. You may wish to recheck your math. It isn't possible to lower the stall speed without improving the coefficient of lift (assuming constant weight, air density, and wing area). What is not increased is the total amount of lift produced, assuming unaccelerated, level flight. -- Terry drinkard@bcstec.boeing.com "Anyone who thinks they can hold the company responsible for what I say has more lawyers than sense." From kls Fri Dec 4 22:30:33 1992 Newsgroups: sci.aeronautics.airliners Path: news From: palmer@icat.larc.nasa.gov (Michael T. Palmer) Subject: Re: Flight envelope protections X-Submission-Date: 4 Dec 92 14:14:18 GMT References: Message-ID: Approved: kls@ohare.Chicago.COM Organization: NASA Langley Research Center, Hampton, VA USA Sender: kls@ohare.Chicago.COM X-Submission-Message-Id: Date: 04 Dec 92 22:30:33 PST rdd@cactus.org (Robert Dorsett) writes: >palmer@icat.larc.nasa.gov (Michael T. Palmer) wrote: >> This has some serious consequences. For example, in the China Airlines >> B-747 incident 300 nm northwest of San Francisco in 1985 (NTSB/AAR-86-03), >> the crew was forced to overstress (and structurally damage) > ^^^^^^ >That might be overstating the case a bit. :-) The NTSB report suggests >they didn't have a clue how to recover from the spiral, once they entered >it, lacking military aerobatic training and being completely disoriented. I >don't believe the report distinguishes the tailplane's damage as being >incidental or intentional. Agreed. I didn't mean to imply necessarily that they KNEW they needed to overstress the airframe, and it is *possible* that this occurred during control inputs that did not actually contribute to the recovery. It's been awhile since I read that report, and I didn't have it handy to refer to. >> crew recovered control with about 10,000 ft of altitude left (from an >> original high-altitude cruise). It is very likely that if the aircraft >> had prevented the crew from initiating control commands that would lead >> to aircraft damage, the aircraft (and passengers) would have been lost. >Your point's well taken, and the risks are certainly worth considering. But >allow me to play devil's advocate, for a minute, without diluting your argu- >ment, and suggest that the EFCS would have prevented an A3[2-4]0 from getting >into the unusual attitude to begin with. The protections are both aerodynamic >and input-filtering (and configuration-evaluating, and...). In the China >Air incident, the flip-over was caused by a "dumb" autopilot/autothrottle >design configuration oversight, following an engine abnormality. If a similar >event had occurred on an A3[2-4]0, the EFCS would probably have limited both >the authority of the FMS to put the airplane into the steep bank, *and* would >have provided maximum corrective action, using opposing controls, to keep the >airplane in the prescribed operating envelope. Well... given the recent post here about the A310 in Moscow going 88 degrees nose-up, I'm not sure that I agree that the Airbus EFCS would necessarily prevent the aircraft from attaining "unusual" attitudes. In fact, it was the "smarts" of the A310 autopilot that actually contributed to that incident. As that poster also mentioned, though, I would like VERY MUCH to see more documentation and a fuller description of exactly what happened. >This is from the Federal Register 54:17, January 27, 1989, pages 3989 and >3996: >P. 3989, the oh-so-enlightening, explanatory commentary: > "The second commenter believes that the flightcrew must be aware of any > failure conditions which affect the structural capability of the > airplane, whether or not a compensating procedure exists. The FAA does > not concur with this comment. It is not necessary for the flight crew > to be aware of a failure in the active control system during the flight > on which the failure occurs if there is no available corrective > action; however, the airplane should not be exposed to the failure > condition for an extended period of time. The flightcrew must > therefore be alerted to the failure condition prior to the next flight." Oh, I get it! Just because a condition exists that may affect OTHER choices I make about how to respond to OTHER occurrences during that flight, that doesn't mean that I have the right to know what is going on with my aircraft. Hmm, seems reasonable... NOT! >This is from the FAA, the agency in charge of establishing airworthiness and >certification practices in the United States! In reality, the A320 likely >*does* provide enough feedback: but the FAA, apparently unnecessarily, has >certainly opened the door for the practice to be introduced in subsequent >types. I agree completely. I work in the Human/Automation Integration Branch in the Flight Management Division at NASA Langley. We have worked for some time examining the complicated interrelationships between events that lead to accidents, and have even constructed software prototypes that try to determine these relationships and make them more explicit. What really scares us is the prevalent attitude of many in the industry that they can anticipate ALL the "important" ways that things will interact, and provide procedures for dealing with them. And whenever you point to an example of how they failed and how that lead to an accident, they respond "Oh, but we've already fixed that." Sure. But what about the NEXT one that you haven't "fixed" yet!?! By the way, the charter of our organization (as if you couldn't tell from what I've said so far) is NOT to solve problems in the cockpit by increasing the amount of automation. Rather, we seek to propose better ways of using the capabilities of both the automation and the flight crew, which may even mean rethinking many of the traditional tasks that automation is used for now. And we do NOT see the "pilot as manager" scenario as being necessarily ideal. Humans tend to make lousy system monitors. Ask the Nuclear people. Human-machine systems work best when the humans are actively *involved*. >> If nothing else, I hope I have brought up some topics that deserve >> discussion among readers of this newsgroup. After all, aren't we the >> ones in positions to influence our industry (all in our own way, of >> course)? >Especially in software, of particular relevance to the net. A lot (if not >most) of the people writing this code--4M on the A320, 10M+ on the >A330 and A340--are *not* aero engineers: just programmers, ostensibly with >CS backgrounds (a more frightening thought I can't imagine! :-)), performing >under strictly governed, structured, controlled environments: to specif- >ication. >Airbus even mentioned the "CS" types it brought in from "outside" to >buttress a comment on its quality-control practices, in an article, as if >to make the point that mere engineers weren't writing this stuff: the >"pros" are doing it. :-) Yeah, we know what we're doing, SURE... :-) Ummm... this point came up in a Newsweek article (now THERE'S an accurate and unbiased source of information!) about digital flight control systems. They were shocked that programmers, not pilots, were writing the software. I feel at least somewhat qualified to address this issue, since my undergrad is Aerospace Engineering, my master's is Computer Science, and I'm working on the Ph.D. in Human-Machine Systems. Pilots and engineers tend to be experts in specifying how things should happen. My experience with their programming ability is that they tend to not be aware of most of the advances in Computer Science that have occurred over the past 25 years. The result is poorly designed and implemented code that takes Herculean efforts to get working properly and maintain. On the other hand, programmers do not necessarily make good system designers... they tend to think in terms of how things will be implemented (and the limitations of that implementation) rather than in terms of what the system MUST be able to do. I have met only a few people who can combine both talents, to become very good system designers AND software designers. These people have the ability to hear what the pilots and engineers say, and translate that into a total system design, including software design, that meets the requirements and can be implemented. At THIS point, the actual programmers become involved. If changes need to be made due to, say, hardware limitations, then these can be incorporated by either a requirements OR an implementation change. So, I don't think you should be afraid that CS people are writing the code. In fact, you should be glad that they are. You just need to make sure that they are filling in the pieces of a software design that was put together by a competent person like I described above. >Robert Dorsett >rdd@cactus.org >...cs.utexas.edu!cactus.org!rdd I hope I get to meet you at a conference sometime soon! It's great to see that other people are grappling with the same issues. -- Michael T. Palmer, M/S 152, NASA Langley Research Center, Hampton, VA 23681 Voice: 804-864-2044, FAX: 804-864-7793, Email: m.t.palmer@larc.nasa.gov PGP 2.0 Public Key now available -- Consider it an envelope for your e-mail From kls Fri Dec 4 22:30:34 1992 Newsgroups: sci.aeronautics.airliners Path: news From: ditka!sgiblab!acd4.acd.com!HDFS1.acd.com!jbii ( John O. Bell II ) Subject: Re: Flight envelope protections, and mistrust of CS people X-Submission-Date: Fri, 4 Dec 1992 16:30:15 GMT References: Message-ID: Approved: kls@ohare.Chicago.COM Summary: Don't walk out in the rain w/o an umbrella AEs... you might drown Organization: Applied Computing Devices, Inc., Terre Haute IN Sender: kls@ohare.Chicago.COM X-Submission-Message-Id: <1992Dec4.163015.1333@acd4.acd.com> Date: 04 Dec 92 22:30:34 PST >Especially in software, of particular relevance to the net. A lot (if not >most) of the people writing this code--4M on the A320, 10M+ on the >A330 and A340--are *not* aero engineers: just programmers, ostensibly with ^^^^^^^^^^^^^^^^ I used to think that way as an AE grad, until I got my CS degree. Let's face it, guys, most AEs don't even know where to _begin_ coding this stuff. They don't have the experience or the background to make clean, fast, fault- tolerant code for these kind of applications. However, it is true that the CS types better have a good math/engineering background if they are going to understand exactly what is required for the code. You can't be a specialist in everything (much to the company's chagrin >:-)... "Darn, we gotta pay money to those CS people too? I thought our AEs could handle this!"). Side Note: Every time I see some aerospace article talk about how engineers are becoming accomplished computer scientists, I laugh my head off. I've been on both sides of the tracks, and I know better. Doing scientific programming in FORTRAN does not make you a CS god. >CS backgrounds (a more frightening thought I can't imagine! :-)), performing >under strictly governed, structured, controlled environments: to specif- >ication. Unlike the AEs, who are allowed to be creative, free-wheeling, loose cannons. Shyeah, and I'm the Easter Bunny... unless you are a conceptual designer, you are a glorified paper shuffler just like anyone else. Why do you think they call Boeing "The Lazy B", anyway (just an example). >Airbus even mentioned the "CS" types it brought in from "outside" to >buttress a comment on its quality-control practices, in an article, as if >to make the point that mere engineers weren't writing this stuff: the >"pros" are doing it. :-) Yeah, we know what we're doing, SURE... :-) Yeah, they do. Evidently the AEs on the project couldn't handle the task or they wouldn't have hired CS types. Either that, or the code they were putting together was such a spaghetti-mess of calls and gotos that they had to get CS people just so they could trace and fix the bugs. >Robert Dorsett >rdd@cactus.org >...cs.utexas.edu!cactus.org!rdd John Bell jbii@hdfs1.acd.com Applied Computing Devices, Inc. From kls Fri Dec 4 22:30:35 1992 Newsgroups: sci.aeronautics.airliners Path: news From: gregw@tribble.rational.com (Greg Wilson) Subject: Re: Boeing Book X-Submission-Date: Fri, 4 Dec 1992 23:08:39 GMT References: Message-ID: Approved: kls@ohare.Chicago.COM Organization: Rational Sender: kls@ohare.Chicago.COM X-Submission-Message-Id: Date: 04 Dec 92 22:30:35 PST michael@is3000.bmr.gov.au (System Support) writes: >Maybe someone can help me. I'm trying to get hold of a new book about >Boeing entitled something like "The Boeing Story". Does anyone have >any ideas about such a book, author? Any help would be much >appreciated. Does Boeing (Seattle) have an email address? >Thanking You in appreciation, >Michael Interestingly, I happen to be just about finished reading "Vision: The Story of Boeing," by Harold Mansfield. This is an older book (1966) that I picked up in a used bookstore. Published by Duell, Sloan, and Pearce, New York. Haven't read "Legend and Legacy" yet - but it looks like I will have to! >michael@is3000.bmr.gov.au >-- Greg Wilson gregw@rational.com From kls Fri Dec 4 22:30:37 1992 Newsgroups: sci.aeronautics.airliners Path: news From: Robert Dorsett Subject: More _Legend_and_Legacy_ X-Submission-Date: Thu, 3 Dec 92 5:57:18 CST References: Message-ID: Approved: kls@ohare.Chicago.COM Reply-To: rdd@rascal.ics.utexas.edu Sender: kls@ohare.Chicago.COM X-Submission-Message-Id: Date: 04 Dec 92 22:30:37 PST This was posted to rec.aviation.misc in October... --------------- From: mtc@hpcc01.corp.hp.com (Mark Cousins) Date: Fri, 9 Oct 1992 08:42:00 GMT Subject: More _Legend_and_Legacy_ Message-ID: <87330001@hpcc01.corp.hp.com> Organization: HP General Systems Division Newsgroups: rec.aviation.misc Okay, now it's my turn to post my favorite parts of _Lengend_and_Legacy_, the book about Boeing that Geoff Peck reviewed a while back. I have two: "Frye ordered six Stratoliners for TWA, and Pan Am signed for four; the only other potential 'launch' customer was the Netherlands' KLM, which expressed enough serious interest to send a technical delegation to Seattle for evaluation. Lindbergh, too, arrived at Boeing as a technical consultant for Pan Am, and secretarial hearts fluttered enough to generate a strong breeze. As was his custom, he went through the charade of using a fictitious name, a transparent device because he was easier to recognize than any movie star; at Boeing he was knwn either as 'Mr. Charles' or 'Mr. Morrow.' "He showed up ahead of schedule for a meeting one day, found Marge Blair rearranging some chairs, and offered to help. While they were lifting a chair together, his hand accidentally brushed hers. Lindbergh's face turned beet red, he mumbled an apology, and fled the room. "'Talk about being bashful and shy,' Marge laughed. 'You would have thought he tried to assault me. Most of the time he looked like a young farmer lost in the big city.'" ===== To be sure, I don't have that effect on secretarial hearts, and he flew better than I, but the story struck a resonance in me. I'm pretty bashful also and probably would react similarly (smile) . . . ===== This next one is, so far, my favorite of them all. "Another early 707 was involved in an incident that might be termed the Great Boeing Air Raid. This was Pan Am's second airplane off the line, being flown by Boeing's Lew Wallick and Water Haldeman, an FAA check pilot. They were doing some tests around the Los Angeles area and just before heading back to Edwards Air Force Base for refueling, Wallick remembered that the DC-8 was to make its first flight that day. "They were about 20 miles from Long Beach where the maiden flight was scheduled to take place later that morning, and Wallick asked the Long Beach control tower for permission to fly over the airport. He requested a modest 5,000 feet but the tower turned out to be more cooperative than requested. "'What is your identification number?' "'Seven-oh-seven Peter Alpha.' "'Are you a Boeing 707?' "'Affirmative.' "'Well, you're cleared to cross the airport at one thousand.' "About one mile out the tower controller, apparently a Boeing fan, changed his mind. "'You're cleared to cross the airport as low as you want,' he decreed. "At this particular moment, the airport coffee shop was jammed with media people having breakfast before covering the long-anticipated first flight of the DC-8. Wallick came screaming over the field at 500 feet, rattling the coffee shop windows. Everyone rushed outside in time to see a four-engine jetliner clawing skyward, four black plumes streaming in its wake. Reporters did the natural thing; they phoned their offices to report that the DC-8 had just taken off on its maiden flight. "Once that news was broadcast, Douglas employees planning to attend the first-flight ceremonies stayed away by the droves. Donald Douglas was livid; Wallick heard later that he called Bill Allen, raised hell, and when he found out that Wallick's copilot was an FAA employee, tried to get Haldeman fired." ===== Enjoy! Mark -- Mark Cousins Hewlett-Packard Co. mtc@hpsemc.cup.hp.com HP-UX VAB programs 19055 Pruneridge Ave., MS 46T5 (408) 447-4659 Cupertino, CA 95014 FAX: (408) 447-4364 From kls Fri Dec 4 22:30:38 1992 Newsgroups: sci.aeronautics.airliners Path: news From: Robert Dorsett Subject: night landings X-Submission-Date: Thu, 3 Dec 92 5:59:51 CST References: Message-ID: Approved: kls@ohare.Chicago.COM Reply-To: rdd@rascal.ics.utexas.edu Sender: kls@ohare.Chicago.COM X-Submission-Message-Id: Date: 04 Dec 92 22:30:38 PST Yet another comment on Legend & Legacy, from last month. --------------- From: dfr@usna.navy.mil (PROF D. Rogers (EAS FAC)) Newsgroups: rec.aviation.misc Subject: night landings Keywords: night, landings Message-ID: <2341@usna.NAVY.MIL> Date: 1 Nov 92 02:10:45 GMT G'day, Here is another quote without permission from `Legend and Legacy' The Story of Boeing and Its People. Now that winter has come and most of us will be flying more at night it's something to consider. Between late 1965 and early 1966 there were 4 fatal 727 crashes. The common denominator in each crash was excessive rate of descent at night. Two aircraft were flown into the water, a third hit high terrain near the airport over an unlighted sloping terrain. "Boeing engineers built a make-believe city on a table top place in front of a cockpit simulator, and put 12 experienced company pilots through identical approaches to the miniature city's airport. All 12 were told they were making a routine approach on a clear night to `Nighterton Field,' well-lighted and just south of the city, locate on a three-degree slope. Bisecting Nighterton was a river. The city lights were bright, but there were no lights between the beginning of the approach path and the runway---a typical approach over water or unlighted sloping terrain. Noe of the pilots had altimeters for reference. They were told to concentrate on flying the best approach path possible, reporting their estimated altitude every two miles starting at a point 18 miles from the airport. Their only active instruments were an airspeed indicator and a vertical velocity gauge. Eleven of the 12 crashed while making the approach. The closest any of these 11 got to the runway before pranging into imaginary ground was five miles. ....." 'Nuff said. Dave Rogers From kls Fri Dec 4 22:30:38 1992 Newsgroups: sci.aeronautics.airliners Path: news From: libove@libove.alf.dec.com (Jay Vassos-Libove) Subject: cost and names of assorted airliners? X-Submission-Date: Thu, 3 Dec 1992 21:50:43 GMT Message-ID: Approved: kls@ohare.Chicago.COM Organization: Digital Equipment Corporation, Atlanta Customer Support Center X-Submission-Message-ID: Sender: kls@ohare.Chicago.COM Date: 04 Dec 92 22:30:38 PST Does someone have a rundown of the approximate purchase cost (either current, or for out-of-production craft, the last price) of all jet passenger airliners? While we're at it, these are the jet passenger commercial airliners that I know of; what am I missing? (is the DC-8 a jet?) DC-9 DC-10 MD-80 series MD-11 BAC-111 B-727 B-737 B-747 B-757 B-767 L-1011 -- Jay Vassos-Libove libove@alf.dec.com Digital Equipment Corporation decwrl!alf.dec.com!libove Atlanta Customer Support Center Opinions? They're mine, mine, all mine! Alpharetta, Georgia and D.E.C. Can't have 'em! From kls Fri Dec 4 22:30:39 1992 Newsgroups: sci.aeronautics.airliners Path: news From: Keith Barr Subject: Re: hydraulic problems with DC-10s?? X-Submission-Date: Thu, 3 Dec 92 11:12:07 MST References: Message-ID: Approved: kls@ohare.Chicago.COM Sender: kls@ohare.Chicago.COM X-Submission-Message-Id: <199212031812.AA10282@ucsu.Colorado.EDU> Date: 04 Dec 92 22:30:39 PST Subject: Re: hydraulic problems with DC-10's?? Robert Dorsett Says: > Normally, given asymmetric thrust, you bank into the good engine(s): rudder's > normally used to augment the ailerons as necessary to control sideslip. Actually, you have this backwards. Rudder is used to control the yaw, and by controlling the yaw you introduce some sideslip that should be counteracted by banking into the good engine (raise the dead is the way I was tought to remember that :^) The way this works is as follows....we will have to make due with ASCII graphics: Normal Flight (Multi-Engine, Both turning) Left thrust Right thrust | | | A | ------X--A--X------ A A --A-- | | | | Drag Engine Out Flight (no correction) left thrust | | A ------X--A--X------ A CW Moment A |_ --A-- | | | | Drag Engine Out Flight (Yaw (moment) correction) left thrust | | A ------X--A--X------ A A --A--===>Rudder Force to counteract rotation | | Now you can see we have fixed the | Rotation with rudder, but we have an | unbalanced vector diagram, so the aircraft Drag will sideslip to the right By raising the dead engine we tilt the lift vector to the left which balances the force from the rudder. > The second issue is the moment produced by the combination of the "dead" > engine (with its drag) and the "good" engines. This is generally a minimal > issue, assuming the airspeed is there, and the pilot applies correct > technique. Most transport aircraft can fly with all engines out on one side, > although I do not know if this is an explicit regulatory requirement. As > long as the inherent longitudinal stability of the airplane (contributed > by the vertical stabilizer, rudder, wings, and fuselage) is sufficient to > overcome the yawing moment, the airplane can be controlled. So *correcting* > for a lost engine is a near-instantaneous correction, applied by the pilot, > needing no altitude reserve. Correct, but here is an added explanation for those who care: There is really only one concern of the pilot in an engine out situation, that is airspeed. The pilot, if he has done an appropriate preflight, will know whether he/she is able to climb on one engine out, so that is not a suprise. The biggest problem with an engine out is loss of control. This airspeed, called Vmc (Velocity Minimum Controllable) is the speed at which the rudder doesn't have enough air flowing over it to create enough force to counteract the moment from the good/dead engine. As long as you are above this speed, you should be controllable (ignoring the fact that one wing may stall if the slat comes up, but I am not talking about that case in particular). On the same thread, but different argument... Michael Weiss writes: >I have a hard time believing that an intact hydraulic system would have >prevented AA191 from crashing. Let's face it, a wing-mounted engine falling >off produces such a rediculous unbalance that even full aileron wouldn't be >able to counter it. >After the third post with this answer, I figure it's time to clarify my >statement. I am referring to the unbalance of WEIGHT, not THRUST. Nonetheless >I suppose we should go on... The change in weight from a lost engine is minimal. A fully loaded DC-10-30 weighs 572,000 pounds. A GE CF6-50C2B weighs only 8,731 pounds. This means that in normal flight each wing needs to support 286,000 pounds. If each wing supports the weight of its engine, now the left wing only needs to create 277,269 pounds of lift, a 3.05% decrease. I would imagine that ailerons easily can create a 3.05% increase in lift per side. References: Aviation Week and Space Technology 3/16/92 p. 102 Illustrated Encyclopedia of Commercial Aircraft pp 148-157 _____________________________ _____ | Keith Barr \ \ K \__ _____ | barr@ncar.ucar.edu \___________\ \/_______\___\_____________ | Comm/AS&MEL/Inst/IGI / < /_/ ..................... `-. |_____________________________/ `-----------,----,--------------' When you think how well basic appliances work, it's _/____/ hard to believe anyone ever gets on an airplane.--Calvin From kls Tue Dec 8 15:50:51 1992 Newsgroups: sci.aeronautics.airliners Path: news From: greg@octopus.dpsi.com (Gregory R. Travis) Subject: Re: hydraulic problems with DC-10s?? X-Submission-Date: Sat, 5 Dec 1992 16:28:27 GMT References: Message-ID: Approved: kls@ohare.Chicago.COM Organization: Data Parallel Systems, Inc Sender: kls@ohare.Chicago.COM X-Submission-Message-ID: Date: 08 Dec 92 15:50:51 PST Keith Barr writes: >The change in weight from a lost engine is minimal. A fully loaded DC-10-30 >weighs 572,000 pounds. A GE CF6-50C2B weighs only 8,731 pounds. This means >that in normal flight each wing needs to support 286,000 pounds. If each >wing supports the weight of its engine, now the left wing only needs to >create 277,269 pounds of lift, a 3.05% decrease. I would imagine that >ailerons easily can create a 3.05% increase in lift per side. It seems to me that they each have to perform at only half this figure: I.e. - the aileron on the wing that lost the engine needs to increase lift by about 1.5% while the aileron on the "good" wing needs to go negative and decrease that wing's lift by 1.5%. This is quite muddled though, as other anti-lift devices (such as spoilers) will deploy at a given amount of aileron deflection. In fact, and I don't have my DC-10 refs handy, I imagine that the ailerons on a -10 are locked in place when the flaps are up (not the case in the Chicago crash, I know) and that the loss of an engine AT CRUISE would have to be countered entirely by lift spoiling devices (i.e. spoilers) on the opposite wing with a suitable increase in the aircraft's overall AOA. In any case, the ORIGINAL poster's position that the loss of an engine from a wing, considering the engine's moment and weight, would render the plane uncontrollable is not supported either by analysis or historic precedent. Engines depart planes all the time... BTW, just to be pedantic: The wings do not each contribute exactly 50% of the total lift. Remember that that fuselage itself contributes a SUBSTANTIAL amount of lift at cruise as do the horizontal stabilizer surfaces (in certain flight regimes!). greg, the math bimbo -- Gregory Reed Travis D P S I Data Parallel Systems Incorporated greg@dpsi.com (For MX mailers only!) Bloomington, IN greg@indiana.edu (For the others) From kls Tue Dec 8 15:50:53 1992 Newsgroups: sci.aeronautics.airliners Path: news From: sandee@Think.COM (Daan Sandee) Subject: Re: 747-400 delivery flight (was :747SP X-Submission-Date: 5 Dec 1992 21:44:28 GMT References: Message-ID: Approved: kls@ohare.Chicago.COM Organization: TMC X-Submission-Message-Id: <1fr7rsINN57s@early-bird.think.com> Sender: kls@ohare.Chicago.COM Date: 08 Dec 92 15:50:53 PST In article , drinkard@bcstec.ca.boeing.com (Terrell D. Drinkard) writes: |> In article davidm@questor.rational.com (David Moore) writes: |> >I have no idea why they were delivering via London. |> |> Possibly to reproduce the MacRobertson Race of 1932 or '33. That went from |> London to Sydney, as I recall. The Boeing 247 and the Douglas DC-2 came in |> second and third behind a special build DeHavilland racer. That was a |> remarkable period of change in the airplane industry. However, that race |> tooks a few weeks (all of this is from memory - I don't have any references |> on it, or if I do I don't know where they are :-). For the sake of historical accuracy, London to Melbourne, 1932. As I remember, the DC-2 (a standard version flown by a KLM crew) came in second (first in the handicapped class) and might have won overall if it hadn't had engine trouble a couple of hundred miles from Melbourne. The race took around ten days. Cruising speed, 250-300 mph. Individual legs 1000 miles max (they couldn't fly Darwin to Sydney nonstop, had to refuel in some awful place in the interior). As Terry says, it showed the ongoing revolution in the airline industry. As to the Qantas delivery, LHR-SYD was presumably the route they needed to advertize, more so than LAX-SYD which would have been the obvious one. Daan Sandee sandee@think.com Thinking Machines Corporation Cambridge, Mass 02142 (617) 234-5044 From kls Tue Dec 8 15:50:54 1992 Newsgroups: sci.aeronautics.airliners Path: news From: rdd@rascal.ics.utexas.edu (Robert Dorsett) Subject: REVIEW of TEX JOHNSTON X-Submission-Date: Sun, 6 Dec 92 03:59:13 CST Message-ID: Approved: kls@ohare.Chicago.COM X-Submission-Message-Id: <9212060959.AA08798@rascal.ics.utexas.edu> Sender: kls@ohare.Chicago.COM Date: 08 Dec 92 15:50:54 PST Tex Johnston, Jet-Age Test Pilot By A. M. Johnston, with Charles Barton. Smithsonian Institution Press, 1991. 274 pp., illustrated. ISBN 1-56098-013-3, hardbound. Contents: Forward by William Randolph Hearst, Jr. First Flights Barnstorming Instructing and Ferrying Test Pilot Beginnings The First U.S. Jet Swept Wings and Rocket Power Cobra I and II: The Thompson Trophy Remote Control and Swept-wing Tests The X-1 Bell Helicopters The Move to Boeing The XB-47 The B-52 Program Problems and Prospects KC-135 Jet Tanker and the Dash 80 Jet Transport Public Relations, Hot Brakes and Flutter, and the 707 Barrel Roll 707 Certification and Pan Am Route Survey The Russian Tu-104 and the 707 European Round-Robin Transition Problems and Sales Trips Air India Accidents and Consequences Dyna Soar and Apollo Index Tex Johnston has gone down in history as "the guy who rolled the 707 prototype," but his career was much more extensive, fascinating one, ranging from flying a home-built glider as a teenager in the 1930's, to barnstorming, to serving as Boeing's most famous test pilot. This book provides an outline of that career, outlining his major life-events, and offering a collection of anecdotes on specific, important, problems and issues, many of which continue to befuddle net.dwellers. :-) These include, among other things, discussions of his ferry work during World War II; his work on the P-39; work on the XB-59; the first American jet aircraft; the X-1, the first American supersonic fighter; the XB-47, the first American jet bomber; the YB-52 (with tandem seating!); the 367-80, the 707 prototype; and the 707 itself. Johnston's best compared to Chuck Yeager, in terms of the legend that has built up around his accomplishments in the industry. His autobiography, however, is written for pilots, not the masses: just the facts. For example: his discussion of the first instance of in-flight spoilers on large aircraft: "The XB-47 contained many new, state-of-the-art configuration and structural design concepts. One of significance was the flexible- stress wing design, which provided a strong, flexible structure, allowing the wing to flex during gust and maneuvering loads, thus relieving high-stress areas and providing a smoother ride. "During a low-level flight demonstration at Wichita for observing dignitaries, I increased the air speed to approximately 435 mph and applied right aileron and up elevator for a right climbing turn. The airplane rolled left. I snapped the throttles closed and the lateral control to neutral, simultaneously increasing the climb angle. As the speed decreased to 425, the lateral control became normal. Analysis of the problem determined that at the high air loads at speeds above 425 mph, when the ailerons were deflected, the flexible wing began to twist, changing the angle of attack of the outboard wing sections, resulting in control reversal. This twist was called wing windup. 'It sure gets your attention,' I said. "That event led to the development of lateral control spoilers to eliminate the wing-twist problem on all Boeing airplanes. A lateral control spoiler is a rectangular door, hinged on its forward edge, which fits flush with the upper surface of the wing. During a turn in flight the doors are raised on the wing at the inside of the turn, decreasing the lieft of that wing so that the airplane rolls in that direction. When the spoilers are raised on both wings simultaneously, they serve as air brakes while retaining their lateral control function, and extremely valuable control for dissipating excessive airspeed." His analysis and discussion of early 707 crashes has particular relevance for the recent "safety" discussions: a snapshot into the industry's early learning curve, which we seem destined to repeat. The book is somewhat stiltish, and certainly doesn't share _Yeager_'s 14-point type and hick dialect. It doesn't provide a very good sense of historical continuity (certain major events never have a date attached to them, and there's sometimes little "filler" between major events, some of which were separated by years). It reads like it may have been a more lengthy text, edited down, perhaps excessively, by non-technical editors. There is a tad too much "pilot's ego" about certain events, but this is certainly forgivable, and a part of the character. But it remains highly readable, fascinating fare for those interested in the advent of jet aircraft in the 1940's and 1950's. About a third involves the barnstorming/WWII days, a third on the B-47 and B-52, and the rest on the 707. -- Robert Dorsett Internet: rdd@rascal.ics.utexas.edu UUCP: ...cs.utexas.edu!rascal.ics.utexas.edu!rdd From kls Tue Dec 8 15:50:56 1992 Newsgroups: sci.aeronautics.airliners Path: news From: lhe@sics.se (Lars-Henrik Eriksson) Subject: Re: hydraulic problems with DC-10's?? X-Submission-Date: Mon, 07 Dec 92 09:04:21 References: Message-ID: Approved: kls@ohare.Chicago.COM X-Submission-Message-Id: <9212070801.AA08453@sics.se> Sender: kls@ohare.Chicago.COM Date: 08 Dec 92 15:50:56 PST In article drinkard@bcstec.ca.boeing.com (Terrell D. Drinkard) writes: In article weiss@turing.SEAS.UCLA.EDU (Michael Weiss) writes: >Which doesn't disprove my theory. As it is, though, the loss of the slats >(which, according to all my aero classes, only lowers the stall speed but does >NOT increase the coefficient of lift!) You may wish to recheck your math. It isn't possible to lower the stall speed without improving the coefficient of lift (assuming constant weight, air density, and wing area). >From what I've learned, slats don't increase the coefficient of lift *for a given angle of attack*, but they do increase the critical angle of attack. That is, using slats you can get a higher coefficient of lift by having a higher angle of attack than normally possible. (So you are both right :-) Of course, this means that when the slats retract, either nothing happens to the lift at all (if the angle of attack is below the no-slats critical angle of attack), or the wing stalls instantly. ---- Lars-Henrik Eriksson Internet: lhe@sics.se Swedish Institute of Computer Science Phone (intn'l): +46 8 752 15 09 Box 1263 Telefon (nat'l): 08 - 752 15 09 S-164 28 KISTA, SWEDEN Fax: +46 8 751 72 30 From kls Tue Dec 8 15:50:58 1992 Newsgroups: sci.aeronautics.airliners Path: news From: dowlatir@cu1.crl.aecl.ca.crl.aecl.ca (Ramin Dowlati) Subject: Re: objects on wing tips X-Submission-Date: Mon, 7 Dec 1992 22:33:46 GMT References: Message-ID: Approved: kls@ohare.Chicago.COM Organization: AECL Research X-Submission-Message-Id: <1992Dec7.223346.25913@cu23.crl.aecl.ca> Sender: kls@ohare.Chicago.COM Date: 08 Dec 92 15:50:58 PST I have a few questions for any of you passenger airplane gurus. Several years ago, the aeropspace industry introduced vertical flap-like things on the ends of their airplane wings. I've only noticed these on 'larger' planes such Airbus, 747-400 and MD-11. Q1. What is the technical name for these flap-like things? Q2. Are they mobile or fixed? Q3. Do they only serve to stabilize the flight? Q4. Why haven't they appeared on smaller aircraft? Q5. The ones I saw on the Airbus were shaped like a 'V' and symmetric with the wing tip, ie. one leg of the 'V' was above the wing and the other pointed below the wing. Whereas the ones on the 747-400 looked like extensions of the actually wing, but bent 90 degrees upwards. Why the difference? I would greatly appreciate any answers. -Cheers From kls Tue Dec 8 15:51:04 1992 Newsgroups: sci.aeronautics.airliners Path: news From: rdd@cactus.org (Robert Dorsett) Subject: New Scientist article X-Submission-Date: Mon, 7 Dec 92 20:19:54 CST Message-ID: Approved: kls@ohare.Chicago.COM X-Submission-Message-Id: <9212080219.AA19173@cactus.org> Sender: kls@ohare.Chicago.COM Date: 08 Dec 92 15:51:04 PST In article nelson_p@apollo.hp.com (Peter Nelson) writ es: >>Remember that the standard definition of an airline pilot's job is 99.999% >>pute boredom, and 0.001% pure terror (I forget where this quote came from, >>and the ratios may be incorrect) - if this is anything like true, maybe >>human pilots really are on the edge of extinction ? > > _New Scientist_ had an article devoted to this about 3 issues ago. > > Basically they said that as the % of "pilot error" crashes increases > we may already be at the point where more lives would be saved by > pilotless airplanes. I looked through recent issues of _New Scientist_, seeking the article Peter referred to. It appears to be a 2-page essay from the October 17 issue, entitled "Will Accidents Always Happen?" The author of the article, Julian Moxon, has written for FLIGHT INTERNATIONAL for a number of years: his specialty appears to be safety; he's produced a number of good, comprehensive summaries and analyses of various crashes. Peter's comment seemed to suggest Moxon was advocating pilotless aircraft; in the context of previous post, I construed this as along the lines of Bernard Ziegler's "The computer can do it better" rhetoric, and reacted accordingly. :-) Moxon's point, however, is a bit more, well, integrated, and, if anything, far more ambitious. It's less an attack against *pilots*, per se, which has characterized Ziegler's remarks, but more a criticism of the ATC system. His basic point is that most crashes are landing crashes, controlled-flight-into-terrain. Some are caused by ATC malfeasance, some are interface problems. From the concluding remarks: "More worrying is that the skies are becoming increasingly congested, with predictions (despite the recession) of a doubling in air traffic movements over the coming decade. This puts extra pressure on the whole air transport system, not least on the pilots and air traffic controllers in the front line. In general, the system is (or will be) good enough to handle the extra traffic but--the statistics suggest--probably not good enough to prevent crashes like that in Kathmandu. It is as if we have arrived at the bare minimum of accidents. The challenge will be to maintain this minimum, given denser air traffic. "An inevitable question being asked in an increasingly automated world is whether we still need pilots. In many modern aircraft, the entire flight apart from the takeoff can handled by the autopilot, once programmed. But for obvious reasons, this is an emotive subject, which aircraft manufacturers carefully avoid in their official statements. Still, some designers are beginning to think seriously about the possibilities of making the flight crew's role more to do with systems management than flying the aircraft. "This would make the pilot part of a team including the entire air traffic system. Direct communication with the aircraft and its systems would be established by a radio-borne digital data link. This would send information on the aircraft's behavior to the ground and receive navigation data and commands that could be fed directly into its flight management system. Global positioning satellites would meanwhile observe it constantly. "Pilots worry that this would reduce them to little more than highly paid observers monitoring the aircraft's progress through the skies. But that time is a long way off. In the meantime, the focus remains on the behavior of the human brain." Those interested in a more extreme version of this would enjoy David Learmount's interview with Bernard Ziegler, in Flight International, September 23-29, 1992, Pp. 35-36. Ziegler is sort of Airbus's chief priest. --- Robert Dorsett rdd@cactus.org ...cs.utexas.edu!cactus.org!rdd From kls Tue Dec 8 15:51:09 1992 Newsgroups: sci.aeronautics.airliners Path: news From: weiss@ada.SEAS.UCLA.EDU (Michael Weiss) Subject: Re: hydraulic problems with DC-10's?? X-Submission-Date: 8 Dec 92 05:27:28 GMT References: Message-ID: Approved: kls@ohare.Chicago.COM Organization: SEASnet, University of California, Los Angeles X-Submission-Message-Id: <8852@lee.SEAS.UCLA.EDU> Sender: kls@ohare.Chicago.COM Date: 08 Dec 92 15:51:09 PST In article drinkard@bcstec.ca.boeing.com (Terrell D. Drinkard) writes: >In article weiss@turing.SEAS.UCLA.EDU (Michael Weiss) writes: >>I am referring to the unbalance of WEIGHT, not THRUST. >It doesn't matter, the loss of weight on that wing was actually a >short-term plus in that incident (effectively generated a right-hand >(positive) roll moment to help counter the loss of lift on the left wing). I'll reply to this below. >>Which doesn't disprove my theory. As it is, though, the loss of the slats >>(which, according to all my aero classes, only lowers the stall speed but does >>NOT increase the coefficient of lift!) was enough to stall the wing >It isn't possible to lower the stall >speed without improving the coefficient of lift (assuming constant weight, >air density, and wing area). Actually, assuming constant wing GEOMETRY, which includes, besides area, shape as well. Slats change the geometry. Their benefit is a reduction of the stall speed, through delay of separation. I believe the way that it does this is by directing a stream of air along the upper surface of the wing. In any case, my point is that there would have been a severe weight unbalance between the wings, and I have doubts that it could have been countered by the ailerons. The whole reason that there was a negative roll moment was that the left wing STALLED, not that it lost lift directly from the retracting slats. I'm still not convinced that even WITH the slats extended it could have been prevented. -- \ | | | | | | | | | | | | | | | | | | | | | | | | | | | | | | | | | | | | | | / - Michael weiss@seas.ucla.edu | School of Engineering & Applied Science - - Weiss izzydp5@oac.ucla.edu | University of California, Los Angeles - / | | | | | | | | | | | | | | | | | | | | | | | | | | | | | | | | | | | | | | \ From kls Tue Dec 8 15:51:12 1992 Newsgroups: sci.aeronautics.airliners Path: news From: raveling@Unify.com (Paul Raveling) Subject: Re: hydraulic problems with DC-10's?? X-Submission-Date: Tue, 8 Dec 92 20:18:22 GMT References: Message-ID: Approved: kls@ohare.Chicago.COM Organization: Unify Corporation (Sacramento) X-Submission-Message-Id: Sender: kls@ohare.Chicago.COM Date: 08 Dec 92 15:51:12 PST In article , drinkard@bcstec.ca.boeing.com (Terrell D. Drinkard) writes: > In article weiss@turing.SEAS.UCLA.EDU (Michael Weiss) writes: > > > >>Flight AA 191 lost the slats on the left hand > >>wing (if memory serves) because of Douglas' failure to include mechanical > >>lockouts on the slat actuators. They were not required to certify the > >>airplane. I'd be inclined to phrase that a bit differently. The certification requirements were satisfield by demonstrating safe flight with asymmetric slats. The catch is that (a) flying safely in this configuration requires keeping airspeed above the minimum (or AOA below the maximum) needed with slats retracted and (b) the crew didn't have a sufficient indication to judge immediately that they had asymmetric slats. This fits in with a pattern that's shown up in virtually all breeds of airliners where the cockpit's 'human interface' fails to supply needed information. This shows up in a fair variety of accidents in various forms -- unloading the autopilot produces surprising gyrations, aircraft FBW control logic reacted to factors other than the pilot's directions and the pilot didn't anticipate it, etc. One human factors problem is how to best inform the crew of simultaneous failures that each can be critical. For this DC-10 accident, they experienced loss of an engine at low altitude, followed quickly by partial loss of hydraulics and asymmetric slats. Each of these three primary circumstances call for prompt attention, and cockpit warnings of these and other consequent failures can overload the crew with failure alarms, becoming more of a problem than a solution. Bottom line: IMHO human factors engineering in the cockpit is more a more important target than airframe engineering for risk reduction. ------------------ Paul Raveling Raveling@Unify.com From kls Tue Dec 8 15:51:14 1992 Newsgroups: sci.aeronautics.airliners Path: news From: Pete Mellor Subject: Re: Gordon Corps X-Submission-Date: Tue, 8 Dec 92 21:26:41 GMT Message-ID: Approved: kls@ohare.Chicago.COM X-Submission-Message-Id: <10689.9212082126@csrsun8.cs.city.ac.uk> Sender: kls@ohare.Chicago.COM Date: 08 Dec 92 15:51:14 PST Edwin, > Can you provide me with any more details on the death of Gordon Corps. The following appeared in The Guardian, Saturday, August 15th 1992, p26:- Triple Tragedy in Nepal ----------------------- Bill Raeper, Martin Hoftun, Gordon Corps [Helen Kidd provided an obituary of Bill Raeper and Martin Hoftun, two British writers who were passengers on the Thai Airbus which crashed in the Himalayas on July 31st, killing all on board. Norman Barfield wrote the following about Gordon Corps.] Gordon Corps, one of Britain's most distinguished and accomplished test pilots, has died of mountain sickness in Nepal, aged 62. He was acting as deputy flight safety director with a team of Airbus Industrie on its way from Katmandu to investigate the Thai Airways crash on the Talkuassir mountain at 11,500ft. He was a graduate of the de Havilland Aeronautical Technical School, Hatfield, and his career took him to the RAF, to the Armament and Aircraft Experimental Establishment at Boscombe Down, and then to the Air Registration Board. He became chief test pilot to the Civil Aviation Authority in 1980. The development of major European commercial aircraft is now in the hands of the Airbus Industrie consortium and Gordon Corps took the significant step of joining AI in Toulouse in 1982 as an engineering test pilot. In the intervening 10 years, he has been involved in flight-testing the Airbus A310, A300-600 and A320 airliner family, with special responsibility for flying qualities. ----------------------- I'm sorry. That is all the information I have. Peter Mellor, Centre for Software Reliability, City University, Northampton Sq., London EC1V 0HB, Tel: +44(0)71-477-8422, JANET: p.mellor@city.ac.uk ----------------------------------------------------------------------------- From kls Tue Dec 8 15:51:19 1992 Newsgroups: sci.aeronautics.airliners Path: news From: Pete Mellor Subject: Northwest cancels Airbus X-Submission-Date: Tue, 8 Dec 92 10:36:45 GMT Message-ID: Approved: kls@ohare.Chicago.COM X-Submission-Message-Id: <9660.9212081036@csrsun8.cs.city.ac.uk> Sender: kls@ohare.Chicago.COM Date: 08 Dec 92 15:51:19 PST BBC Radio 4 news, 0700 GMT today:- Northwest Airlines has cancelled orders for 50 A320s and 14 A340s in a cost-cutting exercise. This was described as a serious blow to the European aircraft industry. Peter Mellor, Centre for Software Reliability, City University, Northampton Sq., London EC1V 0HB, Tel: +44(0)71-477-8422, JANET: p.mellor@city.ac.uk ----------------------------------------------------------------------------- From kls Tue Dec 8 15:51:20 1992 Newsgroups: sci.aeronautics.airliners Path: news From: hoyme@src.honeywell.com (Ken Hoyme) Subject: Northwest cancels Airbus X-Submission-Date: Tue, 8 Dec 92 09:51:54 CST References: Message-ID: Approved: kls@ohare.Chicago.COM X-Submission-Message-Id: <9212081551.AA03298@schrodinger.src.honeywell.com> Sender: kls@ohare.Chicago.COM Date: 08 Dec 92 15:51:20 PST In article mumble mumble Pete Mellor writes: > BBC Radio 4 news, 0700 GMT today:- > Northwest Airlines has cancelled orders for 50 A320s and 14 A340s in > a cost-cutting exercise. This was described as a serious blow to the > European aircraft industry. According to this mornings Minneapolis Star-Tribune (the home paper of NWA), that was 24 A340's canceled - 74 plane cancellations worth $3.4B. According to what I can find, they had 24 firm and 6 options for the A340. The 50 canceled A320s were the options, I believe. (Their original order was 50 firm and 50 options, if I recall correctly.) Their 16 A330s were not affected, but they aren't scheduled to receive any of those until 1997, so they felt no need to address that plane in this agreement. An additional 16 A320s will be delivered in 1993 (they currently have 34) along with two more B747-400s. With these cancellations and the delay of additional 747 and 757 deliveries, NWA currently is not scheduled to receive any new planes in 1994. (It is interesting to note that no Boeing planes were cancelled, but were slipped out instead.) Along with the cancellations, NWA announced $250M in new financing -- including financing from Airbus for the additional A320s! The analysts considered it quite remarkable that Airbus offered financing while at the same time getting a $3.4B cancellation. (But then, NWA is already deeply in debt to Airbus, since they decided to buy A320s based on a dynamite financing package that Boeing could not match.) This, combined with $900M worth of labor concessions (over three years) makes it unlikely that NWA will declare Chapter 11 in 1993. One local note of interest. The big buzz in Minnesota is whether this dooms the Airbus maintenance bases planned for the northern part of the state. About 18 months ago, NWA drove a deal with the state for an operating loan plus financing for the building of two maintenance bases - one for engines, the other for the larger structural checks (C/D). The construction of the bases has been held up in legal wrangling (over whether it is constitutional to use taxpayer money for such a purpose). With these cancellations, it is suspected that the bases will not be built, and NWA will fly their planes to Europe for C/D checks. Of course, they already have received and spent the operating loan.... Ken Ken Hoyme Honeywell Systems and Research Center (612)951-7354 3660 Technology Dr., Minneapolis, MN 55418 Internet: hoyme@src.honeywell.com From kls Tue Dec 8 15:51:21 1992 Newsgroups: sci.aeronautics.airliners Path: news From: Pete Mellor Subject: Re: Northwest cancels Airbus X-Submission-Date: Tue, 8 Dec 92 21:31:58 GMT References: Message-ID: Approved: kls@ohare.Chicago.COM X-Submission-Message-Id: <10699.9212082131@csrsun8.cs.city.ac.uk> Sender: kls@ohare.Chicago.COM Date: 08 Dec 92 15:51:21 PST > According to this mornings Minneapolis Star-Tribune (the home paper of > NWA), that was 24 A340's canceled - 74 plane cancellations worth $3.4B. You're right, Ken. (I'm not too wide awake at 7 am!) Pete ---------------------------------------------------------------------------- From kls Tue Dec 8 15:51:22 1992 Newsgroups: sci.aeronautics.airliners Path: news From: Bob Coggeshall Subject: Thai Airlines (was: Re: Boeing 747-300) X-Submission-Date: Sun, 6 Dec 1992 10:32:46 +0800 References: Message-ID: Approved: kls@ohare.Chicago.COM Reply-To: coggs@Cogwheel.COM X-Submission-Message-Id: <199212060232.AA11601@drewll.cogwheel.com> Sender: kls@ohare.Chicago.COM Date: 08 Dec 92 15:51:22 PST |> From: ashabana@agsm.ucla.edu (Ahmed Shabana) |> Thai is a world class airline that is profit |> oriented. They have a very close relationship with KLM and are one of |> the most successfull airlines in the fast growing Asian market. Thai's cabin service is reknowned, yes. But I really wonder what their finances really look like; I've seen alot of press that portrays Thai Air as just a quasi-private venture of the Thai military. A thread of discussion I'd like to see is _Two_ fatal incidents in as many months involving airbuses aborting approaches at Kathmandu. One of them invovled a Thai Air airbus. //////////////////////////////////////////////////////////////// Bob Coggeshall, President Cogwheel Incorporated - Producers of Low-cost dial-up IP Routers coggs@Cogwheel.COM //////////////////////////////////////////////////////////////// From kls Wed Dec 9 18:45:30 1992 Newsgroups: sci.aeronautics.airliners Path: news From: fredch@phx.mcd.mot.com (Fred Christiansen) Subject: Re: 757 highest thrust to weight ratio ? X-Submission-Date: Thu, 3 Dec 92 13:23:11 -0700 References: Message-ID: Approved: kls@ohare.Chicago.COM Organization: Motorola Computer Group, RT Software Sender: kls@ohare.Chicago.COM X-Submission-Message-Id: <9212032023.AA21978@quad4.phx.mcd.mot.com> Date: 09 Dec 92 18:45:30 PST In article kls@chicago.com (Karl Swartz) writes: >Not surprisingly, the top 11 were all twins. Why "not surprisingly"? As a layman with no background in this stuff, I would have tho't that manufacturers would keep the thrust-to-weight in some ballpark range (for economic reasons). And that the advent of larger twins is because more powerful engines have become available, eliminating the need for a 3rd (or in the case of, say, MD-10 class aircraft, the need for a 4th). -- Fred Christiansen, Motorola, 2900 S Diablo Way, Tempe, AZ 85282 "Canajun, eh?" fredch@phx.mcd.mot.com || uunet!phx.mcd.mot.com!fredch || +1 602-438-3464 ".. I have set before you Life and Death, blessing and cursing; therefore choose Life, that both you and your children may live" Deut 30:19 From kls Wed Dec 9 18:45:32 1992 Newsgroups: sci.aeronautics.airliners Path: news From: Robert Dorsett Subject: Re: Northwest cancels Airbus X-Submission-Date: Tue, 8 Dec 92 19:32:23 CST References: Message-ID: Approved: kls@ohare.Chicago.COM Reply-To: rdd@rascal.ics.utexas.edu Sender: kls@ohare.Chicago.COM X-Submission-Message-Id: Date: 09 Dec 92 18:45:32 PST >Their 16 A330s were not affected, but they aren't scheduled to receive >any of those until 1997, so they felt no need to address that plane in >this agreement. The 1997 figure was apparently arrived at in September, when they deferred the A330 deliveries (originally scheduled between 1994 and 1996) to 1997 to 1999. >With these cancellations, it is suspected that the bases will not be >built, and NWA will fly their planes to Europe for C/D checks. Of >course, they already have received and spent the operating loan.... It'll be interesting to see what ramifications this has on the UAL deal: did United plan on using Airbus/NWA North American facilities? R. From kls Wed Dec 9 18:45:32 1992 Newsgroups: sci.aeronautics.airliners Path: news From: Pete Mellor Subject: Re: Re: Gordon Corps X-Submission-Date: Wed, 9 Dec 92 09:36:10 GMT References: Message-ID: Approved: kls@ohare.Chicago.COM Sender: kls@ohare.Chicago.COM X-Submission-Message-Id: <10870.9212090936@csrsun8.cs.city.ac.uk> Date: 09 Dec 92 18:45:32 PST Ed, > Thanks for the information. You're very welcome. I forgot to add the following footnote from the obituary in The Guardian: Gordon Corps, born November 11, 1929; died August 4, 1992. (At least this tells you the precise date of his death. I do not know why it took so long for the obituary to appear.) Pete ----------------------------------------------------------------------------- From kls Wed Dec 9 18:45:33 1992 Newsgroups: sci.aeronautics.airliners Path: news From: dmarble@magnus.acs.ohio-state.edu (Duane F Marble) Subject: Re: New Scientist article X-Submission-Date: Wed, 9 Dec 1992 14:05:55 GMT References: Message-ID: Approved: kls@ohare.Chicago.COM Organization: The Ohio State University Sender: kls@ohare.Chicago.COM X-Submission-Message-Id: <1992Dec9.140555.25889@magnus.acs.ohio-state.edu> Date: 09 Dec 92 18:45:33 PST A small point with respect to the material quoted from the New Scientist: Global Positioning System (GPS) birds do not "observe" anything, they just permit a ground based unit to compute it's location. -- Duane F. Marble E-mail: dmarble@magnus.acs.osu.edu Department of Geography Telephone: (614) 292-2250 The Ohio State University Columbus, Ohio 43210 Fax: (614) 292-6213 From kls Wed Dec 9 18:45:35 1992 Newsgroups: sci.aeronautics.airliners Path: news From: ckd@eff.org (Christopher Davis) Subject: Re: Boeing Book X-Submission-Date: Wed, 9 Dec 1992 18:14:20 GMT References: Message-ID: Approved: kls@ohare.Chicago.COM Organization: Electronic Frontier Foundation Tech Central Sender: kls@ohare.Chicago.COM X-Submission-Message-Id: Date: 09 Dec 92 18:45:35 PST RD> == Robert Dorsett RD> It might also be "Boeing: Planemaker since 1916," by one Philip M. Bowers. RD> It's an exhaustive review of all the airplanes (and variants) Boeing's RD> produced, sort of a mini Jane's. It's a long book (over 600 pages). It RD> appeared at Bookstop here in Austin sometime this summer; it's about $36. There are three books about Boeing that, together, do an exemplary job of covering the company's growth, products, and people. _Legend & Legacy_ is the best for "reading"; it is the one most likely to keep you up all night until you finish. (I believe it is also the newest.) It focuses on the people, rather than the products, and has been extensively discussed here recently. _Boeing: Planemaker to the World_ is a "coffee-table" book with plenty of wonderful photos and charts and the like, including discussion of the Boeing hydrofoils, furniture, Lunar Roving Vehicle, and streetcars. (Oh, and airplanes :) _Boeing Aircraft since 1916_ (title may not be exact, I have to find my copy) goes into much more detail on the aircraft, including lists of customer codes, production information, and the like. Many b&w photos, though it lacks the spectacular color photos of _Planemaker to the World_. There are others, of course. _Vision_ is a bit out of date. _Boeing Trivia_, by Carl Cleveland (former PR director, as I recall) is a good companion to _Legend & Legacy_, lacking the "storyline" aspects but adding a number of wonderful anecdotes from the "inside". -- Christopher K. Davis | ``Usenet seems to run much like the Kif (or, EFF #14 | for the TV generation, Klingon) high command. System Administrator, EFF | Whoever takes action and can be heard wins.'' +1 617 864 0665 [CKD1] | --Peter da Silva From kls Wed Dec 9 18:45:37 1992 Newsgroups: sci.aeronautics.airliners Path: news From: ckd@eff.org (Christopher Davis) Subject: Re: REVIEW of TEX JOHNSTON X-Submission-Date: Wed, 9 Dec 1992 18:18:11 GMT References: Message-ID: Approved: kls@ohare.Chicago.COM Organization: Electronic Frontier Foundation Tech Central Sender: kls@ohare.Chicago.COM X-Submission-Message-Id: Date: 09 Dec 92 18:45:37 PST RD> == Robert Dorsett RD> Tex Johnston, Jet-Age Test Pilot RD> By A. M. Johnston, with Charles Barton. RD> Smithsonian Institution Press, 1991. RD> 274 pp., illustrated. RD> ISBN 1-56098-013-3, hardbound. The (excellent) Bantam Air & Space series has this available in softcover for a lot less money (and a lot more portability). They also have some other good books for airliner fans, including Gann's _Band of Brothers_ and a treatment of the DC-3. -- Christopher K. Davis | ``Usenet seems to run much like the Kif (or, EFF #14 | for the TV generation, Klingon) high command. System Administrator, EFF | Whoever takes action and can be heard wins.'' +1 617 864 0665 [CKD1] | --Peter da Silva From kls Wed Dec 9 18:45:37 1992 Newsgroups: sci.aeronautics.airliners Path: news From: driscoll@src.honeywell.com (Kevin Driscoll) Subject: Re: Northwest cancels Airbus X-Submission-Date: Wed, 9 Dec 92 13:31:04 CST References: Message-ID: Approved: kls@ohare.Chicago.COM Sender: kls@ohare.Chicago.COM X-Submission-Message-Id: <9212091931.AA08647@couqusmungus.src.honeywell.co> Date: 09 Dec 92 18:45:37 PST > >With these cancellations, it is suspected that the bases will not be > >built, and NWA will fly their planes to Europe for C/D checks. Of > >course, they already have received and spent the operating loan.... > > It'll be interesting to see what ramifications this has on the UAL deal: > did United plan on using Airbus/NWA North American facilities? A local (Minneapolis) TV story said that NWA is considering other airlines' use of the facilities in deciding when/if to build. From kls Thu Dec 10 00:52:08 1992 Newsgroups: sci.aeronautics.airliners Path: news From: Robert Dorsett Subject: Re: hydraulic problems with DC-10's?? X-Submission-Date: Wed, 9 Dec 92 7:50:44 CST References: Message-ID: Approved: kls@ohare.Chicago.COM Reply-To: rdd@rascal.ics.utexas.edu Sender: kls@ohare.Chicago.COM X-Submission-Message-Id: Date: 10 Dec 92 00:52:08 PST Before starting: the left pylon assembly weight, btw, was 13,477 lbs, from the accident report. A whole bunch of figures had been floating around... In Michael Weiss writes: >In any case, my point is that there would have been a severe weight unbalance >between the wings, and I have doubts that it could have been countered by the >ailerons. The whole reason that there was a negative roll moment was that the >left wing STALLED, not that it lost lift directly from the retracting slats. >I'm still not convinced that even WITH the slats extended it could have been >prevented. Allow me to throw in my $0.02 worth. 1. A slat increases the maximum effective lift coefficient for a wing section. How this is done is irrelevant to this discussion: the result is that the lift coefficient goes up. Slats permit the wing to produce a greater lift at slower airspeeds, i.e., they drive the stall speed down. When used with trailing edge flaps, they offer even better lift characteristics, and improve handling characteristics. 2. If we take away the slats, then the maximum lift coefficient goes down. By definition. This means the stall speed goes up for the resulting configuration. By definition. 3. The accepted procedure was to climb at V2 until 800' AGL, then to lower the nose and accelerate. For a normal, undamaged aircraft, at 379,000 lbs, V2 was 153 knots. In the damaged aircraft, the minimum controllable airspeed, with a 4 degree left bank, into the missing engine, was 159 knots. Therefore, if the crew were to fly a standard engine-out profile, at 153 knots, they would have been beneath the minimum controllable airspeed for the damaged aircraft (159 knots). During the investigation, the NTSB asked 13 qualified pilots to fly various takeoff profiles. 70 takeoff simulations were flown. All crashed the airplane when flying the crash profile. Several pilots, when left to their own devices, and with extensive knowledge of the events, managed to control the airplane, nonetheless, by recognizing the initial roll and applying full opposite aileron and significant rudder, and lowering the nose to gain air- speed. All pilots who received appropriate feedback, via a functioning stickshaker, and who increased their airspeed to stay above the stickshaker value--168 knots--saved the airplane. I really fail to see what the problem is, here. The engine fell off after V1. This didn't affect the aerodynamic characteristics of the wing itself: it became a control problem. It also killed the electrical system driving the captain's stick-shaker, and killed a hydraulic system. The latter caused the slats to retract within 20 seconds of failure. The slat retraction DID affect the wing: it then became both a control and aerodynamic problem. Exercising established control practices in an unknown aerodynamic regime crashed the airplane (I'd love to know whether this went into Airbus's "pilot error" database :-)). Had the slats remained down, the airplane would have survived the engine failure, even with the failure of the stall warning system. Other airframe manufacturers have manual locking mechanisms for their slat jackscrews. McDonnell Douglas relied on hydraulic pressure to hold it all together. Incidentally, this problem wasn't corrected: the SUX DC-10 also experienced extension of its slats after it lost all its hydraulics. I'd suggest you obtain a copy of the accident report (NTSB-AAR-79-17), and look it over, closely. It has more than enough data for back-of-the- envelope calculations. Nothing in it suggests that weight or moments following engine separation played a significant role. Lastly, I'd note that there was SIGNIFICANT public and industry concern about the DC-10's safety after this crash: the FAA's extraordinary grounding of the airplane, inappropriate though it may have been, is testimony to that. All of the manufacturers had something to contribute, and a great deal of manpower was invested in finding the cause. There was REAL concern that the airplane wasn't airworthy, even by FAA's standards. Every analysis or comment I've ever seen on this crash has concentrated on the slat retrac- tion being the proximal cause for the crash. I've never seen the weight issue raised. If you have "hard" evidence that it HAS been, some references would be useful, since it's not a well-publicised theory. If you're basing your comments on classroom experience, as I believe you indicated, it might be a worthwhile learning exercise to raise it in class, or print this discussion and privately discuss it with your professor: but be sure to let us know the outcome. -- Robert Dorsett Internet: rdd@rascal.ics.utexas.edu UUCP: ...cs.utexas.edu!rascal.ics.utexas.edu!rdd From kls Thu Dec 10 00:52:10 1992 Newsgroups: sci.aeronautics.airliners Path: news From: barr@ash.mmm.ucar.EDU (Keith Barr) Subject: Re: hydraulic problems with DC-10s?? X-Submission-Date: Wed, 9 Dec 92 8:39:46 MST References: Message-ID: Approved: kls@ohare.Chicago.COM Sender: kls@ohare.Chicago.COM X-Submission-Message-Id: <9212091539.AA01844@walnut.mmm.ucar.edu> Date: 10 Dec 92 00:52:10 PST greg@octopus.dpsi.com (Gregory R. Travis) writes: > BTW, just to be pedantic: The wings do not each contribute exactly > 50% of the total lift. Remember that that fuselage itself contributes > a SUBSTANTIAL amount of lift at cruise as do the horizontal stabilizer > surfaces (in certain flight regimes!). What flight regimes might those be? Unstable flight, as in the F-16? Actually, in most aircraft the horizontal stabilizer is downlifting, so the wings have to create more lift, and the body contributes to the pitching moment, but contributes very little lift (unless we are talking about the B-2). For the horizontal stabilizer to lift up, the CG would have to behind the Center of lift, which is not usually allowed. I would bet a dollar that the DC-10 has a down lifting tail, although I don't know for sure. If you want references for either of these points, I recommend that you look at Etkin's book on stability (an engineering text I used in my aircraft design class that would explain in painful detail why you need the CG in front of the Center of Lift), and you might check out K.D. Wood's text on aircraft design for discussions about pitching moment and lift created by the fuselage. (is it just me, or is this thread becoming less and less appropriate for this newsgroup?) _____________________________ _____ | Keith Barr \ \ K \__ _____ | barr@ncar.ucar.edu \___________\ \/_______\___\_____________ | Comm/AS&MEL/Inst/IGI / < /_/ ..................... `-. |_____________________________/ `-----------,----,--------------' When you think how well basic appliances work, it's _/____/ hard to believe anyone ever gets on an airplane.--Calvin From kls Thu Dec 10 00:52:11 1992 Newsgroups: sci.aeronautics.airliners Path: news From: greg@saltydog.dpsi.com (Gregory R. Travis) Subject: Re: objects on wing tips X-Submission-Date: Wed, 9 Dec 1992 16:24:48 GMT References: Message-ID: Approved: kls@ohare.Chicago.COM Organization: Data Parallel Systems, Inc X-Submission-Message-ID: Sender: kls@ohare.Chicago.COM Date: 10 Dec 92 00:52:11 PST In dowlatir@cu1.crl.aecl.ca.crl.aecl.ca (Ramin Dowlati) writes: >Q1. What is the technical name for these flap-like things? Winglets. >Q2. Are they mobile or fixed? I've always seen them fixed. >Q3. Do they only serve to stabilize the flight? No. >Q4. Why haven't they appeared on smaller aircraft? They have. Look at the new Learjets or, heck, any Cessna after about 1974. As I understand it, the winglets serve to prevent "spillage" of air from the high-pressure area under the wing to the low-pressure area above it. In flight, there is a considerable difference in air pressure between the lower side of the wing and the upper side. Nature, being as it is, finds this situation abhorrent and constantly tries to find ways to equalize the air pressure between the two surfaces. One way of doing this is to allow air to spill out of the underside of the wing at the tip and curl upward to the upper side of the wing. Unfortunately for the airline bean-counters and aerodynamicists, this spillage of air creates strong vortices - miniature tornados lying horizontally along the axis of the fuselage. The vortices themselves and the loss of pressure from the lower side of the wing both contribute to drag and loss of lift & wing efficiency. The "winglets" prevent the spillage up to the upper wing surface via a physical wall. The air escapes from the bottom surface and tries to curl up to the upper surface and is stopped by the winglet. There are several different winglet designs. As I mentioned, Cessna and others have been doing it for some time with drooped wingtips. One can put the barrier so that it is either hanging DOWN (Cessna) from the wing, or so that it protrudes ABOVE the wing (Boeing) - the net effect is, roughly, the same. Likewise, an airplane in ground-effect flies much more efficiently because the proximity of the wing to the ground prevents the full formation of the wingtip vorticies and thus air spillage to the top of the wing is greatly reduced. The big issue is in constructing a winglet that recovers more lost lift than it effectively destroys with added drag. It's interesting that Airbus, I believe, is intending to offer a device which would trail from each wingtip and employ a propeller of some sort. The propeller would hook to a electrical generator and produce power from the wingtip vortices. greg -- Gregory Reed Travis D P S I Data Parallel Systems Incorporated greg@dpsi.com (For MX mailers only!) Bloomington, IN greg@indiana.edu (For the others) From kls Thu Dec 10 00:52:13 1992 Newsgroups: sci.aeronautics.airliners Path: news From: gregory@bcstec.ca.boeing.com (Greg Wright) Subject: Re: hydraulic problems with DC-10s?? X-Submission-Date: Wed, 9 Dec 1992 20:23:39 GMT References: Message-ID: Approved: kls@ohare.Chicago.COM Organization: Boeing Sender: kls@ohare.Chicago.COM X-Submission-Message-Id: Date: 10 Dec 92 00:52:13 PST In article greg@octopus.dpsi.com (Gregory R. Travis) writes: > >BTW, just to be pedantic: The wings do not each contribute exactly >50% of the total lift. Remember that that fuselage itself contributes >a SUBSTANTIAL amount of lift at cruise as do the horizontal stabilizer >surfaces (in certain flight regimes!). > This is not as substantial as one might think. While some lifting bodies are very good at producing lift the typical commercial fuselage is not. Because of it's inablility to be a very efficient producer of lift, ie. it makes LOTS of drag in doing so, we try to keep the lift of the body to a minimun. As far as the stab's go, at cruise they will be counter- acting a nose-down or nose-up pitching moment depending on the CG and gross weight at that particular time in the flight. Because, again, with lift you get drag, the horiz. stab. is best kept at zero lift if possible. >greg, the math bimbo >-- >Gregory Reed Travis D P S I >Data Parallel Systems Incorporated greg@dpsi.com (For MX mailers only!) >Bloomington, IN greg@indiana.edu (For the others) > -- ________Greg Wright____________ "I struggle to be brief | gregory@bcstec.ca.boeing.com | and become obscure." | gregory@halcyon.com | |____uunet!bcstec!gregory_______| NOT A BOEING SPOKESPERSON. From kls Thu Dec 10 00:52:14 1992 Newsgroups: sci.aeronautics.airliners Path: news From: gregory@bcstec.ca.boeing.com (Greg Wright) Subject: Re: objects on wing tips X-Submission-Date: Wed, 9 Dec 92 12:45:47 PST References: Message-ID: Approved: kls@ohare.Chicago.COM Organization: The Boeing Co. (Aerodynamics) Sender: kls@ohare.Chicago.COM X-Submission-Message-Id: <9212092045.AA20037@bcstec.ca.boeing.com> Date: 10 Dec 92 00:52:14 PST I have a few questions for any of you passenger airplane gurus. Several years ago, the aeropspace industry introduced vertical flap-like things on the ends of their airplane wings. I've only noticed these on 'larger' planes such Airbus, 747-400 and MD-11. Q1. What is the technical name for these flap-like things? Winglets.... Q2. Are they mobile or fixed? Fixed on our airframes.... Q3. Do they only serve to stabilize the flight? They help to reduce the induced drag by preventing the wingtip vorticies from forming...well keep them from being as strong at the tips anyway. They also produce lift since they are not mounted absolutly vertical. In the same vain, they produce a little thrust. Q4. Why haven't they appeared on smaller aircraft? The weight. Also, in theory they have big effect on the reduction of total drag, however, this has not been the case. The effect is small enough that we put them only on the long range planes. This is why we removed them from the wings of the 747-400D, which was made for short hauls.. Q5. The ones I saw on the Airbus were shaped like a 'V' and symmetric with the wing tip, ie. one leg of the 'V' was above the wing and the other pointed below the wing. Whereas the ones on the 747-400 looked like extensions of the actually wing, but bent 90 degrees upwards. Why the difference? Same principle, just different methods. As the technology advances you will see many different devices designed to to the same thing. NASA flew a plane that had six or seven winglets, all at different angles to the freestream.... I would greatly appreciate any answers. -Cheers ________Greg Wright____________ High Lift Development | gregory@bcstec.ca.boeing.com | Aerodynamics | gregory@halcyon.com | |____uunet!bcstec!gregory_______| NOT A BOEING SPOKESPERSON. From kls Thu Dec 10 00:52:14 1992 Newsgroups: sci.aeronautics.airliners Path: news From: kls@ohare.Chicago.COM (Karl Swartz) Subject: Re: 757 highest thrust to weight ratio ? X-Submission-Date: Thu, 10 Dec 1992 07:19:56 GMT References: Message-ID: Approved: kls@ohare.Chicago.COM Organization: Chicago Software Works X-Submission-Message-ID: <1992Dec10.071956.2591@ohare.Chicago.COM> Sender: kls@ohare.Chicago.COM Date: 10 Dec 92 00:52:14 PST In article fredch@phx.mcd.mot.com (Fred Christiansen) writes: >In article kls@chicago.com (Karl Swartz) writes: >>Not surprisingly, the top 11 were all twins. >Why "not surprisingly"? As a layman with no background in this stuff, >I would have tho't that manufacturers would keep the thrust-to-weight >in some ballpark range (for economic reasons). And that the advent >of larger twins is because more powerful engines have become available, >eliminating the need for a 3rd (or in the case of, say, MD-10 class >aircraft, the need for a 4th). The catch here is that once committed to takeoff, the aircraft must be able to fly long enough to return for a safe landing with one engine out. With four engines, this in effect means you've got to carry 33% more power than the bare minimum to keep you in the air; with two you need a full 100%. Note that this does not *necessarily* imply a much higher thrust/weight ratio -- depending on how the certification regs are written, one could figure maybe twenty minutes to dump fuel and return and certify the engine for a substantial overload thrust quite a bit above the normal "maximum" thrust. But since essentially the same engines would also be used on three- and four-engine aircraft this probably wouldn't make much difference. Looking at this from the view of normal operations, the engines on a twin will normally not be worked as hard which should lead to higher reliability and lower maintenance costs. And, as mentioned in regard to the 757, better climb performance when needed. The Airbus A330 and A340 offer an interesting comparison, since they are identical save the engines (two on the A330, four on the A340), their wing attachments, and those parts of the controls directly affected by the engine differences. (Fuselage lengths differ too though this is true even amongst A340 models.) The A340, with four engines, offers freedom from ETOPS restrictions for long, overwater routes. The A330, with two larger engines, offers the maintenance economies of fewer engines for shorter or overland routes. It's interesting, then, that Lufthansa chose the A340 even though the A330 would have served most of the routes they had in mind. As I recall, they felt that four less-stressed engines would be cheaper to operate and maintain than two larger engines. Given the overall popularity of twins this doesn't seem to be a widespread view; it may simply reflect greater confidence in the more mature engines of the A340 compared to the A330. -- Karl Swartz |INet kls@ditka.chicago.com 1-415/854-3409 |UUCP uunet!decwrl!ditka!kls |Snail 2144 Sand Hill Rd., Menlo Park CA 94025, USA Send sci.aeronautics.airliners submissions to airliners@chicago.com From kls Thu Dec 10 00:52:16 1992 Newsgroups: sci.aeronautics.airliners Path: news From: kls@ohare.Chicago.COM (Karl Swartz) Subject: Re: Northwest cancels Airbus X-Submission-Date: Thu, 10 Dec 1992 07:25:37 GMT References: Message-ID: Approved: kls@ohare.Chicago.COM Organization: Chicago Software Works X-Submission-Message-ID: <1992Dec10.072537.2670@ohare.Chicago.COM> Sender: kls@ohare.Chicago.COM Date: 10 Dec 92 00:52:16 PST In article rdd@rascal.ics.utexas.edu writes: >>With these cancellations, it is suspected that the bases will not be >>built, and NWA will fly their planes to Europe for C/D checks. Of >>course, they already have received and spent the operating loan.... >It'll be interesting to see what ramifications this has on the UAL deal: >did United plan on using Airbus/NWA North American facilities? I've never seen any mention of United's plans with regard to maintenance of their A320 fleet. However, all of them were to be leased (from GPA and Kawasaki, amongst others) and half of the 50 firm orders came with three-year, walkaway leases. This would not lead one to expect a tremendous investment in maintenance. Perhaps someone from Indiana might be able to shed more light on this -- I believe United's upcoming Indianapolis maintenance base was intended to take over the 737s from San Francisco, and thus it would be the obvious candidate for A320 maintenance if done in-house. -- Karl Swartz |INet kls@ditka.chicago.com 1-415/854-3409 |UUCP uunet!decwrl!ditka!kls |Snail 2144 Sand Hill Rd., Menlo Park CA 94025, USA Send sci.aeronautics.airliners submissions to airliners@chicago.com From kls Thu Dec 10 00:52:18 1992 Newsgroups: sci.aeronautics.airliners Path: news From: sajja@vu-vlsi.vill.edu (Go Skins....) Subject: Book's on Jets X-Submission-Date: Tue, 8 Dec 1992 04:21:12 GMT Message-ID: Approved: kls@ohare.Chicago.COM Organization: Villanova University Sender: kls@ohare.Chicago.COM X-Submission-Message-Id: Date: 10 Dec 92 00:52:18 PST Iam looking for books on Passenger Jets (Boeing,Concord), Please suggest me the books I could buy and also preferably the Authors. ASAP --Ravi. From kls Thu Dec 10 02:09:23 1992 Newsgroups: sci.aeronautics.airliners Path: news From: gregory@halcyon.com (Gregory Wright) Subject: Re: 757 highest thrust to weight ratio ? X-Submission-Date: Thu, 10 Dec 1992 06:01:45 GMT References: Message-ID: Approved: kls@ohare.Chicago.COM Organization: The 23:00 News and Mail Service Sender: kls@ohare.Chicago.COM X-Submission-Message-ID: <1992Dec10.060145.1919@nwnexus.WA.COM> Date: 10 Dec 92 02:09:23 PST In article fredch@phx.mcd.mot.com (Fred Christiansen) writes: >In article kls@chicago.com (Karl Swartz) writes: >>Not surprisingly, the top 11 were all twins. > >Why "not surprisingly"? As a layman with no background in this stuff, >I would have tho't that manufacturers would keep the thrust-to-weight >in some ballpark range (for economic reasons). And that the advent >of larger twins is because more powerful engines have become available, >eliminating the need for a 3rd (or in the case of, say, MD-10 class >aircraft, the need for a 4th). >-- The reason for the this may, as you say, may not be so obvious so alow me to put in my two cents. The FAA, along with the CAA and others, require that we demonstrate a minimum climb out gradient with one engine inoperative. In the case of a two engine aircraft this requires that the airplane be able to climb out with a 50% reduction in gross thrust. For a four engine platform this one engine out condition represents only a 25% reduction in thrust. You can see then, that for a two engine plane it is required to "over" engine the plane... Hope this helps to clear things up, greg HIgh lift development >Fred Christiansen, Motorola, 2900 S Diablo Way, Tempe, AZ 85282 "Canajun, eh?" >fredch@phx.mcd.mot.com || uunet!phx.mcd.mot.com!fredch || +1 602-438-3464 > ".. I have set before you Life and Death, blessing and cursing; therefore > choose Life, that both you and your children may live" Deut 30:19 > -- ________Greg Wright____________ "Sure my YMP is fast, but if it doesn't | gregory@bcstec.boeing.com | run OS2 I don't need it....." | gregory@halcyon.com | |____uunet!bcstec!gregory_______| NOT A BOEING SPOKESPERSON. From kls Thu Dec 10 02:09:25 1992 Newsgroups: sci.aeronautics.airliners Path: news From: hoyme@src.honeywell.com (Ken Hoyme) Subject: Northwest cancels Airbus X-Submission-Date: Thu, 10 Dec 92 01:32:05 CST References: Message-ID: Approved: kls@ohare.Chicago.COM X-Submission-Message-Id: <9212100732.AA12591@darkstar.src.honeywell.com> Sender: kls@ohare.Chicago.COM Date: 10 Dec 92 02:09:25 PST In article mumble mumble driscoll@src.honeywell.com (Kevin Driscoll) writes: KH>With these cancellations, it is suspected that the bases will not be KH>built, and NWA will fly their planes to Europe for C/D checks. Of KH>course, they already have received and spent the operating loan.... RD> It'll be interesting to see what ramifications this has on the UAL deal: RD>> did United plan on using Airbus/NWA North American facilities? KD> A local (Minneapolis) TV story said that NWA is considering other KD> airlines' use of the facilities in deciding when/if to build. I seem to recall that at the time of the United announcement, the Minneapolis Star-Tribune business section specifically addressed the issue of whether United would use NWA's proposed Airbus bases in Minnesota. I recall a United spokesperson saying that United planned to do their own maintenance. NWA has always held out the possibility of contracting out to other airlines as the economic justification for the security of the state taxpayer's money. America West is one possibility. Airlines in Canada are another. (Heck - they might even think northern Minnesota is warm!! :^|) It was only in the recent announcement that I heard that they also planned to maintain some of their Boeing planes in these new bases. I don't know what this means, since they seem to have adequate facilities at MSP for their existing equipment. Maybe that land has more value as something else (like more gates...). Ken Ken Hoyme Honeywell Systems and Research Center (612)951-7354 3660 Technology Dr., Minneapolis, MN 55418 Internet: hoyme@src.honeywell.com From kls Thu Dec 10 16:07:03 1992 Newsgroups: sci.aeronautics.airliners,rec.travel.air Path: news From: rdd@cactus.org (Robert Dorsett) Subject: Re: Airbus safety (was Re: TWAs Status) X-Submission-Date: Thu, 10 Dec 92 5:16:04 CST References: <1992Nov25.191925.27991@news.mentorg.com> <8762@lee.SEAS.UCLA.EDU> <1992Dec01.173212.27936@news.mentorg.com> Message-ID: Approved: kls@ohare.Chicago.COM Followup-To: sci.aeronautics.airliners Sender: kls@ohare.Chicago.COM X-Submission-Message-Id: Date: 10 Dec 92 16:07:03 PST In article <1992Dec01.173212.27936@news.mentorg.com> philip@mentorg.com (Philip Peake) writes: > >It wasn't unintentional - it was a deliberately (contrived) example. >The arguments I have heard so far seem to say that just because its always >been done that way, it always should be - aircraft design has changed a LOT >since the stick control was introduced - maybe this is no longer the >correct control mechanisim ? Transport aircraft design hasn't changed much at all in the last 30 years. We fine-tune various features, change the aspect ratio, develop better drag profiles, better powerplants, occasionally build a better, lighter system. Certainly improved manufacturing techniques. But the *engineering* discipline is so WELL defined that if you give three manufacturers the task of developing three different airplanes for the same mission profile, you'll now come up with almost identical airplanes. It is a discipline so evolved that we can come up with physical implementations which can match design performance objectives to within a percentage point. This is not the result of "wild-catting," or breaking the rules: it's the result of decades of working over the same problem, developing a very intimate understanding of this particular type of development problem. We should expect that the same considerations must be applied to how the pilots control the airplane. The "old" model may not be the best available, but it's well-understood, and is likely preferable to any "replacement" we are likely to produce with current technology. >If you are against the idea of insulating the pilot, maybe we should >remove servo brakes and power steering from cars too ? The pilot IS in the loop. You can complain about that, and try to eliminate that, if you want to. However, since he IS in the loop, the unique feedback requirements needed to let him do his job require a more interactive environ- ment than either contemporary glass cockpits *or*, in this case, the A320 sidestick, provide. Christopher Davis already addressed your point in his reply: *hydraulics* is the equivalent of power steering, not FBW control. However, note that we've been providing completely artificial feel to go along with this, for the past thirty years. Yet all of a sudden, on the pretext that the "FBW" in their airplane mandates it, Airbus, which is in the business of selling technology, cavalierly introduces a control device which: 1. Has no interconnect between the pilots. 2. Has no active feedback. 3. Utilizes artificial control laws in the normal and alternate flight modes. I suggest that the issue has NOTHING to do with technological "advantages" human requirements: it is completely marketing-driven. >|> In essence, my point is that standards don't exist because of happenstance. >|> They exist because it makes life easier for everyone. This is particularly >|> important when human lives are at stake. > >Standards are also perpetuated by vested interests, Yeah, that powerful yoke-manufacturer lobby. The bastards. Just because they won't retool to build sidesticks, they gotta ruin it for the rest of us. :-) Seriously, this is a tremendously conservative industry. What isn't broken, doesn't get fixed. However, when a better mouse-trap is invented, it is almost always adopted, universally. The fact that no other manufacturer is rushing to repeat Airbus' example suggests the arbitrariness of the use of the sidesticks: if there were even minor operational or material advantages in using them (and modified control laws) as interfaces to the EFCS, you could bet your last dollar every other manufacturer would be doing so, not least as the result of airline demand. We don't see that. > even when better ideas >ar around. This isn't one of them. We aren't operating in a vacuum: NASA, as one example, has been running a lot of research (over, and over) over the last 20 years, addressing precisely these issues: the Airbus implementation is arguably on the weaker of a variety of choices available. >If all new pilots were taught nothing but the side stick, >how long would the old arangementy last - and if the old arangement Why should pilots be taught nothing but a unique, *proprietary* side-stick design that no pilot had any experience with before four years ago, and which is only one of a variety of other possible designs? You imply that the sidestick's just a yoke wrapped up in a little handle. It isn't: the issue's a lot more complex, and, within that simple interface, there are *many* ways to proceed. The certification authorities, you will note, have not codified mandatory control qualities of this interface (and WON'T): thus, in a worst-case, we could have Airbus running its stick (+ control laws), Boeing running its own, MDC running its own, etc. >is so wonderful, why do military fighter aircraft, where tight control >by the pilot os ESSENTIAL use side stick controls ? Not all do: several continue to use center-sticks. In either case, the issue is in large degree driven by the need to effectively control the aircraft at high g's--but even then, it's a significantly different design than that used in the A320. I would also note that in fighter aircraft, there isn't the issue of two-pilot "peers" having to quickly and instinctively figure out who is flying the airplane. On the A320, there is no interconnect between the sidesticks: the captain can command a full-left in an emergency evasive maneuver, the F/O full-right, and the net result will be an algebraically added "zero." >The problem is the PILOTS, not the design Here we flip to cockpit integration, not sidesticks. The problem is a design philosophy which is unwilling to accomodate the human element. I also see a great deal of "stick it to the pilots" going on: a number of proponents of pilot-isolation don't even bother to cite alleged economic or safety benefits, anymore: the pilot-isolation increasingly appears to be an engineering-driven goal in itself. The Airbus approach has gone too far. Thankfully, however, it seems to be on its way out: new designs, such as the 777, are more sophisticated, yet have more conventional and interactive interfaces. And the research community is coming squarely on the side of more interactive, appropriate feedback. New designs will be more human-factors-driven, not engineering- driven. And, with luck, we'll see a return to the *evolutionary* application of high technology, rather than the *revolutionary* application of the same. And who knows, in 20 years, when we have enough underlying experience and research under our belts, we can try a *standardized* alternate interface. BTW, and for the record, I *like* the idea of sidesticks: for no other reason than to be able to see the entire instrument panel, unencumbered. I simply don't like this particular implementation, and have concerns about the human requirements any sidestick design could introduce. >Philip --- Robert Dorsett rdd@cactus.org ...cs.utexas.edu!cactus.org!rdd From kls Thu Dec 10 16:07:06 1992 Newsgroups: sci.aeronautics.airliners Path: news From: rdd@cactus.org (Robert Dorsett) Subject: Re: Flight controls X-Submission-Date: Thu, 10 Dec 92 5:22:58 CST References: <1992Nov26.000453.4729@cactus.org> <1992Dec01.025604.17493@news.mentorg.com> airliners.1992.77@ohare.Chicago.COM> Message-ID: Approved: kls@ohare.Chicago.COM Sender: kls@ohare.Chicago.COM X-Submission-Message-Id: Date: 10 Dec 92 16:07:06 PST In philip@rainbow.mentorg.com (Philip Peake) wrote: > In article , rdd@cactus.org (Robert Dorsett > if you want to knock the A320, there are much better grounds for doing so > than ergonomics - without the more serious design problems, there would probably > have been many fewer "accidents", and hence less reason to blame the ergonomics. I am hard pressed to think of many other things. Structurally, the A320 is extremely conservative, highly conventional. In systems layout and design, highly conventional. There are a few frills, such as the cabin lighting system, toilets, or window heat, which have been "automated," but only in relatively self-contained manners (toilet going out doesn't have the slightest ramification on ELAC 1 being able to do its job, for instance: they aren't on the same networks :-)). The EFCS, in turn, has been the focus of so much attention that at least one pundit suggested that other aspects may have been allowed to lapse, as evidenced by the initial problems with the toilets or the cabin intercom/ lighting system, the latter of which, in the words of a BA maintenance engineer, had software so simple "a child could have done it better." These aren't safety-critical items (well, maybe the lighting is: it didn't work at Habsheim). Two of the three accidents were misuses of the FMGS MCU; the other--the first--was so bizarre, such an outrageous case of poor airmanship, that I've yet to fully assess the implications. This therefore seems to call for better ergonomics or training, with the latter recognized as precisely what it is: a kludge, covering up poor design. It's important to note that while, on a quantifiable basis, the A320's EFCS is most subject to criticism, it's equally clear that, thus far, the EFCS has performed almost flawlessly. And even if it doesn't meet the 1-in-a-billion failure rate, it's likely that if it produces even one EFCS-induced catastrophic failure every 10 years, the human and material costs can be easily absorbed by the industry--and when it does fail, we probably wouldn't be able to determine what happens, since the DFDR certainly doesn't record the myriad execution paths. The real issue, of course, is whether this is as safe as a conventional system. And if it isn't, there are tremendous ethical and moral issues at play. > Besides "cosmetic" issues like tactile feedback, and some layout issues, This isn't cosmetic. The choice of using sidesticks, the four major flight control modes, the many possible permutations within those modes, are part of a highly integrated *system* design. If one looks at it for itself, it's a very "sexy" design, a startingly coherent design philosophy. How well it adapts to the real world is another issue, entirely, of course. I would even suggest that if one disqualifies one aspect of this model: sidestick, throttle control, switch design--the totality could suffer irreparable damage. None of this is "cosmetic." It's the heart of how the airplane is controlled. > the 767 > is pretty close to an A320 - as you have said (I think - sorry if I misquote > you) > the 767 is just more conventional in cockpit design - its a pity its automatic > landing system can be as good as the best pilot on a good day, and a rough as > the worst on a bad day ... usually more towards the latter ... I would not have rated the airplanes as equivalents. The 767 is "equivalent" to an A310, but even then, there are significant differences in cockpit design. If I've given the impression of "equivalency," it was by mistake: perhaps in avionics maintenance practices, or the A320 or 747-400 as "consumers" of the benefits of the 767/A310 learning curve; little else. The airplane I'd compare with the A320 is the 747-400, at least in cockpit design, systems design, and AIDS/BITE integration; certainly not the mission requirements. > ) writes: > |> We can automate easily quantifiable issues: simple tasks. Judgement and > |> airmanship has thus far evaded us, on all levels. Until we get a grip on > |> it, talk of fully autonomous aircraft or ground control is nothing more > |> than science fiction. > > [...] history, even modern history is littered with comments from > people writing off things as "science fiction", "can't be done", "will never rep > lace > the current ...." etc who have had to eat their words shortly after. In the software engineering community, words like "Oh, that's easy," or "I can do that on time, on schedule, and under budget" are *always* eaten, later on. Software is an art, not an engineering discipline. I wonder what the aero manufacturers are doing that the rest of us poor sods aren't, that let them miraculously produce highly complex packages right on schedule, in a certification environment in which even a day's delay can costs millions of dollars. 10M of code in an A330/340, indeed. I have a hard enough time keeping my little 1M Microsoft Word in line. Usually, when I write "stupid" things, I regret it an hour later. It's been over 72 hours, now, and I stand by my words. At this point in time, it is not feasible to create fully autonomous transport aircraft, as implied in the original article. By the time it is, I expect my bones to be dust. Incidentally, a few people seem to have interpreted my comments about software engineering as coming from an AE perspective: they weren't. I'm not sanguine about CS types writing this stuff: I simply don't think development technology's at a point where we can write reliable software with the level of confidence I feel is necessary. This is a whole other discussion, though. I actually have little experience with the capabilities of AE-types to write code. Although I suppose if they had done it, the EFCS would have been written in FORTRAN, not C/Pascal/assembly. :-) I will concede that the CS approach is likely the lesser of two evils. --- Robert Dorsett rdd@cactus.org ...cs.utexas.edu!cactus.org!rdd From kls Thu Dec 10 16:07:07 1992 Newsgroups: sci.aeronautics.airliners Path: news From: greg@saltydog.dpsi.com (Gregory R. Travis) Subject: Re: 757 highest thrust to weight ratio ? X-Submission-Date: Thu, 10 Dec 1992 14:28:02 GMT References: Message-ID: Approved: kls@ohare.Chicago.COM Organization: Data Parallel Systems, Inc X-Submission-Message-ID: Sender: kls@ohare.Chicago.COM Date: 10 Dec 92 16:07:07 PST Karl makes some excellent points regarding the reasons twins tend to be overpowered compared to 3 and 4 engined jets. I would just like to add that FAA certification requires not only that an airplane be able to finish takeoff, return, and land with AN engine out, but that it also meet certain climb performance criteria during a single-engine climb. Karl is absolutely right that, on a 747 that loses one, the remaining three need to make up, individually, much less of the lost thrust. Whereas on a 757 the remaining engine must make up ALL of the lost thrust. However, I am confused by Karl's statement that Lufthansa chose the A340 over the A330 because of concerns that the twin-engined A330 would place more stress on its two engines whereas the A340 would enjoy higher engine reliability as its four engines loafed along. Since a twin engine jet is nominally overpowered compared to a four engine jet, it should be able to operate, on aggregate, at a lower thrust setting during takeoff or be operated at high thrust for a shorter overall climb. Since a four-engine jet has all four operating closer to the margin, in normal operation, the engines should suffer from higher demands all around. greg -- Gregory Reed Travis D P S I Data Parallel Systems Incorporated greg@dpsi.com (For MX mailers only!) Bloomington, IN greg@indiana.edu (For the others) From kls Thu Dec 10 16:07:11 1992 Newsgroups: sci.aeronautics.airliners Path: news From: pab@po.CWRU.Edu (Pete Babic) Subject: Re: hydraulic problems with DC-10's?? X-Submission-Date: 10 Dec 1992 14:36:36 GMT References: Message-ID: Approved: kls@ohare.Chicago.COM Reply-To: pab@po.CWRU.Edu (Pete Babic) Organization: Case Western Reserve University, Cleveland, OH (USA) Sender: kls@ohare.Chicago.COM X-Submission-Message-Id: <1g7klkINNagb@usenet.INS.CWRU.Edu> Date: 10 Dec 92 16:07:11 PST In a previous article, rdd@rascal.ics.utexas.edu (Robert Dorsett) says: > >The slat retraction DID affect the wing: it then became both a control and >aerodynamic problem. Exercising established control practices in an >unknown aerodynamic regime crashed the airplane (I'd love to know whether >this went into Airbus's "pilot error" database :-)). Had the slats remained >down, the airplane would have survived the engine failure, even with the >failure of the stall warning system. Other airframe manufacturers have >manual locking mechanisms for their slat jackscrews. McDonnell Douglas >relied on hydraulic pressure to hold it all together. > >Incidentally, this problem wasn't corrected: the SUX DC-10 also experienced >extension of its slats after it lost all its hydraulics. Does anyone know if the MD-11 has a proper locking mechanism for the slats? I'm a layman when it comes to aircraft design, but the DC-10 really looks like a substandard design that has killed a bunch of people due to cost cutting short cuts. -- Pete Babic - pab@po.cwru.edu /// LIVE TO PARTY, SKI TO DIE!!! /// /\ Member of ACE \\\ /// /--\MIGA (American Coaster Enthusiasts) \\\/// MS-DOS or a Mac? What's that? From kls Thu Dec 10 16:07:13 1992 Newsgroups: sci.aeronautics.airliners Path: news From: greg@saltydog.dpsi.com (Gregory R. Travis) Subject: Re: hydraulic problems with DC-10s?? X-Submission-Date: Thu, 10 Dec 1992 14:44:27 GMT References: Message-ID: Approved: kls@ohare.Chicago.COM Organization: Data Parallel Systems, Inc X-Submission-Message-ID: Sender: kls@ohare.Chicago.COM Date: 10 Dec 92 16:07:13 PST In barr@ash.mmm.ucar.EDU (Keith Barr) writes: >greg@octopus.dpsi.com (Gregory R. Travis) writes: >> BTW, just to be pedantic: The wings do not each contribute exactly >> 50% of the total lift. Remember that that fuselage itself contributes >> a SUBSTANTIAL amount of lift at cruise as do the horizontal stabilizer >> surfaces (in certain flight regimes!). >What flight regimes might those be? Unstable flight, as in the F-16? >Actually, in most aircraft the horizontal stabilizer is downlifting, so >the wings have to create more lift, and the body contributes to the >pitching moment, but contributes very little lift (unless we are talking >about the B-2). As I understand it (and I'm not an aero geek, just an historian geek :-)) there are flight regimes in which the horizontal stabilizer on conventional (i.e. dynamically stable) aircraft contributes POSITIVE lift, even when said aircraft is loaded within acceptable C.G. ranges. It is possible (and likely) during slow flight for the center of lift to move forward of the center of gravity on the wing. At such time, the horizontal stabilizer becomes a LIFTING surface, restoring normal stability to the aircraft and the pilot's never the wiser. If you look at many horizontal stabilizers, especially on small GA aircraft, you'll note that the airfoil is completely symmetric top and bottom. If the horizontal stabilizer contributed ONLY to negative lift, one would think it more efficient to design it as an upside-down wing and shorten its overall span. I've talked to a couple of Cessna engineers about this (when I was completely incredulous that it actually happened) and they agreed that the horizontal stabilizer does contribute positive lift and that Cessna routinely measured this lift with strain gauges when evaluating stabilizer structure. The lift is not present only during extreme maneuvering (such as a violent pitch-down) but also during straight and level flight. My good friend Moshe Braner (braner@emily.emba.uvm.edu) can give you a much better technical explanation of the phenomena and, no doubt, correct all my mis-explanations and errors. greg -- Gregory Reed Travis D P S I Data Parallel Systems Incorporated greg@dpsi.com (For MX mailers only!) Bloomington, IN greg@indiana.edu (For the others) From kls Thu Dec 10 16:07:15 1992 Newsgroups: sci.aeronautics.airliners Path: news From: hfunk@src.honeywell.com (Harry Funk) Subject: Re: New Scientist article X-Submission-Date: Thu, 10 Dec 1992 15:13:01 GMT References: Message-ID: Approved: kls@ohare.Chicago.COM Reply-To: hfunk@src.honeywell.com (Harry Funk) Organization: Honeywell Systems & Research Center Sender: kls@ohare.Chicago.COM X-Submission-Message-Id: <1992Dec10.151301.10339@src.honeywell.com> Date: 10 Dec 92 16:07:15 PST In article dmarble@magnus.acs.ohio-state.edu (Duane F Marble) writes: >A small point with respect to the material quoted from the New >Scientist: Global Positioning System (GPS) birds do not "observe" >anything, they just permit a ground based unit to compute it's >location. My guess is that he was referring to Automatic Dependent Surveillance (ADS) systems based on SATCOM, which has even less to to with GPS/Glonass birds. The planes [would] communicate their positions by means of a satellite link to a ground-based tracking/control system. United currently has a few 747-400's that are so equipped. The major benefit envisioned is for oceanic routes, where the fixed spacing (slots) system currently used results in suboptimal tracks for a number of users of the system. ADS is the successor to the Oceanic Display and Planning System (ODAPS), which I think is currently installed at the Oakland and NY Air Route Traffic Control Centers (ARTCCs). -- Harry A. Funk Principal Research Scientist Voice: (612)-782-7396 Honeywell Systems and Research Center FAX: (612)-782-7438 3660 Technology Dr. MS:MN65-2500 Inet: HFunk@src.honeywell.com Minneapolis, MN 55418 Bang: !srcsip!funk From kls Thu Dec 10 16:07:18 1992 Newsgroups: sci.aeronautics.airliners Path: news From: ckd@eff.org (Christopher Davis) Subject: Re: objects on wing tips X-Submission-Date: Thu, 10 Dec 1992 17:28:58 GMT References: Message-ID: Approved: kls@ohare.Chicago.COM Organization: Electronic Frontier Foundation Tech Central Sender: kls@ohare.Chicago.COM X-Submission-Message-Id: Date: 10 Dec 92 16:07:18 PST RD> == Ramin Dowlati (Um, your software's broken.... ^^^^^^^^^^^^) RD> Several years ago, the aeropspace industry introduced vertical RD> flap-like things on the ends of their airplane wings. I've only RD> noticed these on 'larger' planes such Airbus, 747-400 and MD-11. They're on smaller aircraft as well. The Canadair Challenger bizjet and its big brother, the Canadair Regional Jet (which makes this article about airliners :) have them, for example. (Side note: ComAir (The Delta Connection) has a HUGE model of the RJ in their gate area at CVG. [We had a long (2 hrs) connection on Delta, I was bored, so after having some chili I walked all the way down the concourse.]) -- Christopher K. Davis | ``Usenet seems to run much like the Kif (or, EFF #14 | for the TV generation, Klingon) high command. System Administrator, EFF | Whoever takes action and can be heard wins.'' +1 617 864 0665 [CKD1] | --Peter da Silva From kls Thu Dec 10 16:07:19 1992 Newsgroups: sci.aeronautics.airliners Path: news From: rbarnick@mitre.org (Barnick, R.) Subject: 737 Crash In Colorado Springs X-Submission-Date: Thu, 10 Dec 1992 20:05:35 GMT Message-ID: Approved: kls@ohare.Chicago.COM Followup-To: sci.aeronautics.airliners Organization: MITRE Sender: kls@ohare.Chicago.COM X-Submission-Message-Id: Date: 10 Dec 92 16:07:19 PST In March 1991 a UAL 737 went down on final approach into COS. The final accident investigation report was released a couple of days ago and carried no cause. About four months ago a TWA flight leaving COS picked up a federal investigator working the accident. The investigator sat up front in the jump seat. The TWA flight deck crew wanted to know how the investigation was going. The following was relayed to me via one of the flight crew members. UAL recently lost some kind of case from its female employees which charged gender discrimination. UAL either lost or agreed to settle out of court. UAL agreed to correct conditions which caused the suit. One correction was to get more females in the cockpit. The ill-fated 737 had a very junior female first officer. The pilot was male and also junior. Cockpit voice recordings (never yet fully released to public) indicated that when the crew was informed of very gusty wind conditions in the COS area, they seemed overly concerned. They kept commenting about the wind. These frequent comments seemed irregular to the investigators. Experienced pilots wouldn't have spent so much time discussing the wind. On final it was speculated the aircraft took a good jolt from an air current or eddy. The crew, having psyched themselves up substantially, reacted in some uncoordinated knee-jerk fashion resulting in the fatal error. The investigator went on to say that even if this sort of story could be proven, it would never be made public. To do would discredit UAL's gender action, UAL's training, FAA's certification, and maybe further hurt an already hurting industry. It seems odd that the final accident report that came out indeed did say nothing. Please remember, this is a story told me second hand. You're getting it third hand. But, if any truth about this crash is known, a sharing thought would be interesting. From kls Thu Dec 10 18:07:49 1992 Newsgroups: sci.aeronautics.airliners Path: news From: kls@ohare.Chicago.COM (Karl Swartz) Subject: Re: Flight controls X-Submission-Date: Fri, 11 Dec 1992 01:40:40 GMT References: Message-ID: Approved: kls@ohare.Chicago.COM Organization: Chicago Software Works Sender: kls@ohare.Chicago.COM X-Submission-Message-ID: <1992Dec11.014040.5031@ohare.Chicago.COM> Date: 10 Dec 92 18:07:49 PST In article rdd@cactus.org (Robert Dorsett) writes: >The EFCS, in turn, has been the focus of so much attention that at least one >pundit suggested that other aspects may have been allowed to lapse, as >evidenced by the initial problems with the toilets or the cabin intercom/ >lighting system, the latter of which, in the words of a BA maintenance >engineer, had software so simple "a child could have done it better." These >aren't safety-critical items (well, maybe the lighting is: it didn't work at >Habsheim). I believe both intercom and lighting are considered safety-critical items. Not long ago, I sat for half an hour at O'Hare with a United 747 while they fixed the intercom. They considered this critial -- justifiably so, IMO -- because it would be necessary for instructing passengers in the event of an emergency. Lighting in general may not be deemed critical, though certainly the directional lighting in the floor is. >I would not have rated the airplanes as equivalents. The 767 is "equivalent" >to an A310, but even then, there are significant differences in cockpit >design. If I've given the impression of "equivalency," it was by mistake: >perhaps in avionics maintenance practices, or the A320 or 747-400 as >"consumers" of the benefits of the 767/A310 learning curve; little else. The >airplane I'd compare with the A320 is the 747-400, at least in cockpit design, Is the MD-11 comparable to the 747-400 in this regard? I would assume so since they are of comparable vintage. Where do the new generation 737s (-300/-400/-500) fit into this? With the first flight of the 737-300 coming several years after the 767 came on the scene I would expect it to be comparable, yet some of what I've seen suggests it is less sophisticated. I've been in the cockpits of each (well, 757s, not 767s that I recall) but really don't know enough about what I'm looking at to tell the difference. And, for completeness, where do the glass-cockpit version of the MD-80 family fit into the picture? -- Karl Swartz |INet kls@ditka.chicago.com 1-415/854-3409 |UUCP uunet!decwrl!ditka!kls |Snail 2144 Sand Hill Rd., Menlo Park CA 94025, USA Send sci.aeronautics.airliners submissions to airliners@chicago.com From kls Thu Dec 10 18:07:52 1992 Newsgroups: sci.aeronautics.airliners Path: news From: kls@ohare.Chicago.COM (Karl Swartz) Subject: Re: 737 Crash In Colorado Springs X-Submission-Date: Fri, 11 Dec 1992 01:59:27 GMT References: Message-ID: Approved: kls@ohare.Chicago.COM Organization: Chicago Software Works Sender: kls@ohare.Chicago.COM X-Submission-Message-ID: <1992Dec11.015927.5148@ohare.Chicago.COM> Date: 10 Dec 92 18:07:52 PST In article rbarnick@mitre.org (Barnick, R.) writes: >In March 1991 a UAL 737 went down on final approach into COS. The >final accident investigation report was released a couple of days ago >and carried no cause. Just to complete the record, it was UA 585 (DEN-COS) on March 3, 1991; 5 crew and 20 passengers were killed. The aircraft was N999UA, msn 22742, ln 875, a 737-291 (Advanced). It was acquired from Frontier in May, 1986, the last of 25 such aircraft. (Two were 737-2A1(A) models.) >The following was relayed to me via one of the flight crew members. Fascinating. Most of the speculation has been that the aircraft flew into the eye of a rotor (much has been said about this on rec.aviation by folks from the area) which proceeded to flip it. Another story I heard, from some United folks, was that there was some difference in the rudder controls betwee these planes and United's 737-222s, and that this in some way contributed to the crash. I can't recall the details, including whether it was a difference between the Advanced and non-Advanced 737-200s, or an airline-related change that United had not yet applied to the ex-Frontier aircraft. >The investigator went on to say that even if this sort of story could be >proven, it would never be made public. To do would discredit UAL's gender >action, UAL's training, FAA's certification, and maybe further hurt an >already hurting industry. This seems reasonable, and I've heard the same logic applied to snuffing a report that allegedly recommended the immediate grounding of Continental (c. 1989) on grounds of inadequate maintenance. But the CVR from the Air Florida 737 crash into the Potomac in Washington certainly made those pilots look incompetent, and the United DC-8 that went down in Portland in 1978 didn't seem much better. Indeed, the latter, as I understand it, motivated a *major* revision of United's training program. Perhaps gender-discrimination is a more sensitive issue, though. >It seems odd that the final accident report that came out indeed did say >nothing. I believe this is unique insofar as the NTSB is concerned. Robert, is this indeed true? >Please remember, this is a story told me second hand. You're getting it >third hand. But, if any truth about this crash is known, a sharing thought >would be interesting. Thanks for sharing it. If anybody else has anything substantial to contribute on the matter, please send it in. -- Karl Swartz |INet kls@ditka.chicago.com 1-415/854-3409 |UUCP uunet!decwrl!ditka!kls |Snail 2144 Sand Hill Rd., Menlo Park CA 94025, USA Send sci.aeronautics.airliners submissions to airliners@chicago.com From kls Thu Dec 10 20:58:52 1992 Newsgroups: sci.aeronautics.airliners Path: news From: Robert Dorsett Subject: MD-11 (Re: hydraulic problems on DC-10) X-Submission-Date: Thu, 10 Dec 92 20:47:23 CST References: Message-ID: Approved: kls@ohare.Chicago.COM Reply-To: rdd@rascal.ics.utexas.edu Sender: kls@ohare.Chicago.COM X-Submission-Message-Id: Date: 10 Dec 92 20:58:52 PST In article pab@po.CWRU.Edu (Pete Babic) writes: >Does anyone know if the MD-11 has a proper locking mechanism for the slats? >I'm a layman when it comes to aircraft design, but the DC-10 really looks >like a substandard design that has killed a bunch of people due to cost >cutting short cuts. I had wondered about this, and researched it about a year ago. I wasn't able to find a clear-cut answer. Otherwise, my conclusion is that the MD-11 is a very marginal upgrade of the basic DC-10 design. The systems layout is almost identical; none of the major "complaint areas" have changed. There's a high degree of commonality between the DC-10 and MD-11, the "hydraulics plug" adopted after the SUX crash being a good example. The changes incorporate a 6-meter fuselage stretch, the winglets, composites in the tail, new engines, and a new cockpit. The latter appears to be the most radical change, but other than that, what characterizes the industry media is a lot of manufacturer "gee whiz" propaganda, long on "radical changes," but short on specifics. I recall an Av Leak article a couple of years ago, which suggested the full new type-certification wasn't necessary, but McDonnell-Douglas did it anyway, to try to exorcise itself from the political "ghosts" of the DC-10: a regulatory face-lift, if you will. Problem is, not much has changed. So: I see no objective reason to conclude the MD-11 is any "safer" than the DC-10, if one accepts the existence of the problems that characterized the DC-10's development. More detailed information would be welcome. I was really appalled by how little "hard" info was out there, and I spent quite a bit of time on it. Literally hundreds of articles on 767-X and 777 aerodynamic and systems development, dozens on Airbus, but only two or three "skimpy" treaments on the MD-11, the kind of things you'd find in "Aerospace America." Does MDC have some kind of anti-publishing policy or something? -- Robert Dorsett Internet: rdd@rascal.ics.utexas.edu UUCP: ...cs.utexas.edu!rascal.ics.utexas.edu!rdd From kls Fri Dec 11 00:38:19 1992 Newsgroups: sci.aeronautics.airliners Path: news From: cid@athena.mit.edu (Derek H Cedillo) Subject: Re: 757 highest thrust to weight ratio ? X-Submission-Date: Fri, 11 Dec 1992 06:16:27 GMT References: Message-ID: Approved: kls@ohare.Chicago.COM Organization: Massachusetts Institute of Technology Sender: kls@ohare.Chicago.COM X-Submission-Message-Id: <1992Dec11.061627.6973@athena.mit.edu> Date: 11 Dec 92 00:38:19 PST In article greg@saltydog.dpsi.com (Gregory R. Travis) writes: [good outline of engine loading] >However, I am confused by Karl's statement that Lufthansa chose the A340 >over the A330 because of concerns that the twin-engined A330 would place more >stress on its two engines whereas the A340 would enjoy higher engine >reliability as its four engines loafed along. > >Since a twin engine jet is nominally overpowered compared to a four engine >jet, it should be able to operate, on aggregate, at a lower thrust setting >during takeoff or be operated at high thrust for a shorter overall climb. > >Since a four-engine jet has all four operating closer to the margin, in >normal operation, the engines should suffer from higher demands all around. I think the thing here, is that you are imagining two different engines entirely. This isnt exactly the case. I dont have thrust data handy, so I cant compare the A330 engine performance with the A340, but as a quick example, I'd like to point out that the A340, A320 and A321 all have the same GE engine spec (CFM56-5) while the A340 is a four engine plane and the A320 and 21 are two engine planes. Althought the 340 is definately bigger than the 320/1, I dont think the engine loading would be twice as much. (can someone come up with stats to prove or disprove please) Another look is the 767-200ER/300/300ER, MD-11 and 747-200/300/400 which can choose the GE CF6-80C2. All are relatively large planes, and they gradually increase in size, but is it huge enough to say the 747 is Twice as heavy/aero dyn loaded, etc, to require twice the thrust as the 767? Note that they have 2,3,4 engines respectively Again, I would appreciate any airframe data to support the weight/thrust/loading claim, or shoot me out of the sky as seen fit. Can someone help? Thanks, Derek ------------------------------- "He lived a life of going-to-do, and died with nothing done" -J. Albery In other words---JUST DO IT! From kls Fri Dec 11 03:35:19 1992 Newsgroups: sci.aeronautics.airliners Path: news From: lhe@sics.se (Lars-Henrik Eriksson) Subject: Re: 737 Crash In Colorado Springs X-Submission-Date: Fri, 11 Dec 1992 08:41:57 GMT References: Message-ID: Approved: kls@ohare.Chicago.COM Followup-To: sci.aeronautics.airliners Organization: Swedish Institute of Computer Science, Kista Sender: kls@ohare.Chicago.COM X-Submission-Message-Id: Date: 11 Dec 92 03:35:19 PST In article rbarnick@mitre.org (Barnick, R.) writes: UAL recently lost some kind of case from its female employees which charged gender discrimination. UAL either lost or agreed to settle out of court. UAL agreed to correct conditions which caused the suit. One correction was to get more females in the cockpit. The ill-fated 737 had a very junior female first officer. The pilot was male and also junior. Please remember, this is a story told me second hand. You're getting it third hand. But, if any truth about this crash is known, a sharing thought would be interesting. I suspect that many people - particularly people not familiar with aircraft operations - would believe that the problem was that UAL had been "forced" to put a "a very junior female first officer" on the flight. (We must assume that the F/O was had sufficient qualifications, even if she was quite new - every pilot is very junior in the beginning of their careers). That was not the problem at all - it was UALs decision to put a junior captain on the flight together with a very junior first officer. -- Lars-Henrik Eriksson Internet: lhe@sics.se Swedish Institute of Computer Science Phone (intn'l): +46 8 752 15 09 Box 1263 Telefon (nat'l): 08 - 752 15 09 S-164 28 KISTA, SWEDEN Fax: +46 8 751 72 30 From kls Fri Dec 11 03:35:21 1992 Newsgroups: sci.aeronautics.airliners Path: news From: kls@ohare.Chicago.COM (Karl Swartz) Subject: Re: 757 highest thrust to weight ratio ? X-Submission-Date: Fri, 11 Dec 1992 11:33:35 GMT References: Message-ID: Approved: kls@ohare.Chicago.COM Organization: Chicago Software Works X-Submission-Message-ID: <1992Dec11.113335.6455@ohare.Chicago.COM> Sender: kls@ohare.Chicago.COM Date: 11 Dec 92 03:35:21 PST In response to Derek H Cedillo's reply to Gregory R. Travis' reply to my post ... Greg sez ... However, I am confused by Karl's statement that Lufthansa chose the A340 over the A330 because of concerns that the twin-engined A330 would place more stress on its two engines whereas the A340 would enjoy higher engine reliability as its four engines loafed along. Just to make it clear, this was a rather fuzzy memory. I believe I read the details, which I may have distorted badly, in AW&ST, but cannot place it better than that. At the time, it kinda made sense to me, but I'm not sure it does now. If anyone can fill in the missing details I would be most appreciative. Greg continues ... Since a twin engine jet is nominally overpowered compared to a four engine jet, it should be able to operate, on aggregate, at a lower thrust setting during takeoff or be operated at high thrust for a shorter overall climb. Since a four-engine jet has all four operating closer to the margin, in normal operation, the engines should suffer from higher demands all around. and Derek replies ... I think the thing here, is that you are imagining two different engines entirely. This isnt exactly the case. Eh? Don't you have that backwards? Since an A330 is little more than an A340 with two big engines instead of four little ones, it's very much the case that the engines are entirely different. And this may well be the case -- at least by some metrics, a 67,500 lb. thrust engine endures more stress than a 31,200 lb. thrust engine. Of course it depends on the core from which one started, and a lot of other factors. Or perhaps Lufthansa feels the CFM56 is inherently more reliably than any of the engine options for the A330 (CF6-80 first, then PW4000, and RR Trent and GE 90 options later). Not that there's anything *bad* about any of the larger engines, but the CFM56 has the best record of any of the larger jet engines if I'm not mistaken. Derek continues ... I dont have thrust data handy, so I cant compare the A330 engine performance with the A340, but as a quick example, I'd like to point out that the A340, A320 and A321 all have the same GE engine spec (CFM56-5) while the A340 is a four engine plane and the A320 and 21 are two engine planes. It's actually a CFM International (GE and Snecma are equal partners, I believe) engine, and the -5 simply means its for an Airbus as far as I can tell. In its various incarnations a CFM56 ranges from 20,000 lbs. thrust up to 34,000 lbs. The A320 uses a -5A2 (25,000 lbs.) or -5A3 (26,500 lbs.) version, while the initial A340 version uses a -5C2 (31,200 lbs.). Althought the 340 is definately bigger than the 320/1, I dont think the engine loading would be twice as much. (can someone come up with stats to prove or disprove please) I'm not sure this will answer the question or not, but I dug out the power/weight specs I gathered earlier, added numbers for the A330/A340, and added a column for power/weight ratio with one engine out. Here's what I came up with: model pass range MGTOW engines thrust p/wt 1out ----- ---- ----- ----- ------- ------ ---- ---- A320-200 140-179 ? 162 2 CFM56-5A3 26500 0.3272 0.1636 A321-100 180-220 ? 181.2 2 CFM56-5B2 31000 0.3422 0.1711 A330 280-440 ? 467.5 2 CF6-80E1A2 67500 0.2888 0.1444 A340-200 220-440 ? 558.8 4 CFM56-5C2 31200 0.2233 0.1675 A340-300 280-440 ? 558.9 4 CFM56-5C2 31200 0.2233 0.1675 The one-engine-out numbers are remarkably similar for the A320 and A340. Another look is the 767-200ER/300/300ER, MD-11 and 747-200/300/400 which can choose the GE CF6-80C2. All are relatively large planes, and they gradually increase in size, but is it huge enough to say the 747 is Twice as heavy/aero dyn loaded, etc, to require twice the thrust as the 767? Well, sucking a few more figures out of my files: model pass range MGTOW engines thrust p/wt 1out ----- ---- ----- ----- ------- ------ ---- ---- 747-400 412-509 8380 870 4 PW4056 56000 0.2575 0.1931 767-300(ER) 204-290 6650 400 2 PW4060 60000 0.3000 0.1500 I picked these two particular airframe/engine combinations as the best comparison points but have more data if anybody wants it. In any case, yes, the 747 *is* twice as heavy, more than that in fact, but because it uses a slightly lesser rated engine has a bit less than twice the thrust. Again, I would appreciate any airframe data to support the weight/thrust/loading claim, or shoot me out of the sky as seen fit. Can someone help? There's some data, but I'm not understanding Lufthansa's position much better. Maybe it's just the late hour, or the flu I've been fighting off. How 'bout one of you folks from Boeing? (Better yet, Airbus, if any of you are out there!) -- Karl Swartz |INet kls@ditka.chicago.com 1-415/854-3409 |UUCP uunet!decwrl!ditka!kls |Snail 2144 Sand Hill Rd., Menlo Park CA 94025, USA Send sci.aeronautics.airliners submissions to airliners@chicago.com From kls Fri Dec 11 17:42:28 1992 Newsgroups: sci.aeronautics.airliners Path: news From: Robert Dorsett Subject: Re: Safety and design rankings (was Re: Flight controls) X-Submission-Date: Fri, 11 Dec 92 6:44:01 CST References: Message-ID: Approved: kls@ohare.Chicago.COM Reply-To: rdd@rascal.ics.utexas.edu Sender: kls@ohare.Chicago.COM X-Submission-Message-Id: Date: 11 Dec 92 17:42:28 PST In article kls@ohare.Chicago.COM (Karl Swartz) writes: >> These aren't safety-critical items (well, maybe the lighting is: it didn't >> work at Habsheim). > > I believe both intercom and lighting are considered safety-critical > items. Sorry: that was poorly phrased. It is a must-have, and, yes, it did fail. However, I understand the problem was mechanical in nature (CCF?); the software problems were eventually fixed. I don't know about the intercom, but the PA system is, as well. >Lighting in general may not be deemed critical, though certainly the >directional lighting in the floor is. Floor directional lighting is relatively new. It complements, but does not replace, the regular emergency floods: both are now considered critical. For the semantics fans: we should probably be careful in our use of the term "safety-critical" with respect to these systems: it is not, for instance, in the same league as the EFCS, and the software likely doesn't require the same confidence. Anyone known for sure? I would suspect emergency lighting is listed as an "essential function," not critical. > Is the MD-11 comparable to the 747-400 in this regard? I would assume > so since they are of comparable vintage. I would suggest not: the former is more of a derivative, the latter more of a new type, with its new wing (which was designed to support the all-upper- deck concept, plus maybe one more derivative after that), electrical system, extensive use of composites, new APU, etc. Each has a high degree of direct commonality with its predecessor, but from a technology basis, I don't think they're in the same league. One commenter to the paper Pete and I are brewing up took exception to my comparison, though: he feels the 767 was more of an equivalent to the A320. I disagree, from both design and avionics perspectives. Perhaps some of the Boeing people posting here can comment on the commonality of the various versions of the 747. > Where do the new generation 737s (-300/-400/-500) fit into this? FMS, new engines, composites, just about everything else is derivative. The "glass" in the cockpits is hackwork, IMHO, nowhere NEAR as integrated as the "all-new" glass airplanes such as the 747-400. I don't believe systems control has changed much at all. > And, for completeness, where do the glass-cockpit version of the MD-80 > family fit into the picture? My PERSONAL mental "ranking" of the sophistication of these airplanes is about: High-high automation/integration One philosophy | 777 Another philosophy | 747-400<--------------------------------------------------->A320/A330/A340 | HIgh automation/integration | MD-11 | 757/767<--------------------->A310, A300-600 | FMS only, varying or no glass, no standards | 747-300,737-300,-400,-500, MD-8X, F.100 | INS/PMS, conventional otherwise | 747-200/SP | INS only, very smart autopilot, fair integration | L1011 | A300 | INS only, simple, coupled autopilots, fair integration | 747-100/200, DC-10 | First/second-generation design, little integration | two-man | three-man DC-9,737-100,737-200<-----| | KC-135 | --------------->727, DC-9, 707, DC-8 Two-man airplanes have always used more automation than three-man crews; hence, I give them a slight edge among the "first-generation" airplanes. Others may have differing impressions; there's no hard and fast rule to apply. Of all these airplanes, the original 747 family has the best internal cockpit consistency, by far. Otherwise, the new Airbusses have the best design consistency. But I count some 19 fundamental cockpit designs in operation, countless permutations existing in most of them, depending on customer preferences in avionics and cockpit layout. The FMS's used on these airplanes are generally done by Honeywell, except that Boeing's using Smiths Industries for the 737, for some reason. Note that INS's on older-generation airplanes were often not purchased by customers who intended to use them for domestic service: EAL's A300-B4's, for instance, didn't even have them for service to South America. All of the first-generation airplanes currently have INS retrofits available; there are also on-again, off-again plans to offer a relatively sophisticated glass cockpit for the 727, with new engines. But it's important to note that INS interfaces were pretty much localized, with maybe a coupled mode for the autopilot. The devices were nowhere near as integrated in the cockpit design as 1980's/1990's crop, even if they were explicitly sold with airplanes (such as the early 747). They were "packages," not the "essence." LASTLY, note that the manufacturers are MUCH more assertive about preventing customers customizing their cockpits. This really got out of hand: for instance, I have a picture of a KLM 747-200 with some seven HSI's and CDI's and four full-sized ADI's, blanketing every spare square inch of the pilots' panels--that's what THEIR chief pilot apparently felt comfortable with. :-) Options are much more limited on modern airplanes; all customer variations are much "closer" to the manufacturer standard cockpit (the one that gets in all the publicity photos) than they used to be. Then again, nearly all the major airlines don't have anything resembling the engineering and design departments that they used to have, so they've forfeited the right to comment, to a large degree. Performance is now ensured by legal contract, rather than design, with the dollar being the bottom line. Caveat: I generally don't know that much about Douglas products (except for the DC-10 :-)); Boeing and Airbus have always caught my interests. --- Robert Dorsett rdd@cactus.org ...cs.utexas.edu!cactus.org!rdd From kls Fri Dec 11 17:42:30 1992 Newsgroups: sci.aeronautics.airliners Path: news From: palmer@icat.larc.nasa.gov (Michael T. Palmer) Subject: Re: Airbus safety (was Re: TWAs Status) X-Submission-Date: 11 Dec 92 16:22:39 GMT References: <1992Nov25.191925.27991@news.mentorg.com> <8762@lee.SEAS.UCLA.EDU> <1992Dec01.173212.27936@news.mentorg.com> Message-ID: Approved: kls@ohare.Chicago.COM Organization: NASA Langley Research Center, Hampton, VA USA Sender: kls@ohare.Chicago.COM X-Submission-Message-Id: Date: 11 Dec 92 17:42:30 PST rdd@cactus.org (Robert Dorsett) writes: >Seriously, this is a tremendously conservative industry. What isn't broken, >doesn't get fixed. However, when a better mouse-trap is invented, it is >almost always adopted, universally. The fact that no other manufacturer >is rushing to repeat Airbus' example suggests the arbitrariness of the >use of the sidesticks: if there were even minor operational or material >advantages in using them (and modified control laws) as interfaces to the >EFCS, you could bet your last dollar every other manufacturer would be doing >so, not least as the result of airline demand. We don't see that. >This isn't one of them. We aren't operating in a vacuum: NASA, as one example, >has been running a lot of research (over, and over) over the last 20 years, >addressing precisely these issues: the Airbus implementation is arguably on >the weaker of a variety of choices available. My contacts at Boeing agree - Boeing Flight Deck Research has been looking at sidestick controllers for a long time. They have decided that until they develop an airplane that is flown *differently* they will continue to use the column/yoke arrangement. Now, what I mean by differently really refers to switching from ATTITUDE control laws to VELOCITY VECTOR control laws. Mr Dorsett is correct; NASA Langley has decades of experience with sidestick controllers in our B-737 aircraft (it has TWO cockpits - standard in front, and an aft research cab from which you can fly the entire flight profile including landing). The sidestick control has been shown to be best when commanding velocity vector changes instead of attitude changes. This is an interesting way of using automation to ease the burden on the pilot while allowing him to also remain in the loop, since the automation configures the control surfaces to maintain the commanded direction of flight, but the pilot still "flies" the airplane (when not in full-autopilot). The velocity vector control-stick steering mode is by far the mode of choice of the pilots we bring in for experiments. Based on the work here and their own efforts, Boeing has decided that until they build a velocity vector airplane (hint: High-Speed Civil Transport) they will not provide a totally different way to fly an airplane designed with attitude control laws in mind. Please note that I am neither a Boeing employee nor spokesman, and I neither (officially) recommend nor approve of actions taken by them. All the info provided here (about Boeing's position) was provided to me personally by Boeing employees, though, so I have no reason to doubt it. It would be nice if some of you lurking Boeing people jumped in to correct any mistakes I have made. :-) -- Michael T. Palmer, M/S 152, NASA Langley Research Center, Hampton, VA 23681 Voice: 804-864-2044, FAX: 804-864-7793, Email: m.t.palmer@larc.nasa.gov PGP 2.0 Public Key now available -- Consider it an envelope for your e-mail From kls Fri Dec 11 17:42:31 1992 Newsgroups: sci.aeronautics.airliners Path: news From: smith@cyclone.mitre.org (Ralph N. Smith) Subject: Automatic Dependent Surveillance (ADS) X-Submission-Date: Fri, 11 Dec 1992 17:21:25 GMT References: Message-ID: Approved: kls@ohare.Chicago.COM Organization: Mitre Corporation, McLean, VA Sender: kls@ohare.Chicago.COM X-Submission-Message-Id: <1992Dec11.172125.27058@linus.mitre.org> Date: 11 Dec 92 17:42:31 PST In article hfunk@src.honeywell.com (Harry Funk) writes: > >In article dmarble@magnus.acs.ohio-state.edu (Duane F Marble) writes: >>A small point with respect to the material quoted from the New >>Scientist: Global Positioning System (GPS) birds do not "observe" >>anything, they just permit a ground based unit to compute it's >>location. > >My guess is that he was referring to Automatic Dependent Surveillance (ADS) >systems based on SATCOM, which has even less to to with GPS/Glonass birds. > >The planes [would] communicate their positions by means of a satellite link >to a ground-based tracking/control system. United currently has a few >747-400's that are so equipped. The major benefit envisioned is for >oceanic routes, where the fixed spacing (slots) system currently used >results in suboptimal tracks for a number of users of the system. ADS is >the successor to the Oceanic Display and Planning System (ODAPS), which I >think is currently installed at the Oakland and NY Air Route Traffic >Control Centers (ARTCCs). A few clarifying remarks about ADS. Technically, what United is currently doing in the Pacific is position reporting, where the aircraft sends a given set of information to the ground at fixed intervals. ADS systems involve more ground interaction, with the ground-based systems specifying the types of information desired, and the circumstances under which that information is provided, either periodically or at the occurrence of certain events in a flight. Also, there needs to be a differentiation made between the service provided by the aircraft, in this case ADS, and the communications by which the information is being sent. With a proper communications infrastructure, ADS and other air-ground communications based applications, can run without being concerned which particular air-ground data link is carrying the data, whether it be satellite, VHF radio, or any of a variety of communications links. ADS can also be used to retrieve intent information from an aircraft, in addition to current status information. Work is currently under way to integrate ADS reporting into the ODAPS system, rather than replacing it, at least in the near term. Also, an ADS-based application is one of the products to be produced by the Aeronautical Telecommunication Network (ATN) Project (ATNP). Several organizations in the aviation community are involved in the ATNP, including airlines (United is among them), avionics manufacturers, and the FAA. The potential benefits of ADS are substantial, including increased safety, and substantial savings in fuel and flight time. Ralph N. Smith ralph@mitre.org The MITRE Corporation (703)883-6084 McLean, Virginia All views and opinions are my own, so don't try to hold my employer responsible. From kls Fri Dec 11 17:42:32 1992 Newsgroups: sci.aeronautics.airliners Path: news From: palmer@icat.larc.nasa.gov (Michael T. Palmer) Subject: Re: Airbus safety (was Re: TWAs Status) X-Submission-Date: 11 Dec 92 17:26:55 GMT References: <1992Nov25.191925.27991@news.mentorg.com> <8762@lee.SEAS.UCLA.EDU> <1992Dec01.173212.27936@news.mentorg.com> Message-ID: Approved: kls@ohare.Chicago.COM Organization: NASA Langley Research Center, Hampton, VA USA Sender: kls@ohare.Chicago.COM X-Submission-Message-Id: Date: 11 Dec 92 17:42:32 PST rdd@cactus.org (Robert Dorsett) writes: >I would also note that in fighter aircraft, there isn't the issue of >two-pilot "peers" having to quickly and instinctively figure out who is >flying the airplane. On the A320, there is no interconnect between the >sidesticks: the captain can command a full-left in an emergency evasive >maneuver, the F/O full-right, and the net result will be an algebraically >added "zero." I believe this is incorrect, though I don't have the documentation here right now. My understanding is that whenever one of the sticks reaches a critical percentage of deflection (say, 75%), it becomes automatically the selected input device. At this point, the other control stick is ignored. So it's a race. Whoever slams their stick to the stops first wins, and the only way for the other crewmember to override is to physically attack the winner. Neat, huh? I'm not sure how sub-critical deflections are handled - they may indeed be algebraically summed. If any Airbus people can provide the straight scoop, I'd appreciate it. In the sidestick implementations used at NASA, the sticks are interconnected ("logically", really, since they are hydraulically back-driven) so that, like in current cockpits, whoever is strongest (i.e., most scared-to-death) wins. -- Michael T. Palmer, M/S 152, NASA Langley Research Center, Hampton, VA 23681 Voice: 804-864-2044, FAX: 804-864-7793, Email: m.t.palmer@larc.nasa.gov PGP 2.0 Public Key now available -- Consider it an envelope for your e-mail From kls Fri Dec 11 17:42:32 1992 Newsgroups: sci.aeronautics.airliners Path: news From: philip@rainbow.mentorg.com (Philip Peake) Subject: Re: Airbus safety (was Re: TWAs Status) X-Submission-Date: Fri, 11 Dec 1992 18:11:36 GMT References: <1992Nov25.191925.27991@news.mentorg.com> Message-ID: Approved: kls@ohare.Chicago.COM Followup-To: Organization: Mentor Graphics Sender: kls@ohare.Chicago.COM X-Submission-Message-Id: <1992Dec11.181136.2160@news.mentorg.com> Date: 11 Dec 92 17:42:32 PST In article , rdd@cactus.org (Robert Dorsett) writes: |> |> >If all new pilots were taught nothing but the side stick, |> >how long would the old arangementy last - and if the old arangement |> |> Why should pilots be taught nothing but a unique, *proprietary* side-stick |> design that no pilot had any experience with before four years ago, and which |> is only one of a variety of other possible designs? You are avoiding the question - read it again, the operative word is "if". I really don't think that a side-stick qualifies as "*proprietary*" does it ? Does Airbus hold patents on some aspect of it ? (I don't know the answer to this one - but if that IS true, then the result would be proprietary, and would deserve to fail). Changing the subject slightly, the world's safest aircraft (Concorde) uses technology which was new, and for a time unacceptable to various licensing authorities - it didn't have a MECHANICAL link between the stick and the control surfaces - only hydraulic. There was *much* concern over this, and lots of reaction from the pilots and safety mob - they almost won, and the Concorde almost had to be produced with a mechanical linkage, which no FULL CREW would be able to budge one mm if they all tried together - in fact, the linkages would probably have failed, before it would have been possible to move a control surface, when moving at full speed. As I said, it has proved to be the worlds' safest aircraft. Presumably, had a few airlines other than BA and Air France used them, someone would have flown one or two of them into the ground, and we would be arguing (or would have been arguing) about the safety of aircraft with no mechanical backup systems. Technology changes, old interfaces eventually HAVE to give way as they begin to fit less and less well with the new technologies. I don't seem to have noticed any raving about the TGV, the latest versions of which achieve speeds comparable to that of aircraft, and use a side-stick ... (yes, I know, French again ...) There is MUCH more prior art in train design, and they can write off considerably more people than even a fully loaded 747/400 if something goes drastically wrong. Philip From kls Fri Dec 11 17:42:33 1992 Newsgroups: sci.aeronautics.airliners Path: news From: raveling@Unify.com (Paul Raveling) Subject: Re: objects on wing tips X-Submission-Date: Fri, 11 Dec 92 20:58:52 GMT References: Message-ID: Approved: kls@ohare.Chicago.COM Organization: Unify Corporation (Sacramento) Sender: kls@ohare.Chicago.COM X-Submission-Message-Id: Date: 11 Dec 92 17:42:33 PST In article , dowlatir@cu1.crl.aecl.ca.crl.aecl.ca (Ramin Dowlati) writes: > I have a few questions for any of you passenger airplane gurus. > Several years ago, the aeropspace industry introduced vertical > flap-like things on the ends of their airplane wings. I've > only noticed these on 'larger' planes such Airbus, 747-400 and > MD-11. > Q1. What is the technical name for these flap-like things? Winglets. The most common name usage was "Whitcomb winglets" until they became fairly common. Many would say that Whitcomb was their inventor, but watching the tip feathers of large soaring birds suggests that Mother Nature should get some credit. > Q2. Are they mobile or fixed? Fixed > Q3. Do they only serve to stabilize the flight? No. Their purpose is to reduce induced drag, which they do by reducing circulation in wingtip vortices. > Q4. Why haven't they appeared on smaller aircraft? They have. In fact recent times have brought some controversy to competitive soaring, about whether winglets should count for measuring sailplane wingspan. This application uses winglets on airframes that weigh a few hundred pounds and carry one person, sometimes with about 1/4 inch of headroom for a pilot who's already laid out almost flat on his [or her] back. > Q5. The ones I saw on the Airbus were shaped like a 'V' > and symmetric with the wing tip, ie. one leg of the 'V' > was above the wing and the other pointed below the wing. > Whereas the ones on the 747-400 looked like extensions > of the actually wing, but bent 90 degrees upwards. > Why the difference? The 'V' form sounds like the classic Whitcomb design. Sailplanes don't use the downward-pointing winglet because ground clearance at the wingtips won't allow it. The same might be true of many airliners, with variations. This is just a guess: Many need clearance under the wings for servicing vehicles, such as fuel trucks. Having "hanging" winglets would increase the rate of ground damage. Some might also have a ground clearance problem for landing with an engine out, where designers usually plan for the certification limit of 5 degrees of bank (plus rudder of course) to compensate for asymmetric thrust. Finally, the extra winglet might require enough extra structure to negate most of the aerodynamic benefit it would produce. ------------------ Paul Raveling Raveling@Unify.com From kls Fri Dec 11 17:42:34 1992 Newsgroups: sci.aeronautics.airliners Path: news From: raveling@Unify.com (Paul Raveling) Subject: Re: hydraulic problems with DC-10's?? X-Submission-Date: Fri, 11 Dec 92 21:21:35 GMT References: Message-ID: Approved: kls@ohare.Chicago.COM Organization: Unify Corporation (Sacramento) Sender: kls@ohare.Chicago.COM X-Submission-Message-Id: Date: 11 Dec 92 17:42:34 PST Several people: lhe@sics.se (Lars-Henrik Eriksson) drinkard@bcstec.ca.boeing.com weiss@turing.SEAS.UCLA.EDU (Michael Weiss) write about the question of whether slats do or don't improve Cl (coefficient of lift), in addition to affecting stall speed (critical angle of attack)... I'd surmise that slats should increase Cl at any given AOA. The reason should be that they should delay boundary layer transitions (laminar-to-turbulent, turbulent-to-detached), increasing the wing area that's working efficiently. It's also true that this allows using a higher AOA, as noted by Lars-Henrik Eriksson; deployment of slats (and flaps too) DOES increase the AOA at which the wing achieves max Cl. An exaggerated example of this is adding a jib to a sailboat that formerly had only a mainsail. Adding even a small jib improves the main's Cl substantially. ------------------ Paul Raveling Raveling@Unify.com From kls Sat Dec 12 00:06:27 1992 Newsgroups: sci.aeronautics.airliners Path: news From: barney@skat.usc.edu (Barney Lum) Subject: Re: 737 Crash In Colorado Springs X-Submission-Date: 11 Dec 1992 18:05:01 -0800 References: Message-ID: Approved: kls@ohare.Chicago.COM Organization: USC University Computing Services, Los Angeles Sender: kls@ohare.Chicago.COM X-Submission-Message-Id: <1gbhcdINN23m@sol.usc.edu> Date: 12 Dec 92 00:06:27 PST rbarnick@mitre.org (Barnick, R.) writes: >In March 1991 a UAL 737 went down on final approach into COS. The final >accident investigation report was released a couple of days ago and carried >no cause. [ ... possible "hushing of report" due to UA's situation with gender/hiring/crewmember second-hand story by friend...] >It seems odd that the final accident report that came out indeed did say >nothing. (the following is derived from UPI articles in The Wall Street Journal and clari.news.aviation ) The final report issued by the National Transportation Safety Board did not "say nothing", but rather was unable to come to a definite conclusion as to the cause of the crash. However, it did indicate two "most likely" events leading to the crash: 1) malfunction in the directional control system. (The rudder is referenced, but could not be id'd as causal) 2) unusually severe atmospheric disturbance. (rotor) The articles go on to say that it's the "first time since 1974 that the board could not identify the cause of a major aviation accident." -------------------------------------------------------------------------- >| >| barney@usc.edu Barney@USCVM --> --> --> | ======= --- --- --- --- --- --- --- >| Permanent Student Pilot, On the Numbers >| -------------------------------------------------------------------------- From kls Sat Dec 12 00:06:28 1992 Newsgroups: sci.aeronautics.airliners Path: news From: weiss@curtiss.SEAS.UCLA.EDU (Michael Weiss) Subject: Re: hydraulic problems with DC-10s?? X-Submission-Date: 12 Dec 92 06:43:30 GMT References: Message-ID: Approved: kls@ohare.Chicago.COM Organization: SEASnet, University of California, Los Angeles Sender: kls@ohare.Chicago.COM X-Submission-Message-Id: <8905@lee.SEAS.UCLA.EDU> Date: 12 Dec 92 00:06:28 PST In article greg@octopus.dpsi.com (Gregory R. Travis) writes: >This is quite muddled though, as other anti-lift devices (such as spoilers) >will deploy at a given amount of aileron deflection. In fact, and I don't >have my DC-10 refs handy, I imagine that the ailerons on a -10 are locked >in place when the flaps are up (not the case in the Chicago crash, I know) Not true, at least in the case of the inboard ailerons. I still can remember back in 1984 flying a DC-10 from LAX to HON, and noting that when the inboard ailerons deflected to a certain point, the spoilers came up, too. I don't remember about the outboards (at the time, the inboard ones were more interesting to me). >In any case, the ORIGINAL poster's position that the loss of an engine >from a wing, considering the engine's moment and weight, would render the plane >uncontrollable is not supported either by analysis or historic precedent. And I stand (sit?) corrected on this issue. It struck me as an awfully large moment at first (and second, and third) glance, but I didn't have any actual weight numbers to compare. Now that I do, it makes perfect sense. -- \ | | | | | | | | | | | | | | | | | | | | | | | | | | | | | | | | | | | | | | / - Michael weiss@seas.ucla.edu | School of Engineering & Applied Science - - Weiss izzydp5@oac.ucla.edu | University of California, Los Angeles - / | | | | | | | | | | | | | | | | | | | | | | | | | | | | | | | | | | | | | | \ From kls Sun Dec 13 12:14:14 1992 Newsgroups: sci.aeronautics.airliners Path: news From: Robert Dorsett Subject: Re: Airbus safety X-Submission-Date: Sat, 12 Dec 92 6:35:56 CST References: <1992Nov25.191925.27991@news.mentorg.com> Message-ID: Approved: kls@ohare.Chicago.COM Reply-To: rdd@rascal.ics.utexas.edu Sender: kls@ohare.Chicago.COM X-Submission-Message-Id: Date: 13 Dec 92 12:14:14 PST In article philip@rainbow.mentorg.com (Philip Peake) writes: > |> >If all new pilots were taught nothing but the side stick, > |> >how long would the old arangementy last - and if the old arangement > |> > |> Why should pilots be taught nothing but a unique, *proprietary* side-stick > |> design that no pilot had any experience with before four years ago, and > |> which is only one of a variety of other possible designs? > > You are avoiding the question - read it again, the operative word is "if". > I really don't think that a side-stick qualifies as "*proprietary*" does it ? Not if one views it merely as a "substitute" interface. The A320 sidestick is not a "parallel" substitute: it's a replacement design concept. Besides the ergonomics, which raise their own issues, and which, as you note, would be duplicated by just about any manufacturer attempting to develop its own, it's what it DOES that determines its uniqueness. For instance, some pilots seem to like a specific feature: pull straight back on the stick to activate the TOGA mode. A few pilots *prefer* this to the regular stick-throttle combination one would instinctively use in such modes, despite the fact that if you did this on a "real" airplane, you'd soon stall it. So, should Boeing adopt this paradigm, if it goes to a sidestick design? This is one of MANY unique characteristics that solely characterize *Airbus'* sidestick. There are no official standards; no "sidestick specification" is in the public record. The software is jealously guarded. If Boeing decides to duplicate the stick concept, should it do so by examining operating manuals and hope it catches most of the idiosyncrasies? Or should it run away with the *idea*, and improve on it, offering its own version? In many ways, what I fear is what happens with "consumer software." Take a word processor, for instance: a simple idea, with *many* variations. Different companies have different ways of looking at the same problem: indeed, none of them may be a "best" solution. I don't think this comparison is off base: Airbus has REPEATEDLY and PUBLICLY stated that its technology is its selling point: to distinguish itself from Boeing, it MUST continue to do its own thing. The problem is that software is so much more easily changed than hardware, that we could very well start an avionics equivalent of "creeping featurism." Changing the way a stick behaves can be done in just one firmware update: no need to develop new tooling, production techniques, train assemblers and maintenance engineers, offer the retrofit during the next C check, etc. Just the internal development process, which one can assume is faster and cheaper than for hardware. But, by virtue of this ease, it's also more *unstable* than hardware-based solutions. Evidence to support this position? The A320 has about 4M of code. The A330/ A340, 10M. It's happening as we speak... Do any Honeywell people reading know how big the 777 EFCS is going to be? > Changing the subject slightly, the world's safest aircraft (Concorde) uses > technology which was new, and for a time unacceptable to various licensing > authorities - it didn't have a MECHANICAL link between the stick and the > control > surfaces - only hydraulic. There was *much* concern over this, and lots of > reaction from the pilots and safety mob - they almost won, and the Concorde > almost had to be produced with a mechanical linkage, which no FULL CREW would > be able to budge one mm if they all tried together - in fact, the linkages > would probably have failed, before it would have been possible to move a > control surface, when moving at full speed. I think you're overstating the situation considerably. Like they'd build an unflyable mock-up? :-) In the late 1950's and 1960's, there was considerable controversy over hydraulics, much of it justified: there was a low confidence level among pilots with conventional hydraulic systems. O-ring seals tended to break down, and the systems were rather "leaky"; many a flight had at least a partial failure. Pilots fully realized the benefits of hydraulics, and, starting after the 707, accepted the desirability of flying by hydraulics. However, what a lot of pilots wanted was a hybrid system, fly-by-cable, COMBINED with hydraulic boost. So, for instance, the 727 was developed with full-time hydraulic flight controls, but also a cable-driven "back-up" mode, which used control tabs to aerodynamically move the surfaces. When hydraul- ics were completely lost, controls became heavy, but the airplane had a "get it on the ground" capability. The 727 was the last such airliner to have this capability. Much of the controversy surrounding the BIG airliners, such as the 747, but even the 737, was that the manufacturers wanted to take away these tabs. This debate was an important part of an industry-wide *process*, which helped induce the manufacturers to develop more reliable systems: a 747 with the hydraulics reliability of, say, a 707, would not have been acceptable. This technological advance, combined with the *necessity* of using it in the specified mission profiles, helped silence objections. So Concorde certainly wasn't the first to go all-hydraulic, and the debate didn't start there. I can't comment on Concorde-ish control forces, but on even the 747, the "raw" forces in cruise, while high, don't require superhuman effort: the figures I've seen ranged from 50 lbs to 150 lbs. This would be unacceptable for normal operations, but is hardly the equivalent of trying to lift a ton of cement with one's little finger. > As I said, it has proved to be the worlds' safest aircraft. Sure, 14 airplanes, piloted by superbly qualified and trained aircrews, with immaculate and detailed maintenance. Flying what, TWO flights a day (no, not two per plane, two FLIGHTs, in the fleet) on 2 or so very well-defined routes, to major international airports? Concorde's an interesting experiment, but let's face it: its contribution is merely that it can be done, not that it can be done economically, or, even, safely, in the same types of conditions other airplanes are flown in. It is, however, an engineering achievement that France and England can be proud of, and I hope British Airways and Air France continue turning their Concorde profits, if, for nothing else, the living history the airplane represents. > I don't seem to have noticed any raving about the TGV, the latest versions of > which achieve speeds comparable to that of aircraft, and use a side-stick ... You will no doubt be DELIGHTED to note that I know nothing about trains. Nor do I particularly care to learn. :-) My interest here is airliners, not mass transportation. I'd suggest we compare standards within the genre. -- Robert Dorsett Internet: rdd@rascal.ics.utexas.edu UUCP: ...cs.utexas.edu!rascal.ics.utexas.edu!rdd From kls Sun Dec 13 12:14:17 1992 Newsgroups: sci.aeronautics.airliners Path: news From: Robert Dorsett Subject: A320 sidestick description + references (Re: Airbus safety) X-Submission-Date: Sat, 12 Dec 92 6:44:02 CST References: <1992Nov25.191925.27991@news.mentorg.com> <8762@lee.SEAS.UCLA.EDU> <1992Dec01.173212.27936@news.mentorg.com> Message-ID: Approved: kls@ohare.Chicago.COM Reply-To: rdd@rascal.ics.utexas.edu Sender: kls@ohare.Chicago.COM X-Submission-Message-Id: Date: 13 Dec 92 12:14:17 PST In palmer@icat.larc.nasa.gov (Michael T. Palmer) writes: > rdd@cactus.org (Robert Dorsett) writes: > >On the A320, there is no interconnect between the > >sidesticks: the captain can command a full-left in an emergency evasive > >maneuver, the F/O full-right, and the net result will be an algebraically > >added "zero." > I believe this is incorrect, though I don't have the documentation here > right now. My understanding is that whenever one of the sticks reaches > a critical percentage of deflection (say, 75%), it becomes automatically > the selected input device. At this point, the other control stick is > ignored. So it's a race. Whoever slams their stick to the stops first > wins, and the only way for the other crewmember to override is to physically > attack the winner. Neat, huh? I've looked into this closely. Unless there have been significant, recent changes, it doesn't work this way (other designs do, though). Here's an excerpt from the impending A320 paper that Pete Mellor and I are writing ("The A320 Electronic Flight Control System," title subject to change), which might help clear things up. I've inserted a couple of comments in brackets; these clarify passages, based on respondent comments. --------------------------- 4.1. Sidesticks; [AIRB88, 1.09.20, 3-5; ZIE86, COR88] The main flight control interface for the EFCS is one of two "sidestick" controllers. Conventional airplanes have two "control columns," mounted between each pilot's legs. The A320 does away with these, and instead has sidesticks mounted on the side-walls of the cockpit (incidentally resulting in outstanding pilot visibility of flight instrumentation). Conventional control devices reflect control forces to some degree. This "feel" is usually either supplied aerodynamically by the airplane, or, as in most airliners, via an artificial feed-back system. The sidesticks on the A320 do not have artificial feel. On the A320, springs are used to discourage abrupt control movements. Flight control specialists seem to regard this as a valid "artificial" feedback mechanism, but the point must be emphasized that the pilot is only reacting to the qualities of the spring: no tactile feedback relates to what the airplane may be doing (unlike a conventional control system); thus, secondary cues, such as the design of the flight displays, take on more importance. [...] Following are some of the force-characteristics of the sidesticks on the A320 [adapted from COR86]: Limits: Roll Pitch In Out Max. load 10 daN 3 daN 2 daN Threshold 0.5 daN 0.4 daN 0.4 daN Deflection +-16 deg. 20 deg. 20 deg. Orientation 20 deg fwd. 12 deg in 12 deg in. Proper pilot arm position is important for proper use of the sidestick. [actually, not that important: an early concern that proved unjustified] Thus, Airbus has included a fully-adjustable seat-arm, which features an LCD readout for arm angle. A pilot entering an aircraft need only remember his optimal settings, and set them up. The arm-rest rest position may be changed within an interval of [+20,-15] degrees. Supporting arm position may be adapted in the interval of [+15, -12] degrees. The sidesticks are not mechanically interconnected: inputs to one can't be felt on the other. Their inputs are algebraically added, with a maximum limit corresponding to the maximum deflection of one sidestick. No weight is given to the captain. Thus, if the captain pulls full left, and his first officer pulls full right, the net effect is zero. The last pilot to click on an override thumb-button (also used to disconnect the autopilot) obtains control: a small indicator light in front of the other pilot signals this fact. The potential exists for the pilots to "fight" over control of the sidesticks. Rather than the "strongest" pilot winning, the one with the fastest thumb will win. If the override button is held down for more than 30 seconds, it will "deactivate" the other sidestick. If the deactivated sidestick's override button is pushed, it will re-activate. This scheme has been the subject of much criticism from pilots: it is widely felt that one pilot should feel what the other pilot is up to, through the stick. Inter-pilot communication in an emergency may also be enhanced through better tactile feedback [PIK88, HEL86, SUM87]. Note that there is no trim control on the sidestick, since this is normally handled by the Normal and Alternate control laws (see below). In Direct law, pitch [trim!] must be set through controls on the center control pedestal. The only other button, besides the override switch, is a push-to-talk trigger, for the radios. The sidesticks are not merely a different interface, otherwise causing the same functional effects as a conventional airplane. The pilot's role in flight management is fundamentally changed, depending upon the mode the flight control computers are in. For example, in Normal Law, which is what the airplane is normally flown in (see below, 5.1), in a turn, a pilot must normally pull back on the stick, to compensate for lost lift. On the A320, this is not necessary: the pilot just moves the stick in the direction he wishes to turn, and the airplane will turn, automatically supplying the necessary elevator to maintain altitude [HOP87]. The sidestick is more of a "flight path command" interface, rather than a conventional "flight control surface deflection" interface. If, that is, the appropriate computer support is there: if not, the same sidesticks are used with one of the other "redundant" control laws, which are much more conventional in design. This raises an interesting issue of whether a pilot, who, with a properly-functioning system, will fly in "Normal" law almost all the time, will be "current" enough to satisfactorily fly the airplane in a significantly degraded mode, more akin to conventional control laws. This issue is addressed in training, not the interface. AIRB88 Airbus Industrie/Aeroformation, Flight Crew Operating Manual, 1988. COR88 S. G. Corps, "Airbus A320 side stick and flyPbyPwirePPan update," Society of Automotive Engineers Paper 861801. [Very GOOD paper] HEL86 Peter H. Heldt, "Airline requirements on a fly-by-wire aircraft--a pilot's view," Society of Automotive Engineers paper 861804. [so-so] HOP87 Harry Hopkins, "Simulating the A320," Flight International, 12 September 1987. [good article, weak on this issue] PIK88 J. R. Pike, "A320 in service--initial report," British Air Line Pilots Association, July 31, 1988. [extensive comments] SUM87 L.G. Summers, et. al., "Fly-by-wire sidestick controller evaluation," a paper presented at the SAE Aerospace Technology Conference and Exposition, Oct. 5-8, 1987. [ this is a decent overview of the MANY options available ] ZIE86 Bernard Ziegler, "Front seat on the future," Aerospace America, April 1986. [nicely-illustrated pap] ---------------------- How accurate is all this? Besides the cited sources, I've run into several other comments on the "algebraic" nature of the sidesticks. I haven't tried a neutral deflection in a simulator, but a recent email comment indicated that pilots, while trying to avoid an aircraft on the ground, commanded opposite inputs, thus leaving the flight path unchanged. If true, this would also tend to support the "algebraic," additive nature of inputs. In addition, of the A320 pilots who have reviewed the paper thus far, none have contested this point. One pilot did raise the issue that later transition work isn't as difficult as might be gathered from the last paragraph. So far, this is a minority viewpoint. -- Robert Dorsett Internet: rdd@rascal.ics.utexas.edu UUCP: ...cs.utexas.edu!rascal.ics.utexas.edu!rdd From kls Sun Dec 13 12:14:17 1992 Newsgroups: sci.aeronautics.airliners Path: news From: Christopher Davis Subject: Hysterical movie goof X-Submission-Date: Sat, 12 Dec 1992 16:11:38 -0500 Message-ID: Approved: kls@ohare.Chicago.COM Sender: kls@ohare.Chicago.COM X-Submission-Message-Id: <199212122111.AA26421@loiosh.eff.org> Date: 13 Dec 92 12:14:17 PST I was just watching a movie on TV ("Gotcha"). One of the characters flew to Paris, and there was the obligatory "airplane landing" shot to establish that he "really flew" there... ...on an Air France Cargo 747. (The side titles very clearly had three words, though the last was hard to read; however, the "under-nose" titles indicated the 747 cargo variant with a nose door.) Wonder what the in-flight meal was... From kls Sun Dec 13 18:07:13 1992 Newsgroups: sci.aeronautics.airliners Path: news From: ctillier@phoenix.princeton.edu (Clemens Emmanuel Tillier) Subject: Re: Control Sticks (was Re: Airbus safety) X-Submission-Date: Sun, 13 Dec 1992 13:01:53 GMT References: <1992Nov25.191925.27991@news.mentorg.com> Message-ID: Approved: kls@ohare.Chicago.COM Organization: Princeton University Sender: kls@ohare.Chicago.COM X-Submission-Message-Id: <1992Dec13.130153.3984@Princeton.EDU> Date: 13 Dec 92 18:07:13 PST philip@rainbow.mentorg.com (Philip Peake) writes: (discussion about control technologies deleted) >I don't seem to have noticed any raving about the TGV, the latest versions of >which achieve speeds comparable to that of aircraft, and use a side-stick ... >(yes, I know, French again ...) There is MUCH more prior art in train design, >and they can write off considerably more people than even a fully loaded 747/400 >if something goes drastically wrong. > >Philip This may be a little bit off the subject, but I think comparison with the TGV is unwarranted. The engineer in a train has control over only one degree of freedom of motion; the pilot in an aircraft has control over all three. It is therefore not surprising that entrusting so much control to one component will cause some questions. Besides, the TGV is utterly devoid of any control sticks... It uses the same throttle control as previous French locomotives (a horizontal wheel). Commercial TGV speed (300 kph) is not quite comparable to airliner speed. The maximum speed ever achieved by a TGV (515 kph) is also lower. Clem From kls Sun Dec 13 18:07:14 1992 Newsgroups: sci.aeronautics.airliners Path: news From: weinss@rpi.edu (Stephen Andrew Weinstein) Subject: Re: Hysterical movie goof X-Submission-Date: Sun, 13 Dec 1992 22:28:11 GMT References: Message-ID: Approved: kls@ohare.Chicago.COM Organization: Rensselaer Polytechnic Institute, Troy NY Sender: kls@ohare.Chicago.COM X-Submission-Message-Id: <+-j2lnh@rpi.edu> Date: 13 Dec 92 18:07:14 PST At one time (i.e. 1st half of 20th century ?), it was common for people flying between places without regular comercial airline service to hitch rides with either cargo or military planes. Scientists going to Antartica still do this and reporters needing local transportation in Vietnam when roads were closed did during the war there. (In 'Nam, helicopters were more common though because it was too expensive to build a runway that might get blown up.) Stephen Weinstein weinss@rpi.edu I will be away from the net and e-mail 12/17-1/10. If more college students voted (and could afford lobbyists), there would be more financial aid. From kls Mon Dec 14 14:11:42 1992 Newsgroups: sci.aeronautics.airliners Path: news From: rdd@cactus.org (Robert Dorsett) Subject: Yet more on the El Al crash X-Submission-Date: Mon, 14 Dec 92 00:56:54 CST Message-ID: Approved: kls@ohare.Chicago.COM Sender: kls@ohare.Chicago.COM X-Submission-Message-Id: <9212140656.AA22726@cactus.org> Date: 14 Dec 92 14:11:42 PST Today, I ran across a copy of a real, live, 747-200 Airplane Flight Manual. The AFM is the manufacturer's legal statement of airplane capabilities; it is custom-outfitted for each customer configuration, must be kept up to date, and is kept in the actual airplane: it's the bottom line for normal operations, , "outranking" even normal pilot Operations Manuals, which present processed data, based on the AFM, in a more user-friendly format. It contained some information which might be of interest to the net, particularly given the impression some people seemed to have of the ramifications of a two-engine failure. I'm also referring to the AvLeak of October 12 for particulars on the flight. The manual describes a 747-200, with CF6-50E engines, which produce a static thrust of ~52,000 lbs. The El Al airplane was powered with JT9D-7J's, which produce ~50,000 lbs of thrust). So it's not entirely applicable to the El Al crash, and I emphasize that the following is simply a "what-if," using the crash profile. We'll use a basic operating weight empty of 170,000 kgs, and the actual cargo load of 114,000 kg and the fuel load of 70,000 kgs. That gives us a gross weight of 354,000 kgs. The crash airplane achieved a maximum altitude of 5000' at 285 knots. It later achieved a maximum airspeed of 313 knots at 4900'. About six minutes after the initial failure, the captain reported problems with flaps. By the time the plane had descended to 2900', 25 seconds later, the crew issued a mayday call, indicating they were losing control; impact was 45 seconds after that. The slowest airspeed the airplane attained was 260 knots or so. The AFM gives some information that wasn't available during the original discussion. Such as: - 2-engine operation is *certainly* an in-envelope contingency. - It is possible to maintain altitude at up to 360,000 kgs. Some numbers. Draw your own conclusions: again, we're talking a different airplane, with different thrust capabilities. 1. Placarded flap speed limits. We can assume that if the crash airplane was following these limits, it was at flaps-up by the time the failure occurred. 1 275 knots 5 250 knots 10 238 knots 20 231 knots 25 205 knots 2. Gear-up stall speeds at 355,000 kgs: Flaps Speed 1 203 knots 5 153 knots 10 150 knots 20 144 knots 25 124 knots (landing flaps: assumes weight is down to 295,000 kgs). 3. At 355,000 kgs, *with two engines out*, our -200 would have been able to maintain level flight. It should also have been able to establish a climb gradient of 0.4% (175 ft./min) at 280 knots, the prescribed en route climb speed for this condition. --- Robert Dorsett rdd@cactus.org ...cs.utexas.edu!cactus.org!rdd From kls Mon Dec 14 14:11:43 1992 Newsgroups: sci.aeronautics.airliners Path: news From: palmer@icat.larc.nasa.gov (Michael T. Palmer) Subject: Re: A320 sidestick description + references (Re: Airbus safety) X-Submission-Date: 14 Dec 92 11:56:02 GMT References: <1992Nov25.191925.27991@news.mentorg.com> <8762@lee.SEAS.UCLA.EDU> <1992Dec01.173212.27936@news.mentorg.com> Message-ID: Approved: kls@ohare.Chicago.COM Organization: NASA Langley Research Center, Hampton, VA USA Sender: kls@ohare.Chicago.COM X-Submission-Message-Id: Date: 14 Dec 92 14:11:43 PST Robert Dorsett writes: >In palmer@icat.larc.nasa.gov (Michael T. Palmer) writes: >> rdd@cactus.org (Robert Dorsett) writes: >> >On the A320, there is no interconnect between the >> >sidesticks: the captain can command a full-left in an emergency evasive >> >maneuver, the F/O full-right, and the net result will be an algebraically >> >added "zero." >> I believe this is incorrect, though I don't have the documentation here >> right now. My understanding is that whenever one of the sticks reaches >> a critical percentage of deflection (say, 75%), it becomes automatically >> the selected input device. At this point, the other control stick is >> ignored. So it's a race. Whoever slams their stick to the stops first >> wins, and the only way for the other crewmember to override is to physically >> attack the winner. Neat, huh? >I've looked into this closely. Unless there have been significant, recent >changes, it doesn't work this way (other designs do, though). Here's an >excerpt from the impending A320 paper that Pete Mellor and I are writing >("The A320 Electronic Flight Control System," title subject to change), which >might help clear things up. Okee dokee. You're right; I must have been remembering the specs for a different system. I can now recall the stories about the thumb switch for overriding the other stick (my officemate and some of our test pilots participated in a week of A320 training down in Florida last year). Their reaction to it sparked quite a lengthy debate about the various sidestick implementations. Personally, the lack of feedback about what the other crewmember is doing is just astounding. Does nobody remember their flight training?!? How do you think your instructors (or YOU, if you teach) were able to know what you were doing even before the aircraft responded to your inputs? Tactile feedback can be a powerful and rich source of information. And when, pray tell, would an algebraic sum of the control inputs be the desired method of responding to the flight crew's actions? Did the designers think that the pilots would agree that the captain would only move his stick left/right and the first officer only fore/aft? "I'll be the base of the triangle, you be the height, and we'll let the EFCS do the hypoteneuse!" (For those with a math/statics background: "I'll be the i, you be the j, and we'll let the EFCS do the resultant vector!"). My point is that if the crewmembers are trying to do something different, the system should make that MORE not LESS visible. Otherwise, when the aircraft does not respond as they expect it to, each crewmember will simply increase the magnitude of his control input without really understanding what the h--- is going on. I guess this is what Reason would identify as a "latent system error." The pilots will eventually make an error; yep, the designers made sure of that. -- Michael T. Palmer, M/S 152, NASA Langley Research Center, Hampton, VA 23681 Voice: 804-864-2044, FAX: 804-864-7793, Email: m.t.palmer@larc.nasa.gov PGP 2.0 Public Key now available -- Consider it an envelope for your e-mail From kls Mon Dec 14 14:11:44 1992 Newsgroups: sci.aeronautics.airliners Path: news From: ncole@nyx.cs.du.edu (Noah Cole) Subject: Re: Hysterical movie goof X-Submission-Date: Mon, 14 Dec 92 04:11:44 GMT References: Message-ID: Approved: kls@ohare.Chicago.COM Organization: Macalester College, St. Paul Minnesota USA Sender: kls@ohare.Chicago.COM X-Submission-Message-Id: <1992Dec14.041144.7723@mnemosyne.cs.du.edu> Date: 14 Dec 92 14:11:44 PST ckd@eff.org (Christopher Davis) writes: >I was just watching a movie on TV ("Gotcha"). One of the characters >flew to Paris, and there was the obligatory "airplane landing" shot to >establish that he "really flew" there... >...on an Air France Cargo 747. (The side titles very clearly had three >words, though the last was hard to read; however, the "under-nose" >titles indicated the 747 cargo variant with a nose door.) I think that it would be interesting to list all of the errors that TV and Film writers make in the airline/avation market. For example, on Dir Hard II many of the flights were not wide-bodied flights that came from the West VCoast etc.. Anyone else seen things like this in films? -MN, er Noah Cole -- Noah Cole "Outside is America, NCOLE@MACALSTR.EDU Macalester College and also the car park" ncole@nyx.cs.du.edu St. Paul, MN 55105 - Bono, 27 December 1989 cncole@coos.dartmouth.edu 612-696-7388 Dublin aj909@cleveland.freenet.edu From kls Tue Dec 15 00:13:24 1992 Newsgroups: sci.aeronautics.airliners Path: news From: rdd@cactus.org (Robert Dorsett) Subject: Errata (Re: A320 sidestick description + references) X-Submission-Date: Mon, 14 Dec 92 20:55:35 CST Message-ID: Approved: kls@ohare.Chicago.COM Sender: kls@ohare.Chicago.COM X-Submission-Message-Id: <9212150255.AA13063@cactus.org> Date: 15 Dec 92 00:13:24 PST I thought I had caught most of these, but someone pointed them out: 1. A "daN" is a deca Newton, or 2.248 lbs. Airbus's main redeeming feature is that it's gone SI. 2. The "thumb-override" design means the guy with the *slowest* thumb will win, in the final estimation, not the fastest. Then again, if we really did have a thumb-war, the next guy would be fast to hit it again; I believe that's what I was thinking when I originally wrote the sentence. Apologies for any confusion this caused. 3. The comments on the necessity of applying back-stick in a turn were ambiguous. I was using as an example a situation of an airplane, straight and level. Suppose you're in a conventional airplane. You want to turn. You'd turn the wheel. This causes the airplane to bank. However, this decreases the net lift vector, which means the airplane will also descend. To counteract this effect, you'd apply slight back-stick, to command up- elevator, thus a greater angle of attack, thus more lift, to maintain level flight in the turn. It's all very coordinated, very natural. On the A320, one would simply use the stick to command a yaw. The system automagically applies the appropriate elevator correction to maintain the ancipated flight-path. If the pilot were to command any pitch-up, the airplane would CLIMB in the turn. This takes place in "Normal" law, the default flight mode. This is not "normal" as in "conventional": that's the "Direct" law, which is also the landing mode, so as to allow the pilot to handle a cross-wind landing and flare properly. If there are any more ways I can make this more confusing, please let me know. :-) --- Robert Dorsett rdd@cactus.org ...cs.utexas.edu!cactus.org!rdd From kls Tue Dec 15 00:13:28 1992 Newsgroups: sci.aeronautics.airliners Path: news From: kls@ohare.Chicago.COM (Karl Swartz) Subject: Re: Hysterical movie goof X-Submission-Date: Tue, 15 Dec 1992 07:33:23 GMT References: Message-ID: Approved: kls@ohare.Chicago.COM Organization: Chicago Software Works X-Submission-Message-ID: <1992Dec15.073323.297@ohare.Chicago.COM> Sender: kls@ohare.Chicago.COM Date: 15 Dec 92 00:13:28 PST In article ncole@nyx.cs.du.edu (Noah Cole) writes: >I think that it would be interesting to list all of the errors that >TV and Film writers make in the airline/avation market. I agree. But TV and film is a bit far afield from airliners; perhaps rec.arts.movies would be a better place. If someone compiles a list I'll consider it as for sci.aeronautics.airliners. One bit of film goof trivia that perhaps *is* directly relavnt to airliners is in United's safety video. As has been mentioned else- where on Usenet, the shot of folks sliding down the emergency slides in the DC-10 version depicts a 747, as can be identified by the doors and the fact that they open out to the side, not inside and up. The 757 appears to use the same shot (awfully high up and it doesn't look like a narrow-body aircraft). One question I've had about this film, besides where they could find a flight attendent or model who could smile so continuously, is what type of aircraft the opener of the cockpit was taken on. I always manage to think about it when the scene is almost over. >For example, on Dir Hard II many of the flights were not wide-bodied >flights that came from the West VCoast etc.. Are you suggesting that all flights from the West Coast are wide bodies? That's hardly the case, of course. -- Karl Swartz |INet kls@ditka.chicago.com 1-415/854-3409 |UUCP uunet!decwrl!ditka!kls |Snail 2144 Sand Hill Rd., Menlo Park CA 94025, USA Send sci.aeronautics.airliners submissions to airliners@chicago.com From kls Tue Dec 15 00:13:29 1992 Newsgroups: sci.aeronautics.airliners Path: news From: kls@ohare.Chicago.COM (Karl Swartz) Subject: Re: Safety and design rankings (was Re: Flight controls) X-Submission-Date: Tue, 15 Dec 1992 08:11:13 GMT References: Message-ID: Approved: kls@ohare.Chicago.COM Organization: Chicago Software Works X-Submission-Message-ID: <1992Dec15.081113.632@ohare.Chicago.COM> Sender: kls@ohare.Chicago.COM Date: 15 Dec 92 00:13:29 PST In article rdd@rascal.ics.utexas.edu writes: >> Is the MD-11 comparable to the 747-400 in this regard? I would assume >> so since they are of comparable vintage. >I would suggest not: the former is more of a derivative, the latter more of >a new type, with its new wing (which was designed to support the all-upper- >deck concept, plus maybe one more derivative after that) ... My understanding was that the 747-400 does *not* have a new wing but rather a tweaked version of the original. I recall some statement from Boeing regarding the lack of winglets on the 777, which noted that the 777 had a new wing and starting from a clean slate it was more efficient to not have them, whereas working from an existing design as with the 747-400 it was helpful to have them. >... electrical system, extensive use of composites, new APU, etc. Hmmm ... sounds a lot like the MD-11 as well the 747-400. >My PERSONAL mental "ranking" of the sophistication of these airplanes is >about: ... > FMS only, varying or no glass, no standards > | > 747-300,737-300,-400,-500, MD-8X, F.100 What I've seen suggests the F.100 is quite advanced, probably not far behind the A320 and perhaps closer to the Airbus philosophy than to Boeing's. > INS/PMS, conventional otherwise Ok, I'll risk it ... what's PMS? (We're talking about airplanes!) > two-man | three-man > DC-9,737-100,737-200<-----| Every first-generation 737 I've seen has a third seat for the flight engineer. I believe this was one of the selling points of the DC-9 over the 737. >The FMS's used on these airplanes are generally done by Honeywell, except >that Boeing's using Smiths Industries for the 737, for some reason. Boeing recently made a substantial change to the FMS on new 737s, and offers a retrofit kit for older new-generation 737s. I believe this was a replacement ... perhaps away from Smiths? Having observed the trials and tribulations of friends with MGs and their Smiths electrics I'm not enthusiastic about a Smiths FMS! :-) >LASTLY, note that the manufacturers are MUCH more assertive about preventing >customers customizing their cockpits. This really got out of hand ... I've always wondered just what the flight engineer really does on a 767 equipped for three flight crew. I believe QANTAS does this. Also, some A310s lack the FFCS (Forward Facing Crew Cockpit) having instead what I assume is a cockpit more like an older A300. All of these are due primarily to union/labor pressures. >Performance is now ensured by legal contract, rather than design, >with the dollar being the bottom line. Well, mandated, at least, if not ensured. -- Karl Swartz |INet kls@ditka.chicago.com 1-415/854-3409 |UUCP uunet!decwrl!ditka!kls |Snail 2144 Sand Hill Rd., Menlo Park CA 94025, USA Send sci.aeronautics.airliners submissions to airliners@chicago.com From kls Wed Dec 16 04:19:47 1992 Newsgroups: sci.aeronautics.airliners Path: news From: gregory@bcstec.ca.boeing.com (Greg Wright) Subject: Re: Yet more on the El Al crash X-Submission-Date: Tue, 15 Dec 1992 20:15:58 GMT References: Message-ID: Approved: kls@ohare.Chicago.COM Organization: Boeing Sender: kls@ohare.Chicago.COM X-Submission-Message-Id: Date: 16 Dec 92 04:19:47 PST In article rdd@cactus.org (Robert Dorsett) writes: > >The manual describes a 747-200, with CF6-50E engines, which produce a >static thrust of ~52,000 lbs. The El Al airplane was powered with JT9D-7J's, >which produce ~50,000 lbs of thrust). So it's not entirely applicable to >the El Al crash, and I emphasize that the following is simply a "what-if," There is also a difference in engine mountings, fairings, and low speed behavior. You could find a significant change in CLmax because of these things. > >--- >Robert Dorsett >rdd@cactus.org >...cs.utexas.edu!cactus.org!rdd -- ________Greg Wright____________ "I struggle to be brief | gregory@bcstec.ca.boeing.com | and become obscure." | gregory@halcyon.com | |____uunet!bcstec!gregory_______| NOT A BOEING SPOKESPERSON. From kls Wed Dec 16 04:19:48 1992 Newsgroups: sci.aeronautics.airliners Path: news From: John DiMarco Subject: Economics of new vs. older planes X-Submission-Date: Tue, 15 Dec 1992 15:32:59 -0500 Message-ID: Approved: kls@ohare.Chicago.COM Sender: kls@ohare.Chicago.COM X-Submission-Message-Id: <92Dec15.153313est.30980@marvin.cdf.toronto.edu> Date: 16 Dec 92 04:19:48 PST I understand there's an airfield in Arizona where unused airliners are parked for extended periods. I'm wondering, considering the economic doldrums most airline companies seem to be in, why these planes are not put into use in lieu of buying new ones? The answer, I think, would depend on the difference in operating costs between new and older planes. Could someone with a better understanding of these issues shed some light on this? Solid numbers (eg. purchase prices of new vs. used planes, fuel consumption differences, etc.) would be highly appreciated. Thanks, John -- John DiMarco jdd@cdf.toronto.edu Computing Disciplines Facility Systems Manager jdd@cdf.utoronto.ca University of Toronto EA201B,(416)978-1928 From kls Wed Dec 16 04:19:48 1992 Newsgroups: sci.aeronautics.airliners,rec.aviation.simulators Path: news From: gary@maestro.mitre.org (Gary Bisaga) Subject: Simulation software for transport category aircraft X-Submission-Date: Tue, 15 Dec 1992 21:31:11 GMT References: <1992Dec11.033041.4518@nmsu.edu> Message-ID: Approved: kls@ohare.Chicago.COM Reply-To: gbisaga@mitre.org Organization: The Mitre Corporation, McLean Virginia Sender: kls@ohare.Chicago.COM X-Submission-Message-Id: <1992Dec15.213111.296@linus.mitre.org> Date: 16 Dec 92 04:19:48 PST I am looking for some real-time flight simulation software for commercial transport category aircraft, preferably public domain. It can run on most any kind of Unix platform (we use Sun, SGI, HP, IBM) or DOS. It need not have any graphical output (out the window, avionics, etc.) as we have existing software to provide all of those functions, but it does need a couple of things: 1) Relatively realistic (simple 6dof?) equations that can support commercial airliner-type flight models 2) Autoflight system including: a) At a minimum, inner loop auto-pitch/roll/throttle control b) Much better, outer loop altitude/heading/VS capture c) Preferably VOR/LOC/APP tracking loop 3) Flight model to go with all this We have looked at point-mass models and they can do many things but we need to be able to hand-fly with or without FD for certain operations. (Obviously FD was not included in the above list because we can easily develop that given the pitch/roll commands and other software we have.) Any information is much appreciated. -- Gary Bisaga (gbisaga@mitre.org, 703-883-5543) From kls Wed Dec 16 04:19:49 1992 Newsgroups: sci.aeronautics.airliners Path: news From: drinkard@bcstec.ca.boeing.com (Terrell D. Drinkard) Subject: Re: Northwest cancels Airbus X-Submission-Date: Wed, 16 Dec 1992 02:37:14 GMT References: Message-ID: Approved: kls@ohare.Chicago.COM Organization: Boeing Sender: kls@ohare.Chicago.COM X-Submission-Message-Id: Date: 16 Dec 92 04:19:49 PST In article hoyme@src.honeywell.com (Ken Hoyme) writes: [much deleted material about the NWA/Airbus cancelations] >.... (But then, NWA is already >deeply in debt to Airbus, since they decided to buy A320s based on a >dynamite financing package that Boeing could not match.) We here at Boeing also like to think that the Northwest, and the United, purchase of the A320 were both driven by financial considerations only. Not true. The plain facts are that the A320 flies higher, faster, and farther than the competing Boeing 737-400 while carrying a heavier load and burning less gas to boot. That isn't fuel per seat, that is trip fuel. Northwest's decision, as noted by their VP of Finance a couple of months ago, was based on superior performance and a higher acquisition cost than that of the 737. United came to pretty much the same conclusion. None of the above is intended to make little of the financial implications of each of those deals, just put it in a technical frame of reference. -- Terry drinkard@bcstec.boeing.com "Anyone who thinks they can hold the company responsible for what I say has more lawyers than sense." From kls Wed Dec 16 04:19:50 1992 Newsgroups: sci.aeronautics.airliners Path: news From: drinkard@bcstec.ca.boeing.com (Terrell D. Drinkard) Subject: Re: hydraulic problems with DC-10's?? X-Submission-Date: Wed, 16 Dec 1992 02:50:15 GMT References: Message-ID: Approved: kls@ohare.Chicago.COM Organization: Boeing Sender: kls@ohare.Chicago.COM X-Submission-Message-Id: Date: 16 Dec 92 04:19:50 PST In article weiss@ada.SEAS.UCLA.EDU (Michael Weiss) writes: [Much aero-techie argument deleted] >In any case, my point is that there would have been a severe weight unbalance >between the wings, and I have doubts that it could have been countered by the >ailerons. The whole reason that there was a negative roll moment was that the >left wing STALLED, not that it lost lift directly from the retracting slats. >I'm still not convinced that even WITH the slats extended it could have been >prevented. As one gentleman pointed out earlier, there is a history of airplanes losing engines and remaining perfectly controllable. A 737-200 lost one, I believe it was the #2, on takeoff just last week. It impacted along the side of the runway, the airplane turned around without further incident and landed safely. While certainly not as common as an airline misplacing your luggage :-), engine departures do happen and the airplanes do tend to land without further damage or excitement. -- Terry drinkard@bcstec.boeing.com "Anyone who thinks they can hold the company responsible for what I say has more lawyers than sense." From kls Wed Dec 16 04:19:50 1992 Newsgroups: sci.aeronautics.airliners Path: news From: drinkard@bcstec.ca.boeing.com (Terrell D. Drinkard) Subject: Re: Northwest cancels Airbus X-Submission-Date: Wed, 16 Dec 1992 02:59:37 GMT References: Message-ID: Approved: kls@ohare.Chicago.COM Organization: Boeing Sender: kls@ohare.Chicago.COM X-Submission-Message-Id: Date: 16 Dec 92 04:19:50 PST In article rdd@rascal.ics.utexas.edu writes: > >It'll be interesting to see what ramifications this has on the UAL deal: >did United plan on using Airbus/NWA North American facilities? > It is my understanding from chatting with the maintenance engineers at UAL on separate occassions that UAL is forming an A320 maintenance group (I have a resume in, therefore might not be considered an unbiased observer) which may be based at their new maintenance facility in Indianapolis. -- Terry drinkard@bcstec.boeing.com "Anyone who thinks they can hold the company responsible for what I say has more lawyers than sense." From kls Wed Dec 16 04:47:56 1992 Newsgroups: sci.aeronautics.airliners Path: news From: Karl Swartz Subject: archive of articles (and other stuff) now available X-Submission-Date: Wed, 16 Dec 92 4:40:07 PST Message-ID: Approved: kls@ohare.Chicago.COM Sender: kls@ohare.Chicago.COM X-Submission-Message-Id: Date: 16 Dec 92 04:47:56 PST A first pass at an Airliners ftp archive is now available on ftp.eff.org, thanks to Helen Rose . Access is via the usual anonymous ftp; the following files are in the /pub/airliners directory: archive-1992.Z (187393 bytes) A compressed mail folder of the first 160 articles posted to sci.aeronautics.airliners. (The last was dated December 14, 1992.) Next month, hopefully, the archive will be updated as soon as articles are posted, but for now this is being done by hand when the mood strikes. :-) boeing-code (6726 bytes) A list of Boeing customer codes -- the xx part of a designation such as 747-2xxB. specifications (1776 bytes) Some e-scribblings of mine showing various useful parameters of some airliners, from which I computed the power/weight ratios discussed a few weeks ago. Hopefully this will evolve (or be replaced) into a complete reference. Again, this is a first pass, mainly to get something out for some folks who wanted it before the holidays. In the next month or two real-time archives of the newsgroup will be implemented, as will several other sites. More reference data will be made available as well. -- Karl Swartz |INet kls@ditka.chicago.com 1-415/854-3409 |UUCP uunet!decwrl!ditka!kls |Snail 2144 Sand Hill Rd., Menlo Park CA 94025, USA Send sci.aeronautics.airliners submissions to airliners@chicago.com From kls Wed Dec 16 14:00:40 1992 Newsgroups: sci.aeronautics.airliners Path: news From: Robert Dorsett Subject: Re: Safety and design rankings (was Re: Flight controls) X-Submission-Date: Wed, 16 Dec 92 6:03:09 CST References: Message-ID: Approved: kls@ohare.Chicago.COM Reply-To: rdd@rascal.ics.utexas.edu Sender: kls@ohare.Chicago.COM X-Submission-Message-Id: Date: 16 Dec 92 14:00:40 PST In article kls@ohare.Chicago.COM (Karl Sw artz) writes: >In article rdd@rascal.ics.utexas.edu wri tes: >My understanding was that the 747-400 does *not* have a new wing but >rather a tweaked version of the original. There were significant changes in structure and composition, plus the addition of the wingtip extensions and the winglets. As I understand it, the new wing is intended to support both the 747-400 and a full-upper- deck 747-500. >I recall some statement >from Boeing regarding the lack of winglets on the 777, which noted >that the 777 had a new wing and starting from a clean slate it was >more efficient to not have them, whereas working from an existing >design as with the 747-400 it was helpful to have them. One doesn't really relate to the other: the 777 has a high-aspect-ratio wing, reducing drag in its long-range profile. The drag reduction is what the winglets were purportedly for; the 777 will achieve a better effect, naturally. >> INS/PMS, conventional otherwise > >Ok, I'll risk it ... what's PMS? (We're talking about airplanes!) Performance Management System. It's the precursor of FMS's; it was originally deployed on the 747-SP. It was used to provide a lot of "nice to know" and performance-related flight guidance, to obtain optimal fuel burn. >Every first-generation 737 I've seen has a third seat for the flight >engineer. Hmm. I have some vague recollection of a three-man 737, but I think I'm thinking of that 767. The 737 was designed for a two-man crew. If three- man ships were produced, there are precious few of them. I wasn't able to find any explicit references to three-man variants in my notes. Be careful to distinguish between a "flight engineer" and someone occupying the jump seat. Quite a few airlines will run a "third man" due to either union pressures, or to provide training experience for new-hires; one often sees "transients" (instructors, check pilots, deadheading pilots) in the jump seat. Lufthansa, I believe, ran a third man on the 747-400 for a long time; Air Inter's unions had a major fight over the A320. The 737 itself is certifi- cated for two-man operation, and that's what is legally needed in order to fly it. It's an interesting question whether the union desire to safeguard its members' economic interests might actually *decrease* safety, in this case. I question the wisdom of inserting a third man "for the ride," who has no real operational significance in the cockpit. It would seem that such a person could serve as a destabilizing influence. If they're going to go three-man, they should go whole-hog, and give him something to do, in the fundamental cockpit design. But I digress. >I believe this was one of the selling points of the DC-9 >over the 737. Yet ANOTHER excerpt from _Legend & Legacy_, pp. 255-257: "The DC-9 head start was a killer for Boeing's sales force, yet it wasn't the only handicap Boeing's new baby faced in its adolescence. A deep hole was dug by the men who would fly the plane, and unwittingly it was Delta's pilots who handed their brethren the shovel that almost buried the 737. When Delta bought the DC-9, it won an agreement from its pilots that the cockpit be designed for a two-man crew, eliminating the flight engineer. This was permissible under an FAA regulation that allowed any jet transport weighing less than 80,000 lbs to be flown by two pilots. Both the original DC-9 and BAC-111 met the so-called '80,000-pound rule,' heresy and anathema to the Air Line Pilots Association (ALPA). It could do nothing about the precedent Delta's pilots had set for the DC-9, but the union began pressuring the FAA to change it regulation for the 737, and at the same time warned US airlines planning to buy the Boeing jet that future pilot contracts would specify a three-man crew for the 737. "ALPA argued that with no flight engineer to help them, the pilots' increased workload made it difficult to watch out for other traffic, thus enhancing the chances for mid-air collisions, and also created more danger during bad-weather landings. The union's case might have sounded logical until one began wondering why a two-man crew was safe for the DC-9 and BAC-111, and not for the 737. Nevertheless, the FAA changed its regulations to the extent of requiring Boeing to prove that the 737 could be operated safely with two pilots. The irony was that once having been certificated for a two-man crew, three subsequent larger DC-9 models weighing far more than 80,000 pounds also were automatically certificated for two pilots, but not the 737. "Thus the baby Boeing's late start was saddled with a further sales han- dicap--many airlines considering the 737 bought the DC-9 instead, unwilling to add the expense of a third cockpit crew member who literally was nothing except an extra set of eyeballs. United and Western, after arbitration, agreed to a three-man crew, although that third man was a classic case of feather-bedding--or 'feather birding,' as then-FAA administrator Najeeb Halaby expressed it. "Western's pilots referred to the extra crew member as GIBs, for 'Guy in Back,' but abandoned this nomenclature hastily when a pilot ran across the word 'gib' in a dictionary and discovered it meant castrated tomcat. "Lew Wallick once asked a Piedmont captain what the third crew member did, riding in a jump seat just behind the pilots, unable to reach any controls. "'He doesn't do much,' the captain admitted. 'He sits back there and spills coffee in my brainback [nickname for the briefcase holding air- way maps and aircraft manuals]. But come next summer, he's gonna mow my lawn.' "Brien Wygle was in charge of the 737's flight test program, and worked with engineering to design a cockpit whose workload would put the least possible stress on two pilots. "'We went to a lot of trouble proving this out,' Wygle said. 'We didn't have much computer input--they weren't as sophisticated then--but we designed a simple cockpit management system because the FAA told us that when we came up for certification, they were going to be very tough. They were under great pressure from ALPA, which wanted the FAA to say that the 737 needed a flight engineer or any third crew member.' "Tough they were. "'The FAA made us jump through a lot of hoops,' Wygle recalled. 'There was an unprecedented amount of testing, all kinds of simulated engine and systems failures, low-visibility approaches and landings, and even test flights through high-density traffic on the eastern seaboard. And to the FAA's credit, they ruled that the 737 was completely safe to fly with a two-man crew. "The ruling, however, couldn't recoup the sales Boeing had already lost because of the ALPA campaign; the union itself eventually came around to admitting that a sophisticated, well-designed cockpit didn't need a flight engineer [MUCH later! --rdd] And in one sense, ALPA did the 737 a favor. It forced Boeing to improve the plane to the point where it would be so good it didn't matter how many men were in the cockpit." >I've always wondered just what the flight engineer really does on a >767 equipped for three flight crew. I believe QANTAS does this. There was only one 3-man 767 built, and that was for UAL. Ansett uses a three-man crew, but there's no major change to the structure-- probably just a CDU interface on the rear on the control pedestal, so the third man can play navigator, to keep busy. Again, be sure to distinguish between a "GIB" (:-)) and someone who plays an intended operational role in the cockpit. >Also, some A310s lack the FFCS (Forward Facing Crew Cockpit) having >instead what I assume is a cockpit more like an older A300. All of >these are due primarily to union/labor pressures. This is the first I've heard of it. Got any references? It'd be REAL difficult to do, and I'd question whether the returns would be enough to entice the manufacturer to do it. >>Performance is now ensured by legal contract, rather than design, >>with the dollar being the bottom line. > >Well, mandated, at least, if not ensured. I suspect that when dollar figures reach low earth orbit, performance is whatever you say it is. :-) Two good references: Incidentally, there are good design summaries of the 757/767 and A310/A300-600 cockpits in _Aerospace Crew Station Design_, G. P. Carr, editor, Elsevier Science Publishers, 1984. There's also a superb design overview of the 737 in the February 3, 1966 issue of FLIGHT INTERNATIONAL. --- Robert Dorsett rdd@cactus.org ...cs.utexas.edu!cactus.org!rdd From kls Wed Dec 16 14:00:43 1992 Newsgroups: sci.aeronautics.airliners Path: news From: jongsma@esseye.si.com (Ken Jongsma) Subject: FMS Suppliers (Was: Safety) X-Submission-Date: Wed, 16 Dec 1992 09:42:37 -0500 (EST) References: Message-ID: Approved: kls@ohare.Chicago.COM Sender: kls@ohare.Chicago.COM X-Submission-Message-Id: Date: 16 Dec 92 14:00:43 PST >>The FMS's used on these airplanes are generally done by Honeywell, except >>that Boeing's using Smiths Industries for the 737, for some reason. Perhaps because Smiths offered a better product at the time? Perhaps because Boeing does not want to depend on one supplier for all it's FMS design? >Boeing recently made a substantial change to the FMS on new 737s, and >offers a retrofit kit for older new-generation 737s. I believe this >was a replacement ... perhaps away from Smiths? Having observed the >trials and tribulations of friends with MGs and their Smiths electrics >I'm not enthusiastic about a Smiths FMS! :-) Ohh... It helps to know a bit about the corporate history before one makes comments like "Smiths electrics." OK. The division of Smiths that makes the 737 FMS is located in Grand Rapids, Michigan. Prior to being purchased by Smiths a few years back, it used to be known as Lear Siegler, Inc. (LSI) and was originally formed by Bill Lear, the designer of the Lear Jet. All of the FMS work that this division does was initiated well before SI bought the division, so references to UK product lines aren't really valid. Some history on the 737 FMS. The original 737-300 FMS was based on work done for the 727 and early 737s (Performance Data Computer) and a system that was not fielded called the PNCS. A few years ago, along with Update 4 of the software, the "Eagle" FMS hardware was fielded. It had a faster processor and more memory. More recently, Update 6 came with additional memory. I assume this is the update you were refering to. Next month, the "Dual" FMS will be shipped. This system is a 4 MCU, two computer system. In addition to commercial FMSs, SI also makes the Nav System for all 600+ USAF C-130s. This same system has been installed on some international C-130s. (Although I work on the military side of the house, I checked with our commercial people to verify the above. Obligatory Disclaimer: I speak for myself...) Ken -- Ken Jongsma Smiths Industries jongsma@esseye.si.com Grand Rapids, Michigan 73115.1041@compuserve.com From kls Wed Dec 16 14:00:44 1992 Newsgroups: sci.aeronautics.airliners Path: news From: hoyme@src.honeywell.com (Ken Hoyme) Subject: Re: Safety and design rankings (was Re: Flight controls) X-Submission-Date: Wed, 16 Dec 1992 18:43:16 GMT References: Message-ID: Approved: kls@ohare.Chicago.COM Organization: Honeywell Systems & Research Center, Mpls. MN, USA. Sender: kls@ohare.Chicago.COM X-Submission-Message-Id: Date: 16 Dec 92 14:00:44 PST In article kls@ohare.Chicago.COM (Karl Swartz) writes: > My understanding was that the 747-400 does *not* have a new wing but > rather a tweaked version of the original. I recall some statement > from Boeing regarding the lack of winglets on the 777, which noted > that the 777 had a new wing and starting from a clean slate it was > more efficient to not have them, whereas working from an existing > design as with the 747-400 it was helpful to have them. I believe the winglets issue on the 777 was also complicated by the folding wing option. Which no one has ordered -- even those airlines who originally expressed interest in the option. Has development on the folding wing stopped?? I had heard that Boeing was getting tired of the investment required to keep the option open while not receiving any orders for it. >>The FMS's used on these airplanes are generally done by Honeywell, except >>that Boeing's using Smiths Industries for the 737, for some reason. > Boeing recently made a substantial change to the FMS on new 737s, and > offers a retrofit kit for older new-generation 737s. I believe this > was a replacement ... perhaps away from Smiths? Having observed the > trials and tribulations of friends with MGs and their Smiths electrics > I'm not enthusiastic about a Smiths FMS! :-) Me too! But then, I *am* biased on this issue. :-) However, I do not believe that the part of Smiths that supports cars and the part that designs avionics has a whole lot to do with each other. I suspect (but do not know for certain) that Boeing's choice of Smiths might be related to trying to get some competition in this area. Ken Hoyme Honeywell Systems and Research Center (612)951-7354 3660 Technology Dr., Minneapolis, MN 55418 Internet: hoyme@src.honeywell.com From kls Wed Dec 16 14:00:45 1992 Newsgroups: sci.aeronautics.airliners Path: news From: hoyme@src.honeywell.com (Ken Hoyme) Subject: Re: Airbus safety X-Submission-Date: Wed, 16 Dec 1992 19:16:27 GMT References: <1992Nov25.191925.27991@news.mentorg.com> Message-ID: Approved: kls@ohare.Chicago.COM Organization: Honeywell Systems & Research Center, Mpls. MN, USA. Sender: kls@ohare.Chicago.COM X-Submission-Message-Id: Date: 16 Dec 92 14:00:45 PST In article Robert Dorsett writes: > The problem is that software is so much more easily changed than hardware, > that we could very well start an avionics equivalent of "creeping featurism." > Changing the way a stick behaves can be done in just one firmware update: no > need to develop new tooling, production techniques, train assemblers and > maintenance engineers, offer the retrofit during the next C check, etc. > Just the internal development process, which one can assume is faster and > cheaper than for hardware. But, by virtue of this ease, it's also more > *unstable* than hardware-based solutions. > Evidence to support this position? The A320 has about 4M of code. The A330/ > A340, 10M. It's happening as we speak... > Do any Honeywell people reading know how big the 777 EFCS is going to be? Well, this is an apples-to-oranges comparison. I can only speak (and only in vauge terms, of course) about the portion of the 777 that Honeywell is producing. Honeywell's Airplane Information Management System (AIMS) contains the FMS function similar to previous generation airplanes, but does not encompass the autopilot (Rockwell Collins) nor the FBW Flight Controls (GEC). The FMS alone requires about 2Mbytes of executable code. It also requires a Nav Data Base and RAM for operation. I do not know the complexity of the other components of the "EFCS". Of course, Displays could be considered another part of the system. That function lives in AIMS on the 777. From kls Thu Dec 17 03:35:15 1992 Newsgroups: sci.aeronautics.airliners Path: news From: rdd@cactus.org (Robert Dorsett) Subject: Re: Safety and design rankings (was Re: Flight controls) X-Submission-Date: Thu, 17 Dec 92 02:34:17 CST References: Message-ID: Approved: kls@ohare.Chicago.COM Organization: Capital Area Central Texas UNIX Society, Austin, Tx Sender: kls@ohare.Chicago.COM X-Submission-Message-Id: <9212170834.AA17343@cactus.org> Date: 17 Dec 92 03:35:15 PST In article you write: >> was a replacement ... perhaps away from Smiths? Having observed the >> trials and tribulations of friends with MGs and their Smiths electrics >> I'm not enthusiastic about a Smiths FMS! :-) > >Me too! But then, I *am* biased on this issue. :-) However, I do not >believe that the part of Smiths that supports cars and the part that >designs avionics has a whole lot to do with each other. I suspect (but >do not know for certain) that Boeing's choice of Smiths might be related >to trying to get some competition in this area. And if we're going to pick on Smiths for that, we should note that Delco Electronics, which produced the INS used on just about every airliner produced in the late 60's and 70's, was a subsidiary of General Motors. :-) Same logo as the car-stereo systems, too. :-) But boy, is Smiths' 737 engine display tacky. :-) --- Robert Dorsett rdd@cactus.org ...cs.utexas.edu!cactus.org!rdd From kls Thu Dec 17 03:35:17 1992 Newsgroups: sci.aeronautics.airliners Path: news From: rdd@cactus.org (Robert Dorsett) Subject: REVIEW of _FMC User's Guide_ X-Submission-Date: Thu, 17 Dec 92 02:43:07 CST Message-ID: Approved: kls@ohare.Chicago.COM Sender: kls@ohare.Chicago.COM X-Submission-Message-Id: <9212170843.AA17392@cactus.org> Date: 17 Dec 92 03:35:17 PST Title: FMC User's Guide: Advanced Guide to the Flight Management Computer Author: Bill Bulfer Published by: Bill Bulfer Technical Publications 2031 River Falls Drive Kingwood, TX 77339 713-358-7252 Cost: $40. Optional update service, $12. Pages: ~200; extensive illustrations. It is designed to be carried in a flight bag, printed on "half-pages," in a flexible, compact, three-ring binder. No ISBN. The Flight Management System is the "heart" of modern transport operations. It is the core of navigation functionality and automatic flight control, and permits a flight to be flown very economically. Despite its overall usefulness, standard interfaces leave something to be desired: consequently, a high proportion of training time is currently dedicated to the FMS, at the inevitable expense of other systems. There is evidence this training is somewhat lacking, with hands-on time limited. This means that "real learning" occurs in-flight, on the job. This is not a desirable situation, since it increases heads-down operations, thus decreasing the situational awareness of the pilot(s). The author, a Continental 737 pilot, wrote the book (manual, really) in an attempt to provide a high-quality, goal-oriented overview of FMS functions, as a supplement to airline training programs. It is a result of his own exposure, extensive research, and feedback from the manufacturers. The book is oriented around the Smiths Industries FMS, in use on the 737, but the author explicitly addresses differences and similarities with the Honeywell lineage, which is in use on more types of airplanes. The book is written for pilots, but may also be of interest to researchers and hard-core airliner enthusiasts. It is oriented around CDU (control data unit) operation, but includes mode control unit notes, where appropriate. As indicated, it's heavily goal-oriented, showing precisely what the pilot would see on various screens, with relevant fields highlighted, as he attempts to set up a solution to a given problem. An update service is available, on a yearly basis, for a nominal fee. Bulfer plans on issuing updates about every six months: the current update is about 80 pages. He's also working on a "final exam," to go with the manual. My main gripe is that, although the book is based on laser-printed originals, his printing service seems to have scaled the originals to fit on the pages. Consequently, some thin lines, such as boxes surrounding notes, look somewhat odd, with varying print intensity along the line. Otherwise, the type and illustrations look fine. I heartily recommend this book for anyone seriously interested in the intricacies of FMS operation. It is one of the best pilot-oriented technical publications I've ever seen. Disclaimer: I have no financial connection with or interest in this project; I'm just a very satisfied customer. I received my copy in September, and have been working through it (slowly :-)) since then. --- Robert Dorsett rdd@cactus.org ...cs.utexas.edu!cactus.org!rdd From kls Thu Dec 17 13:27:31 1992 Newsgroups: sci.aeronautics.airliners Path: news From: pal@regent.e-technik.tu-muenchen.dbp.de (Peter Loibl) Subject: Re: Northwest cancels Airbus X-Submission-Date: Thu, 17 Dec 1992 10:41:31 GMT References: Message-ID: Approved: kls@ohare.Chicago.COM Organization: Technical University of Munich, Germany Sender: kls@ohare.Chicago.COM X-Submission-Message-ID: Date: 17 Dec 92 13:27:31 PST drinkard@bcstec.ca.boeing.com (Terrell D. Drinkard) writes: >In article rdd@rascal.ics.utexas.edu writes: >> >>It'll be interesting to see what ramifications this has on the UAL deal: >>did United plan on using Airbus/NWA North American facilities? >> >It is my understanding from chatting with the maintenance engineers at UAL >on separate occassions that UAL is forming an A320 maintenance group (I >have a resume in, therefore might not be considered an unbiased observer) >which may be based at their new maintenance facility in Indianapolis. BTW: The Sueddeutsche Zeitung here in Germany (one of the best ones here) wrote yesterday, that UA is negotiating with Boeing about delays and cancellations of their orders. No other details given. Peter Loibl pal@regent.e-technik.tu-muenchen.de From kls Thu Dec 17 13:27:34 1992 Newsgroups: sci.aeronautics.airliners Path: news From: driscoll@src.honeywell.com (Kevin Driscoll) Subject: Re: REVIEW of _FMC User's Guide_ X-Submission-Date: Thu, 17 Dec 92 10:48:24 CST References: Message-ID: Approved: kls@ohare.Chicago.COM X-Submission-Message-Id: <9212171648.AA18464@couqusmungus.src.honeywell.co> Sender: kls@ohare.Chicago.COM Date: 17 Dec 92 13:27:34 PST > The book is oriented around the Smiths Industries FMS, in use on the 737, > but the author explicitly addresses differences and similarities with the > Honeywell lineage, which is in use on more types of airplanes. The phrase "more types" more correctly should be "all other types" since the FMS on all commercial transport airplanes except the 737 are Honeywell's. However, saying that there is a lineage is a bit strong. Being a modern customer driven company 8^), we build the FMS that the airframer wants. That is why on over half the flights where I fly the A320 jump seat, the pilots ask me why the A320 FMS is not as good as the Boeing versions. And the answer is -- "That is the way Airbus wanted it." With the differences in FMSs, it is hard to see how one manual can cover them all. The NW pilots I have talked to universal like the A320's FMS manual. You may want to order one from Honeywell pubs. With the way that ATAs are now piloted, I think an FMS manual is the closest thing there is to a "pilot's guide" to the airplane. From kls Thu Dec 17 13:27:35 1992 Newsgroups: sci.aeronautics.airliners Path: news From: palmer@icat.larc.nasa.gov (Michael T. Palmer) Subject: Re: Safety and design rankings (was Re: Flight controls) X-Submission-Date: 17 Dec 92 18:47:22 GMT References: Message-ID: Approved: kls@ohare.Chicago.COM Organization: NASA Langley Research Center, Hampton, VA USA X-Submission-Message-Id: Sender: kls@ohare.Chicago.COM Date: 17 Dec 92 13:27:35 PST In a previous post, somebody wrote: >>Every first-generation 737 I've seen has a third seat for the flight >>engineer. Then, somebody else wrote: >Hmm. I have some vague recollection of a three-man 737, but I think I'm >thinking of that 767. The 737 was designed for a two-man crew. If three- >man ships were produced, there are precious few of them. I wasn't able >to find any explicit references to three-man variants in my notes. Well, NASA Langley operates Boeing B-737-100 Hull Number 1. That's right, Number 1. I can tell you for a fact that it was designed for two pilots. >Be careful to distinguish between a "flight engineer" and someone occupying >the jump seat. Quite a few airlines will run a "third man" due to either >union pressures, or to provide training experience for new-hires; one often >sees "transients" (instructors, check pilots, deadheading pilots) in >the jump seat. Correct. However, putting a third pilot on that tiny, flip/fold-down seat would require hazardous duty pay. I have ridden in that seat for quite a few hours, and it is NOT repeat NOT like riding in a 767 cockpit!! Still, there is no rear "engineer's station" for a third crewmember anyway. And the seat blocks access to the cabin door! It was never designed for constant use. -- Michael T. Palmer, M/S 152, NASA Langley Research Center, Hampton, VA 23681 Voice: 804-864-2044, FAX: 804-864-7793, Email: m.t.palmer@larc.nasa.gov RIPEM Public Key available soon --- Consider it an envelope for your e-mail From kls Fri Dec 18 04:51:00 1992 Newsgroups: sci.aeronautics.airliners Path: news From: kls@ohare.Chicago.COM (Karl Swartz) Subject: Happy Holidays! X-Submission-Date: Fri, 18 Dec 1992 12:48:08 GMT Message-ID: Approved: kls@ohare.Chicago.COM Organization: Chicago Software Works Sender: kls@ohare.Chicago.COM X-Submission-Message-ID: <1992Dec18.124808.6922@ohare.Chicago.COM> Date: 18 Dec 92 04:51:00 PST I'll be in Chicago for the holidays, starting tomorrow (courtesy a United 747 for the obligatory airliner content!) until after Christmas. My brother has network access so I'll be processing submissions to the group from there, though response will not be quite as timely as I usually try to maintain. My machines here in California will be cared for, so you needn't (and shouldn't) change how you submit articles. I hope the holidays are safe and happy for all of you, and I'll look forward to the continuing interesting airliners discussions and debates in the new year! -- Karl Swartz |INet kls@ditka.chicago.com 1-415/854-3409 |UUCP uunet!decwrl!ditka!kls |Snail 2144 Sand Hill Rd., Menlo Park CA 94025, USA Send sci.aeronautics.airliners submissions to airliners@chicago.com From kls Sun Dec 20 16:07:59 1992 Newsgroups: sci.aeronautics.airliners,rec.travel.air Path: news From: eandersn@mach1.wlu.ca (Niels Ejvind Andersen) Subject: Surviving a Commercial Aircraft Accident X-Submission-Date: Thu, 17 Dec 1992 21:36:21 GMT Message-ID: Approved: kls@royko.Chicago.COM Organization: Wilfrid Laurier University Sender: kls@royko.Chicago.COM X-Submission-Message-ID: Date: 20 Dec 92 16:07:59 CST I attended the Toronto Transport Canada Aviation Safety Seminar last night. One of the topics was "Surviving a Commercial Aviation Accident", and it was interesting enough that I'll post it here. Please bear in mind that flying is extremely safe. These comments are intended as *risk management*. In the very remote possibility that your aircraft has an accident, and your survival depends on your knowledge of the aircraft exits and safety systems, do the smart thing, and learn them. According to a survey of airline passengers, the commonly held opinion is that 75% of commercial aircraft accidents have one or more fatalities. In reality, only 15% of such accidents have fatalities. The belief in an inflated death rate is a result of media concentration on accidents with fatalities. Pay attention to the safety briefing given by the cabin attendants at the beginning of the flight. Read the pamphlet that describes the aircraft exits and safety systems such as oxygen masks and life vests (life vests are difficult to put on, and it is important that you understand). Make sure your family pays attention. Explain the items to your family, if need be. Take note of which passengers between you and your exit don't pay attention to the safety briefing -- these people won't know what to do after an accident, and they may well keep you and your family from getting out safely as well. The rules follow, but first, remember that flying is the safest mode of transportation available to you. ENJOY YOUR FLIGHT. -- Six Rules for Surviving a Commercial Aviation Accident (as a passenger). 1. Bring a coat that won't burn or melt, and wear it both during takeoff and during landing. 2. Wear flat shoes. Don't remove them until after takeoff, and put them back on for the landing. 3. Make sure your seatbelt is snug, and not too high on your hips, during the takeoff and the landing. 4. Know what brace position you need to get into for a crash (this should be the position your body will end up in after the deceleration, if you were sitting upright). With a 1.5G or 2G deceleration in the forward direction, your head, arms and legs will strike the seat in front of you. If an accident is imminent, get into the brace position immediately. Don't wait for "brace" instructions from the crew. The brace position will minimize your risk of injury. 5. Know where the nearest exits are. Get an aisle seat within three rows of an emergency exit, and note the position of the next nearest exit as well (your alternate exit). Count the number of seats between you and your exit, as well as the number of seats between you and your alternate exit. Make sure you know where the nearest exits are, because if there is a post-crash fire, the signs and lights above the exits will be obscured by dense smoke. [I have difficulty with this rule -- I like to sit by a window -- if I wanted an aisle seat, I'd go see a movie at our local theatre!] 6. Know how to open the exit doors. After a crash, get to an exit as quickly as you can, any way you can. Remember to release your seat belt first. Climb over the seats, if you have to. Leave all your possessions behind; some items will have sharp protrusions that can puncture the escape slide, and any item you carry will slow down your progress. The cabin attendants will confiscate any possession you are carrying anyway, as you pass by them on your way out the exit. Smoke and fumes kill more people than the impact forces, so it is extremely important to evacuate the aircraft as quickly as possible; you may only have a few seconds. The smoke tends to form in layers, with the most toxic and thick layers near the ceiling. Stay down low, in crouch position, as you make your way to the exit. Try to keep on your feet, but crawl if necessary. Once outside the aircraft, get well clear, preferably upwind. If the aircraft has ditched, don't inflate your life vest until you are outside the aircraft. If the water is cold, try to climb up onto a piece of floating debris, to get out of the water. Move around as little as possible in the water, to conserve your body heat. -- Niels Ejvind Andersen [eandersn@mach2.wlu.ca / 70511.2302@compuserve.com] Information Systems, Wilfrid Laurier University, Waterloo, ON N2L 3C5 Canada ... VFR NC4 From kls Sun Dec 20 16:08:03 1992 Newsgroups: sci.aeronautics.airliners Path: news From: rdd@cactus.org (Robert Dorsett) Subject: Re: REVIEW of _FMC User's Guide_ X-Submission-Date: Fri, 18 Dec 92 01:50:15 CST References: Message-ID: Approved: kls@royko.Chicago.COM Organization: Capital Area Central Texas UNIX Society, Austin, Tx X-Submission-Message-Id: <9212180750.AA15197@cactus.org> Sender: kls@royko.Chicago.COM Date: 20 Dec 92 16:08:03 CST In article you write: >> The book is oriented around the Smiths Industries FMS, in use on the 737, >> but the author explicitly addresses differences and similarities with the >> Honeywell lineage, which is in use on more types of airplanes. > >The phrase "more types" more correctly should be "all other types" since >the FMS on all commercial transport airplanes except the 737 are Honeywell's. Good point! >However, saying that there is a lineage is a bit strong. Being a >modern customer driven company 8^), we build the FMS that the airframer >wants. That is why on over half the flights where I fly the A320 jump >seat, the pilots ask me why the A320 FMS is not as good as the Boeing >versions. And the answer is -- "That is the way Airbus wanted it." The author notes that there are two major "baselines": one Boeing, the other "European," which he lumps the MD-11, Fokker, and Airbus aircraft into. He explicitly notes that he's describing the Boeing baseline. --- Robert Dorsett rdd@cactus.org ...cs.utexas.edu!cactus.org!rdd From kls Sun Dec 20 16:08:05 1992 Newsgroups: sci.aeronautics.airliners Path: news From: todamhyp@unlv.edu (Brian M. Huey) Subject: Tire burn-out during landings X-Submission-Date: Fri, 18 Dec 92 18:06:49 GMT Message-ID: Approved: kls@royko.Chicago.COM Organization: University of Nevada at Las Vegas, College of Engineering Sender: kls@royko.Chicago.COM X-Submission-Message-ID: <1992Dec18.180649.15191@unlv.edu> Date: 20 Dec 92 16:08:05 CST I realize this a relatively basic question (I think) nevertheless: I am neither a pilot nor an airliner engineer. However, everytime I watch a airliner land, a cloud of smoke is produced when the wheels come in contact from the ground due to friction. Couldn't this cloud and the effect of friction be lessened by inducing a angular velocity upon the wheels to match the airliners speed with respect to the runway? -- DISCLAIMER: I can neither confirm nor deny any opinions expressed in this article directly reflect my own personal or political views and furthermore, if they did, I would not be at liberty to yield such an explanation of these alleged opinions. From kls Sun Dec 20 16:08:07 1992 Newsgroups: sci.aeronautics.airliners Path: news From: James R Ebright Subject: Re: Hysterical movie goofs X-Submission-Date: Fri, 18 Dec 92 13:56:45 EST References: Message-ID: Approved: kls@royko.Chicago.COM X-Submission-Message-Id: <9212181856.AA06323@bottom.magnus.acs.ohio-state.edu> Sender: kls@royko.Chicago.COM Date: 20 Dec 92 16:08:07 CST >I was just watching a movie on TV ("Gotcha"). One of the characters >flew to Paris, and there was the obligatory "airplane landing" shot to >establish that he "really flew" there... > >...on an Air France Cargo 747. (The side titles very clearly had three >words, though the last was hard to read; however, the "under-nose" >titles indicated the 747 cargo variant with a nose door.) > >Wonder what the in-flight meal was... About eight years ago I flew LAX to CDG direct on Air Chance and it was ACTUALLY a CARGO 747 with one cabin (appx 80 seats) of passengers. The in-flight meal was normal US airline food...quite a let down from normal Air Chance fare -- so much so, it prompted me to complain in writing. (Why else fly AF except for the food?) -- Jim Ebright (james.ebright@osu.edu) "Spam, eggs, sausage and spam - that's not got much spam in it." >>> All kids can be educated -- even yours and mine. <<< From kls Sun Dec 20 16:08:08 1992 Newsgroups: sci.aeronautics.airliners Path: news From: rdd@cactus.org (Robert Dorsett) Subject: Aha! The three-man 767 rears its ugly head... X-Submission-Date: Sun, 20 Dec 92 05:36:50 CST Message-ID: Approved: kls@royko.Chicago.COM X-Submission-Message-Id: <9212201136.AA13911@cactus.org> Sender: kls@royko.Chicago.COM Date: 20 Dec 92 16:08:08 CST I knew I wasn't going mad... In a recent post, I commented on a three-man 767. Karl hadn't heard of it, which surprised me, so I went looking for it--and couldn't find it--which surprised me even more. I finally ran across this blurb. I have no recollection of a picture of one of these critters, though. Can someone clear up this matter? FI, 3/20/82, p. 685: "So far, most of the "live" flight-test experience has been with a three- pilot-configured flightdeck on the first four 767s. [...] All 757s, and the fifth and many subsequent 767s will have a two-pilot-configured cockpit. In the two-man flightdeck, the early 767's conventional bank of engine instruments on the center panel is replaced by the two CRT screens. These are the display element of the engine-indicating/crew-alerting system (EICAS), which replaces normal engine instruments and continually scans the aircraft systems for abnormalities, relaying any findings." I interpret that as meaning the three-man ships had electromechanical in- struments, which means there had to be a real, live, flight engineer. I am certain three of the four were retrofitted to two-man configuration, but am almost certain the first one lingers on, somewhere. Memory suggests it's Boeing's 767 testbed. The maddening thing is, I read an article within the last two or three years, which went into all this in great detail, but I can't remember where it was published... This article seemed to indicate that the customer (UAL, I THINK) which initially wanted three-man ships saw the two-man configuration, fell in love with it, and abandoned all plans for three-man operation. R. From kls Mon Dec 21 11:30:44 1992 Newsgroups: sci.aeronautics.airliners Path: news From: megazone@obsidian.WPI.EDU (MegaZone23) Subject: Re: Tire burn-out during landings X-Submission-Date: 21 Dec 1992 04:08:11 GMT References: Message-ID: Approved: kls@royko.Chicago.COM Organization: WPI USAF -- United Society of Animation Fans Sender: kls@royko.Chicago.COM X-Submission-Message-Id: <1h3fvbINNs9a@bigboote.WPI.EDU> Date: 21 Dec 92 11:30:44 CST In article todamhyp@unlv.edu (Brian M. Huey) writes: >the wheels come in contact from the ground due to friction. Couldn't >this cloud and the effect of friction be lessened by inducing a >angular velocity upon the wheels to match the airliners speed with Spinning the wheels would indeed cut down on the tire wear, and I believe that this has been tested. However, there are problems. When you spin a tire it acts as a gyroscope and will resists having it's course altered. This can make handling tricky as you would have 10 or more wheels all spinning on most airliners. Have you ever held a spinning bicycle tire in your hands and tried to move it? Same effect. You also need to weigh the advantages against the disadvantages. How much does the tire wear cost the airline? How much would mantenance on a system to spin the wheels cost? Plus you will have the initial cost for the system and the added weight. It is one more system to break down, etc... All in all it just doesn't appear to be worth it... ############################################################################### # I have one prejudice, and that is against stupidity. Use your mind, think! # #Email megazone@wpi.wpi.edu Moderator, WPI anime FTP site 130.215.24.1 /anime# ############################################################################### From kls Sat Dec 26 00:01:45 1992 Newsgroups: sci.aeronautics.airliners Path: news From: van_heel@rz-berlin.mpg.de Subject: Martinair DC10 crash? X-Submission-Date: 21 Dec 92 14:05:17 +0100 Message-ID: Approved: kls@royko.Chicago.COM Organization: Rechenzentrum MPG Berlin Sender: kls@royko.Chicago.COM X-Submission-Message-ID: <1992Dec21.140517.7887@rz-berlin.mpg.de> Date: 26 Dec 92 00:01:45 CST Martinair DC10 crash around 9.30 am in Portugal. According to a news broadcast which I heard at 10 o'clock this morning, a Martinair DC10 charter plane had just crash-landed on a Portuguese airport with 322 passenger aboard after being struck by lightning. The plane exploded immediately after the unsuccesful crash-landing. Marin van Heel From kls Sat Dec 26 00:01:47 1992 Newsgroups: sci.aeronautics.airliners Path: news From: Hayes_Press@qmgate.larc.nasa.gov (Hayes N. Press) Subject: Re: Aha! The three-man 767 rears its ugly head... X-Submission-Date: 22 Dec 1992 13:25:26 GMT References: Message-ID: Approved: kls@royko.Chicago.COM Reply-To: Hayes_Press@qmgate.larc.nasa.gov (Hayes N. Press) Organization: Lockheed Engineering and Sciences Company X-Submission-Message-Id: <1h7507INNo79@rave.larc.nasa.gov> Sender: kls@royko.Chicago.COM Date: 26 Dec 92 00:01:47 CST >I knew I wasn't going mad... In a recent post, I commented on a three-man >767. Karl hadn't heard of it, which surprised me, so I went looking for >it--and couldn't find it--which surprised me even more. I finally ran across >this blurb. >I have no recollection of a picture of one of these critters, though. Can >someone clear up this matter? I don't know if I can clear this up but I can assure you that you are not going mad, unless madness is common in aerospace professionals. The original design for the 767 was indeed for a 3 person-crew (even though the anthropometrics were probably 3 man-crew biased). There was an extensive article done in Harvard Business Review (sorry I don't recall the year but it is pre-1990) on the design/production aspects. The article dealt with the management approaches used to determine whether the production line of the first 30 or so, 767 should be stopped and retrofited "in-position" or whether the line should be allowed to continue with the first ~30 being 3 person-crew and the ~31st and then on being the two person variety. In summary, the decision was to build the first ~30 as 3 person-crew and then if (when) they got certification of a two person configuration they would tear-out the third station and make the other necessary changes needed by use of "RAMS" teams. These teams would then have only one learning curve on the modifications needed, rather than ~30 since the aircraft on the line would all be at different stages of production. There was even one airline (which I don't recall either, but it may have been UAL or someone in the South Pacific) that accepted their aircraft in the three person configuration, whether that configuration has since been changed or not I'm also not so sure. The extra flight station being removed from the original design may also explain the roominess of the 767 cockpit. It is by far the largest I have been in with more free space and comfort for the jumpseat passenger. Speaking only for myself. Hayes N. Press e-mail address: Hayes_Press@qmgate.larc.nasa.gov phone (804) 864-2715 fax (804) 864-7793 Good old Postal Service: Lockheed Engineering and Sciences Co. 144 Research Drive MS 156A Hampton, Virginia 23666 From kls Sat Dec 26 00:01:48 1992 Newsgroups: sci.aeronautics.airliners,rec.travel.air Path: news From: davidm@questor.rational.com (David Moore) Subject: Re: Surviving a Commercial Aircraft Accident X-Submission-Date: Tue, 22 Dec 1992 20:48:48 GMT References: Message-ID: Approved: kls@royko.Chicago.COM Organization: Rational X-Submission-Message-Id: Sender: kls@royko.Chicago.COM Date: 26 Dec 92 00:01:48 CST eandersn@mach1.wlu.ca (Niels Ejvind Andersen) writes: >If the aircraft has ditched, don't inflate your life vest until you are >outside the aircraft. One is usually instructed to inflate one half of the life vest while in the cabin, and to fully inflate the vest once you have left the aircraft. Were reasons given why this alternative procedure is preferable? Did they say anything about US flights on which life vests are not provided? I find it hard to believe that those floating cushions are of any value at all. There is one other point which comes across clearly from the crew demonstrations - it is important to remain smiling at all times during emergency procedures. Would I be right in thinking this is to reduce panic? From kls Sat Dec 26 00:01:49 1992 Newsgroups: sci.aeronautics.airliners Path: news From: rdd@cactus.org (Robert Dorsett) Subject: First Flights on airliners X-Submission-Date: Thu, 24 Dec 92 06:30:34 CST Message-ID: Approved: kls@royko.Chicago.COM X-Submission-Message-Id: <9212241230.AA19239@cactus.org> Sender: kls@royko.Chicago.COM Date: 26 Dec 92 00:01:49 CST I caught the tail-end of an episode of "First Flights," on the Arts & Entertainment Network, which dealt with advanced-technology airliners, Wednesday night. There were segments on the MD-12, the Fokker 100, and the MD-80. I only saw about the last 8 minutes, so assume the A320 or A310, and Boeing products, were featured in the first 20. A&E also shows a "First Flights" early Sunday evening, but I don't know whether it's a repeat of the Wednesday show. Whichever it is, they repeat that night's episode at 0530 EST on Monday. There was nothing earth-shattering about what they were talking about (Neil Armstrong: "So I guess the pilot's turning into more of a... supervisor..."). But the visuals are pretty, with little flicker. "First Flights" is a nicely photographed, half-hour show, about *aviation*, and not all about blowing up things to a rock & roll beat, unlike "Wings." :-) --- Robert Dorsett rdd@cactus.org ...cs.utexas.edu!cactus.org!rdd From kls Sat Dec 26 00:01:49 1992 Newsgroups: sci.aeronautics.airliners Path: news From: Geoff.Miller@Corp.Sun.COM (Geoff Miller) Subject: Re: Tire burn-out during landings X-Submission-Date: 24 Dec 1992 20:54:29 GMT References: Message-ID: Approved: kls@royko.Chicago.COM Followup-To: sci.aeronautics.airliners Reply-To: Geoff.Miller@Corp.Sun.COM Organization: Sun Microsystems, Menlo Park, Ca. X-Submission-Message-Id: <1hd825INN7v@jethro.Corp.Sun.COM> Sender: kls@royko.Chicago.COM Date: 26 Dec 92 00:01:49 CST megazone@obsidian.WPI.EDU (MegaZone23) writes: >Spinning the wheels would indeed cut down on the tire wear, and I >believe that this has been tested. It was tested on the Lockheed Constitution, back in the late Forties or early Fifties. >When you spin a tire it acts as a gyroscope and will resists having it's >course altered. This can make handling tricky as you would have 10 or more >wheels all spinning on most airliners. Two points: First, I'd think that the mass of the spinning wheels, and therefore the magnitude of their gyroscopic effect on handling, would be insignificant compared to the mass of the rest of the airframe and payload and the power of the control surfaces. Second, assuming that the effect *was* significant, wouldn't it simply be a stabilizing influence, i.e., a resistance to changes in attitude about the longitudinal (roll) axis? That doesn't sound all that hazardous. >Plus you will have the initial cost for the system and the added weight. >It is one more system to break down, etc... I suspect that this is the most likely reason this idea hasn't been implemented; tires are simply cheaper to deal with in the long run. Geoff -+-+-+-+-+-+-+-+-+-+-+-+-+-+-+-+-+-+-+-+-+-+-+-+-+-+-+-+-+-+-+-+-+-+-+-+-+-+- Geoff Miller + + + + + + + + Sun Microsystems geoffm@purplehaze.Corp.Sun.COM + + + + + + + + Menlo Park, California -+-+-+-+-+-+-+-+-+-+-+-+-+-+-+-+-+-+-+-+-+-+-+-+-+-+-+-+-+-+-+-+-+-+-+-+-+-+- From kls Sun Dec 27 17:34:54 1992 Newsgroups: sci.aeronautics.airliners Path: news From: inc@tc.fluke.COM (Gary Benson) Subject: Re: Tire burn-out during landings X-Submission-Date: Wed, 23 Dec 1992 19:11:28 GMT References: Message-ID: Approved: kls@royko.Chicago.COM Organization: John Fluke Mfg. Co., Inc., Everett, WA Sender: kls@royko.Chicago.COM X-Submission-Message-ID: <1992Dec23.191128.13678@tc.fluke.COM> Date: 27 Dec 92 17:34:54 CST In article megazone@obsidian.WPI.EDU (MegaZone23) writes: >In article todamhyp@unlv.edu (Brian M. Huey) writes: >>the wheels come in contact from the ground due to friction. Couldn't >>this cloud and the effect of friction be lessened by inducing a >>angular velocity upon the wheels to match the airliners speed with > >Spinning the wheels would indeed cut down on the tire wear, and I believe that >this has been tested. However, there are problems. When you spin a tire it acts >as a gyroscope and will resists having it's course altered. This can make >handling tricky as you would have 10 or more wheels all spinning on most >airliners. Have you ever held a spinning bicycle tire in your hands and tried >to move it? Same effect. > >You also need to weigh the advantages against the disadvantages. How much does >the tire wear cost the airline? How much would mantenance on a system to spin >the wheels cost? > >Plus you will have the initial cost for the system and the added weight. It is >one more system to break down, etc... > >All in all it just doesn't appear to be worth it... > >############################################################################### ># I have one prejudice, and that is against stupidity. Use your mind, think! # >#Email megazone@wpi.wpi.edu Moderator, WPI anime FTP site 130.215.24.1 /anime# >############################################################################### > I'm glad this question was asked - I've often wondered the same thing! And I think the answer was very thoughtful and all in all, probably states the real reason spinnning the wheels is not done. However, if I may opine: RE: Gyroscope effect It seems that this could be used to advantage. After all, the wheels would tend to make the bird retain its current course. If you didn't start spinning till you were lined up with the runway, it seems that the spinning wheels could conceivably even help counteract sheer forces. RE: Maintenance Maybe there would be NO cost, for example if the tires were designed so that their tread caught the wind and got their spin from that. Or maybe the hubs could be fitted with fans. RE: Initial cost of system I think this could be done for free, too; particularly if the tread- or hub- induced spin just mentioned were employed. All you'd need to do is apply the breaks to keep them from spinning until you were ready. For safety's sake, you might want to have a "mini-brake" that would be enough to keep the wheels from spinning but which would easily be overcome if you landed with it applied. Seems cheap enough. Does anyone have any estimates about the costs using the current "cloud of smoke" and friction method of landing? How much does one of those tires cost? What is the expected number of landings it can endure? How fast would you have to spin the tire to get a 10% reduction in wear? 10% of the speed of the aircraft? -- Gary Benson -_-_-_-_-_-_-_-_-_-inc@sisu.fluke.com_-_-_-_-_-_-_-_-_-_-_-_-_-_- Stupidity cannot be cured with money, or through education, or by legislation. Stupidity is not a sin; the victim can't help being stupid. But stupidity is the only universal capital crime; the sentence is death, there is no appeal, and execution is carried out automatically and without pity. -Lazarus Long From kls Mon Dec 28 22:47:43 1992 Newsgroups: sci.aeronautics.airliners Path: news From: megazone@obsidian.WPI.EDU (MegaZone23) Subject: Re: Tire burn-out during landings X-Submission-Date: 28 Dec 1992 03:13:01 GMT References: Message-ID: Approved: kls@ohare.Chicago.COM Organization: WPI USAF -- United Society of Animation Fans Sender: kls@ohare.Chicago.COM X-Submission-Message-Id: <1hlrbtINN5dp@bigboote.WPI.EDU> Date: 28 Dec 92 22:47:43 PST In article Geoff.Miller@Corp.Sun.COM writes: >megazone@obsidian.WPI.EDU (MegaZone23) writes: >>When you spin a tire it acts as a gyroscope and will resists having it's >>course altered. This can make handling tricky as you would have 10 or more >>wheels all spinning on most airliners. >First, I'd think that the mass of the spinning wheels, and therefore the >magnitude of their gyroscopic effect on handling, would be insignificant >compared to the mass of the rest of the airframe and payload and the >power of the control surfaces. That's probably correct, although I don't have the weight of an average wheel and tire on hand to compare it seems so. Though a 26" 12-speed tire is enough to affect my 300lb body, I uspect the ratio of the gear to the aircraft weight is MUCH less... I too feel the major reason is cost, both for the hardware and lifetime costs. >Second, assuming that the effect *was* significant, wouldn't it simply be >a stabilizing influence, i.e., a resistance to changes in attitude about >the longitudinal (roll) axis? That doesn't sound all that hazardous. Arg,old physics comes back to haunt me... Making the large assumption that the mass is significant and the effect is as stated; I can see a few problems in that course correction includes rolling for turns, although I realize the angles used by airliners are a bit less than what I use in a C-172.... But that is just speculation, thinking about it now I would agree that the mass difference makes the gyroscopic effects negligible. ############################################################################### # I have one prejudice, and that is against stupidity. Use your mind, think! # #Email megazone@wpi.wpi.edu Moderator, WPI anime FTP site 130.215.24.1 /anime# ############################################################################### From kls Mon Dec 28 22:47:45 1992 Newsgroups: sci.aeronautics.airliners Path: news From: rdd@cactus.org (Robert Dorsett) Subject: Re: Tire burn-out during landings X-Submission-Date: Mon, 28 Dec 92 06:48:40 CST References: Message-ID: Approved: kls@ohare.Chicago.COM Organization: Capital Area Central Texas UNIX Society, Austin, Tx Sender: kls@ohare.Chicago.COM X-Submission-Message-Id: <9212281248.AA13397@cactus.org> Date: 28 Dec 92 22:47:45 PST In article you write: > >I'm glad this question was asked - I've often wondered the same thing! And I >think the answer was very thoughtful and all in all, probably states the >real reason spinnning the wheels is not done. However, if I may opine: > >RE: Gyroscope effect > >It seems that this could be used to advantage. After all, the wheels would >tend to make the bird retain its current course. If you didn't start >spinning till you were lined up with the runway, it seems that the spinning >wheels could conceivably even help counteract sheer forces. Strictly speaking, I don't see this as a gyroscopic effect. We're just talking about the rotational momentum set up by a spinning tire, and what to do about it. We need to consider three issues: (1), the means by which the tires get "spinning," (2) the actual control benefits by having the tires spinning on touch-down, and (3) the *additional* wear and tear on the brakes, as they must absorb the spinning energy, in addition to performing their normal task of slowing down the airplane. We could also add a (4), having the wheel assemblies spinning at high speed for extended periods of flight (outer marker to completion of roll-out), with the ramifications on the wheel structure (for one thing, a balancer to stop in-air "wobbling" would be needed). (3) seems the major disqualifier of the idea. With an inert tire, you'll have *minor* control problems ("bump", and that's it), but the energy absorbed by the tire in *spinning up*, on landing, in itself helps slow the airplane. That smoke's the energy being absorbed by the tire. If the tire's already up to landing speed, I can easily see landing distances lengthened considerably. In addition, with the excess energy being mopped up by the brakes, you've got a mandatory "cooling-down" time to consider. This could lengthen stop-over times considerably: an airplane can't take off again with hot brakes, since braking efficiency (which one would need for a rejected takeoff) goes WAY down, not to mention the resulting dangers of tire damage or wheel well fires. In reality, the issue is distance, not controllability. Anything to shorten takeoff and landing distances is to be supported; anything increasing them had better have some whopping benefits. :-) The current system is obviously cost-effective enough to be used. I don't have stats on tires handy, but the airlines do get a lot of wear out of them. >Does anyone have any estimates about the costs using the current "cloud of >smoke" and friction method of landing? How much does one of those tires >cost? What is the expected number of landings it can endure? How fast would >you have to spin the tire to get a 10% reduction in wear? 10% of the speed >of the aircraft? How would a "modified" tire design work on wet or snowy runways? And would a 20% increase in landing distance, resulting in a 30% reduction in the number of airports the carrier can service, be worth it? With companies eliminating movable autothrottles for 20-lb savings, do we really expect them to go for something with a potentially high number of "unforeseen" variables? :-) Landing and takeoff performance is an awesomely complex discipline. There are a lot of variables to consider. --- Robert Dorsett rdd@cactus.org ...cs.utexas.edu!cactus.org!rdd From kls Mon Dec 28 22:47:45 1992 Newsgroups: sci.aeronautics.airliners Path: news From: EGEISELMAN@FALCON.AAMRL.WPAFB.AF.MIL Subject: VR COCKPIT QUESTION 12/16-3 X-Submission-Date: Mon, 28 Dec 1992 16:11 EDT Message-ID: Approved: kls@ohare.Chicago.COM Followup-To: poster Sender: kls@ohare.Chicago.COM Date: 28 Dec 92 22:47:45 PST I have an exercise that I would like to propose to the net. This exercise is intended to demonstrate that the collective creativity and expertise of net participants can be harnessed via a specific methodology. I think the net should be exploited as a population of subject matter experts and a source of user input from which the extracted information may be applied to solving real design problems. Through an iterative process of concept refinement, using the collective knowledge base of the net, it may be possible that otherwise undiscovered design questions, problems, concepts, capabilities, and etc be revealed. I am going to take a look at this idea by doing the following: I will post a purposely vague design question to the participants of the net. Do with the information what you will. For those of you who choose to participate in the exercise, e-mail your individual responses to me. Please use the date of the post you are responding to in the subject field of the message. Also, be sure to include the date of the post you are responding to in the body of the mail message. Feel free to submit questions to the net for clarification and discussion but I will not extract information directly from the net. If you need a definition or have a question, ask the net first. About four weeks after the original post I will submit an edited compilation of the net response. This post will hopefully act to spark more ideas, make clarifications, and identify problem areas. The refinement process will continue. The net may then respond to the new description in order to patch holes, make corrections, and propose changes. This iterative process will continue until responses die off and/or the concept is solidified. I will document this process and report the results (I will post the report). -------------------------------------------------------------------------------- Sample question: Let's say your are given a virtual reality system. Your system consists of a high resolution wide field-of-view full color head mounted display devise (display), an extremely accurate head tracking system (transducer), and a 60Hz graphics generator (image generator). Given this technology, how should it be applied to the flight deck of a commercial airliner? -------------------------------------------------------------------------------- Note: Please indicate, in your e-mail responses, if you would like your name and/or affiliation to be excluded from any publication which may result from this exercise. Any information on personal background or experience you want to include may be of some use. All credit and acknowledgements will be made as appropriate. My thanks to all who participate. e-mail to: EGEISELMAN@FALCON.AAMRL.WPAFB.AF.MIL From kls Mon Dec 28 22:49:50 1992 Newsgroups: sci.aeronautics.airliners,sci.aeronautics Path: news From: Chris Scott Subject: GPS (DGPS) RTCA std.? X-Submission-Date: 21 Dec 92 13:52:52 CST Message-ID: Approved: kls@ohare.Chicago.COM Followup-To: sci.aeronautics Organization: Western Kentucky University, Bowling Green, KY Sender: kls@ohare.Chicago.COM X-Submission-Message-ID: <1992Dec21.135252.4665@wkuvx1.bitnet> Date: 28 Dec 92 22:49:50 PST Clearly GPS is going to change aviation navigation; The power it has to create approaches into anywhere makes the imagination wander... -- Differential GPS has been demonstrated by (Trimble?) for the FAA and I think it has been discussed somewhat in *Navigation*. The concept is relatively simple, so it seems to me that all that is lacking is 1. A correction data protocol standard, and 2. The transport medium: (sat, vhf, atis burst, awos append...?) Recently I read in *Avionics* a mention about GPS GIC ..GPS Integrity Channel, but I have not heard about any RTCA or FAA formal standards being proposed in this area. Anyone know the direction that this is likely to go? Chris Scott, C/E Public Radio, Western KY Univ, (502) 745-3834 SCOTTCR@WKUVX1.BITNET FAX OFFICE: 745-2084 FAX HM: 781-1232 From kls Tue Dec 29 09:43:53 1992 Newsgroups: sci.aeronautics.airliners Path: news From: spackman@disco-sol.dfki.uni-sb.de (Stephen Spackman) Subject: Re: Tire burn-out during landings X-Submission-Date: 29 Dec 1992 11:47:44 GMT References: Message-ID: Approved: kls@ohare.Chicago.COM Reply-To: stephen@acm.org Organization: DFKI Saarbruecken GmbH, D-W 6600 Saarbruecken Sender: kls@ohare.Chicago.COM X-Submission-Message-Id: Date: 29 Dec 92 09:43:53 PST In article rdd@cactus.org (Robert Dorsett) writes: |>RE: Gyroscope effect |>It seems that this could be used to advantage. After all, the wheels would |>tend to make the bird retain its current course. If you didn't start |>spinning till you were lined up with the runway, it seems that the spinning |>wheels could conceivably even help counteract sheer forces. | |Strictly speaking, I don't see this as a gyroscopic effect. We're just |talking about the rotational momentum set up by a spinning tire, and what to |do about it. There *is* definitely also a gyroscopic effect, but I agree that it's hard to imagine that it would be very significant. |(3) seems the major disqualifier of the idea. With an inert tire, you'll |have *minor* control problems ("bump", and that's it), but the energy absorbed |by the tire in *spinning up*, on landing, in itself helps slow the airplane. |That smoke's the energy being absorbed by the tire. If the tire's already up |to landing speed, I can easily see landing distances lengthened considerably. | |In addition, with the excess energy being mopped up by the brakes, you've |got a mandatory "cooling-down" time to consider. This could lengthen |stop-over times considerably: an airplane can't take off again with hot |brakes, since braking efficiency (which one would need for a rejected |takeoff) goes WAY down, not to mention the resulting dangers of tire damage |or wheel well fires. We should go carefully here. The auto industry has only recently started putting anitlock brakes on consumer vehicles because it is "intuitively obvious" that a sliding tire brakes you better than a rolling one. It's also completely false - sliding friction is significantly lower than static friction (and it's static friction, not rolling friction, that's the right analysis when the brakes are on and you are on a good surface) - which is part of the reason trucks have used such brakes for much of my lifetime (the other is to avoid jackknifing. If aircraft have trouble in bad weather from uneven lateral braking forces causing them to slew suddenly at touchdown, that argument applies here, too; theoretically it might also help compensate for crosswinds, but now it's *really* time for me to defer to an expert). The effect is so pronounced that the method of *releasing the brakes completely* whenever adhesion drops is apparently a win (of course, you reapply them as soon as the sliding stops). I would guess that in fact having the tyres hit the ground synchronous would let you apply full mechanical braking force *immediately* without having to wait for the bumping and sliding to stop, and this should let you stop sooner. As to the brake cooling issue (assuming it is real and you can't just put bigger fins on :-), my computer scientist's instinct (not necessarily to be followed in the mechanical domain...) is to say, let the brakes look to themselves: we could put a processor in there that measures *both* temperature and adhesion and controls brake application - with respect to measured speed and a reported distance left to roll. It should be easy enough to programme the thing so that it is kinder to the brakes when there is lots of room, but knows enough to overstress them in emergencies. We can easily arrange it so that there is a mechanical override, if you like, too. Increased complexity, perhaps, but a more tightly controlled braking profile would again argue in favour of the non-slip approach. |Landing and takeoff performance is an awesomely complex discipline. There |are a lot of variables to consider. Sounds like we may have found another whole batch. Disclaimer: I'm in programming languages, so what would I know? ---------------------------------------------------------------------- stephen p spackman +49 681 302 5288(o) 5282(sec) stephen@acm.org dfki / stuhlsatzenhausweg 3 / d-w-6600 saarbruecken 11 / germany ---------------------------------------------------------------------- From kls Tue Dec 29 12:39:16 1992 Newsgroups: sci.aeronautics.airliners Path: news From: drchambe@tekig5.pen.tek.com (Dennis Chamberlin) Subject: Re: Tire burn-out during landings X-Submission-Date: 29 Dec 92 17:38:58 GMT References: Message-ID: Approved: kls@ohare.Chicago.COM Reply-To: drchambe@tekig5.pen.tek.com (Dennis Chamberlin) Organization: Tektronix, Inc., Beaverton, OR. Sender: kls@ohare.Chicago.COM X-Submission-Message-Id: <8105@tekig7.PEN.TEK.COM> Date: 29 Dec 92 12:39:16 PST In article inc@tc.fluke.COM (Gary Benson) writes: > >Does anyone have any estimates about the costs using the current "cloud of >smoke" and friction method of landing? How much does one of those tires >cost? > I recall an old issue of AWST that described the certification flight test program for the first 747. The flight test budget included a 3 million $ line item for "Wheels, tires, and brakes". At least one of these tests was known to be destructive. The worst-case demonstration of a takeoff abort would seem to thoroughly cook all of the tires and brakes. I don't know if any of the wheels were savable. From kls Tue Dec 29 22:53:45 1992 Newsgroups: sci.aeronautics.airliners Path: news From: wolfgang@trout.nosc.mil (Lewis E. Wolfgang) Subject: Re: Tire burn-out during landings X-Submission-Date: Tue, 29 Dec 1992 19:45:12 GMT References: Message-ID: Approved: kls@ohare.Chicago.COM Reply-To: wolfgang@trout.nosc.mil Organization: NCCOSC, NRaD Division Sender: kls@ohare.Chicago.COM X-Submission-Message-Id: <1992Dec29.194512.10054@nosc.mil> Date: 29 Dec 92 22:53:45 PST In article 194@ohare.Chicago.COM, rdd@cactus.org (Robert Dorsett) writes: >Strictly speaking, I don't see this as a gyroscopic effect. We're just >talking about the rotational momentum set up by a spinning tire, and what to >do about it. (Stuff deleted) > > (3) the *additional* wear and tear on the brakes, as they >must absorb the spinning energy, in addition to performing their normal >task of slowing down the airplane. >(3) seems the major disqualifier of the idea. With an inert tire, you'll >have *minor* control problems ("bump", and that's it), but the energy absorbed >by the tire in *spinning up*, on landing, in itself helps slow the airplane. >That smoke's the energy being absorbed by the tire. If the tire's already up >to landing speed, I can easily see landing distances lengthened considerably. (lots more stuff deleted) Robert, I think you have your mass off by several orders of magnitude. If the mass of the rotating tires is insignificant compared to the total mass of the airframe (to negate the gyroscopic effect) then it will contribute negligibly to the stopping distance. Consider, if you will, when the pilot manages to "grease" one on: no noticeable deceleration is observe at the moment of touchdown. There may be a vertical "bump", but not a horizontal one. (if there is no crosswind component) IMHO you would see no measurable difference in rollout distance or residual heat in the brake system. Luck Lewie wolfgang@nosc.mil From kls Tue Dec 29 22:53:48 1992 Newsgroups: sci.aeronautics.airliners Path: news From: yarvin-norman@CS.YALE.EDU (Norman Yarvin) Subject: Re: Tire burn-out during landings X-Submission-Date: 29 Dec 1992 17:32:00 -0500 References: Message-ID: Approved: kls@ohare.Chicago.COM Organization: Yale Computer Science Department Sender: kls@ohare.Chicago.COM X-Submission-Message-Id: <1hqjl0INNsk6@CATHY.NA.CS.YALE.EDU> Date: 29 Dec 92 22:53:48 PST rdd@cactus.org (Robert Dorsett) writes: > [...] (3) the *additional* wear and tear on the brakes, as they >must absorb the spinning energy, in addition to performing their normal >task of slowing down the airplane. [...] This additional energy is negligible. Consider just the energy of the wheel itself. For a wheel which is rolling along the ground there is the relation: (rotational energy) = (some constant) * (energy of forward motion) where the constant is independent of speed. I seem to recall that for a cylindrical wheel of uniform consistency, the constant is 2/7. At absolute worst the constant will be 1. (This would occur if the entire mass of the wheel were on the tread of the tire.) Furthermore the energy of forward motion of the wheel is an insignificant portion of the energy of the entire aircraft. This goes by weight; if the airliner weighs 100,000 pounds and a wheel weighs 300, the proportion of energy in that wheel would be 3/1000 of the aircraft's energy. Then, using the 2/7 figure, the spinning energy of the wheel would be 3/1000*2/7 = .08% of the energy of forward motion of the aircraft. Assuming constant deceleration force, stopping distance would be lengthened by that same .08%. Even the weight of the mechanism required to speed up the tires might be a bigger factor. In any case, the practicality of preventing tires from disintegrating depends on how fast tires presently disintegrate. How much matter really is there in that cloud of smoke? Perhaps a gram per cubic meter of smoke? And how much tire is left on the runway? Do they have to go out and scrape it off now and then? (I imagine not.) Seems to me the loss of tire material is negligible also. Compared, that is, with the other costs of running the airplane. -- Norman Yarvin yarvin@cs.yale.edu From kls Tue Dec 29 22:53:48 1992 Newsgroups: sci.aeronautics.airliners Path: news From: mweiss@mis.mi04.zds.com (Mitchell Weiss) Subject: Re: Aha! The three-man 767 rears its ugly head... X-Submission-Date: Tue, 29 Dec 1992 23:26:40 GMT References: Message-ID: Approved: kls@ohare.Chicago.COM Organization: Zenith Data Systems Sender: kls@ohare.Chicago.COM X-Submission-Message-Id: <1992Dec29.232640.16703@mis.mi04.zds.com> Date: 29 Dec 92 22:53:48 PST In article rdd@cactus.org (Robert Dorsett) writes: > >I knew I wasn't going mad... In a recent post, I commented on a three-man >767. Karl hadn't heard of it, which surprised me, so I went looking for >it--and couldn't find it--which surprised me even more. I finally ran across >this blurb. > Harvard MBA folks publish an entire case study on the changeover from a three-man cockpit to a two man cockpit. It seems that the initial production runs of the 767's were three-man. Now, as a case study they never said what Boeing did. Did they change them before sale? I don't know. I know little about airlines, except of course how to buckle a seat belt and pray that my luggage arrives. :-) From kls Tue Dec 29 22:53:49 1992 Newsgroups: sci.aeronautics.airliners Path: news From: drinkard@bcstec.ca.boeing.com (Terrell D. Drinkard) Subject: Re: Safety and design rankings (was Re: Flight controls) X-Submission-Date: Tue, 29 Dec 1992 23:35:50 GMT References: Message-ID: Approved: kls@ohare.Chicago.COM Organization: Boeing Sender: kls@ohare.Chicago.COM X-Submission-Message-Id: Date: 29 Dec 92 22:53:49 PST In article kls@ohare.Chicago.COM (Karl Swartz) writes: >In article rdd@rascal.ics.utexas.edu writes: >>I would suggest not: the former is more of a derivative, the latter more of >>a new type, with its new wing (which was designed to support the all-upper- >>deck concept, plus maybe one more derivative after that) ... Both airplanes are derivatives, actually. The MD-11 was certified under the DC-10's certification basis. >My understanding was that the 747-400 does *not* have a new wing but >rather a tweaked version of the original. I recall some statement >from Boeing regarding the lack of winglets on the 777, which noted >that the 777 had a new wing and starting from a clean slate it was >more efficient to not have them, whereas working from an existing >design as with the 747-400 it was helpful to have them. I can't speak to exactly how much of the 747-400's wing design could be considered 'new', but I do know that it was modified extensively to change the pressure distribution across the wing - particularly the inboard wing where we were seeing double shocks on the older models. >What I've seen suggests the F.100 is quite advanced, probably not far >behind the A320 and perhaps closer to the Airbus philosophy than to >Boeing's. The Fokker 100's flight deck is much more Boeing-esk than Airbus-ish. There are control columns, and the FMS is very similar to the Boeing models. The F 100 does have envelope protection, as do the Airbus airplanes, but I don't think you would find it much different from what is already flying. It is a very advanced flight deck, very clean. >> INS/PMS, conventional otherwise > >Ok, I'll risk it ... what's PMS? (We're talking about airplanes!) Could it be the Performance Management System, a la Douglas MD-80? :-) >I've always wondered just what the flight engineer really does on a >767 equipped for three flight crew. I believe QANTAS does this. >Also, some A310s lack the FFCS (Forward Facing Crew Cockpit) having >instead what I assume is a cockpit more like an older A300. All of >these are due primarily to union/labor pressures. You are quite right, three-crew cockpits are union requirements on some airlines. The flight engineer on a 767 would do the same job done on other three-crew airplanes - deal with onboard systems. All those nifty controller boxes are left behind in Seattle. :-) -- Terry drinkard@bcstec.boeing.com "Anyone who thinks they can hold the company responsible for what I say has more lawyers than sense." From kls Tue Dec 29 22:53:50 1992 Newsgroups: sci.aeronautics.airliners Path: news From: drinkard@bcstec.ca.boeing.com (Terrell D. Drinkard) Subject: Re: Economics of new vs. older planes X-Submission-Date: Tue, 29 Dec 1992 23:46:42 GMT References: Message-ID: Approved: kls@ohare.Chicago.COM Organization: Boeing Sender: kls@ohare.Chicago.COM X-Submission-Message-Id: Date: 29 Dec 92 22:53:50 PST In article John DiMarco writes: >I understand there's an airfield in Arizona where unused airliners are parked >for extended periods. I'm wondering, considering the economic doldrums >most airline companies seem to be in, why these planes are not put into >use in lieu of buying new ones? > >The answer, I think, would depend on the difference in operating costs between >new and older planes. Could someone with a better understanding of these issues >shed some light on this? Solid numbers (eg. purchase prices of new vs. used >planes, fuel consumption differences, etc.) would be highly appreciated. Well, it is not a simple question John asks here. For instance, lots of 727-200s parked at Davis-Monthan, and at Mojave. A used 727-200 Advanced should bring between one and two million (prices are subject to inbelieveable fluctuations). 727s of any sort are no longer available new. The nearest replacement airplane, regardless of what the Boeing PR office may say, is the Airbus A320 - catalog price of about $41 MILLION. Granted, the 727 has a fuel burn half again larger for the same route, same payload, same speed, but it costs a whole lot less to acquire. Unfortunately for 727 owners, they are stage two noise compliant. This means that they will no longer be allowed to operate in the domestic US after 2000 or so. Europe is much the same. This short economic life must be take into account before purchasing our hypothetical 727. Another aspect is if one has already purchased a 757, for instance, then one is still obligated to make the loan payment each month. And the loan payments are huge. So, if you already have the airplane, you may as well put it to work, especially if it is more efficient than some other airplane. Lastly, I'd like to mention that over-capacity is one primary problem that the airlines are trying to deal with. Adding more available seat miles is not a solution. -- Terry drinkard@bcstec.boeing.com "Anyone who thinks they can hold the company responsible for what I say has more lawyers than sense." From kls Tue Dec 29 22:53:50 1992 Newsgroups: sci.aeronautics.airliners Path: news From: chuckh@apex.com (Chuck Huffington) Subject: Re: Tire burn-out during landings X-Submission-Date: Tue, 29 Dec 1992 23:57:30 GMT References: Message-ID: Approved: kls@ohare.Chicago.COM Organization: Apex Computer Company X-Submission-Message-ID: <1992Dec29.235730.7247@apex.com> Sender: kls@ohare.Chicago.COM Date: 29 Dec 92 22:53:50 PST In article inc@tc.fluke.COM (Gary Benson) writes: >RE: Maintenance > >Maybe there would be NO cost, for example if the tires were designed so that >their tread caught the wind and got their spin from that. Or maybe the hubs >could be fitted with fans. This has been done. I have seen old tires with molded in "scoops" on the sidewall that spun them up in the airstream. I'm not sure but I seem to recall the idea was dropped because the the additional wear on the brakes more than made up for the reduced wear on the tires. Apparently a lot of braking action occurs when the not spinning tires hit the pavement. From kls Tue Dec 29 22:53:51 1992 Newsgroups: sci.aeronautics.airliners Path: news From: drinkard@bcstec.ca.boeing.com (Terrell D. Drinkard) Subject: Re: Safety and design rankings (was Re: Flight controls) X-Submission-Date: Wed, 30 Dec 1992 00:01:22 GMT References: Message-ID: Approved: kls@ohare.Chicago.COM Organization: Boeing Sender: kls@ohare.Chicago.COM X-Submission-Message-Id: Date: 29 Dec 92 22:53:51 PST In article hoyme@src.honeywell.com (Ken Hoyme) writes: > >In article kls@ohare.Chicago.COM (Karl Swartz) writes: >> My understanding was that the 747-400 does *not* have a new wing but >> rather a tweaked version of the original. I recall some statement >> from Boeing regarding the lack of winglets on the 777, which noted >> that the 777 had a new wing and starting from a clean slate it was >> more efficient to not have them, whereas working from an existing >> design as with the 747-400 it was helpful to have them. One further comment. The 747 is constrained on span, therefore the winglets were the optimal choice for improving the efficiency of the wing. The 777 has chosen to offer two options: folding wingtips, and 'ignore the existing infrastructure'. Both options allow an unconstrained span, which gives better induced drag performance. >I believe the winglets issue on the 777 was also complicated by the >folding wing option. Which no one has ordered -- even those airlines >who originally expressed interest in the option. Has development on the >folding wing stopped?? I had heard that Boeing was getting tired of the >investment required to keep the option open while not receiving any >orders for it. I don't think I'm giving anything away by saying that the folding wingtip option is still being studied, primarily by the New Large Airplane Division. They apparently don't think they can get away with a 'damn the infrastructure' attitude. :-) -- Terry drinkard@bcstec.boeing.com "Anyone who thinks they can hold the company responsible for what I say has more lawyers than sense." From kls Tue Dec 29 22:53:52 1992 Newsgroups: sci.aeronautics.airliners Path: news From: kls@ohare.Chicago.COM (Karl Swartz) Subject: Re: Aha! The three-man 767 rears its ugly head... X-Submission-Date: Wed, 30 Dec 1992 06:51:50 GMT References: Message-ID: Approved: kls@ohare.Chicago.COM Organization: Chicago Software Works X-Submission-Message-ID: <1992Dec30.065150.13652@ohare.Chicago.COM> Sender: kls@ohare.Chicago.COM Date: 29 Dec 92 22:53:52 PST In article rdd@cactus.org (Robert Dorsett) writes: >I have no recollection of a picture of one of these critters, though. Can >someone clear up this matter? Yesterday I flew back from Chicago on a 757, and since United takes advantage of the common rating for flight crews on 757s and 767s I figured the guys up front might be able to answer this question. They agreed that none of the United 767s have a three-person flight deck, including the second 767 built (N601UA; the first is still with Boeing) which one of the guys had been on recently. As far as either could recall these aircraft were delivered without the third position though they pointed out that the luxuriously roomy cockpit was because the original design did have the third position, and one mentioned that the first simulators had the FE panel. Note that the first 767 actually delivered (to United) was the ninth built. The second one (again, Boeing kept the first) was delivered to United six months later, time which could well have been spent removing the FE panel as well as restoring the aircraft after the flight tests. -- Karl Swartz |INet kls@ditka.chicago.com 1-415/854-3409 |UUCP uunet!decwrl!ditka!kls |Snail 2144 Sand Hill Rd., Menlo Park CA 94025, USA Send sci.aeronautics.airliners submissions to airliners@chicago.com From kls Tue Dec 29 23:02:31 1992 Newsgroups: sci.aeronautics.airliners Path: news From: kls@ohare.Chicago.COM (Karl Swartz) Subject: Re: Safety and design rankings (was Re: Flight controls) X-Submission-Date: Wed, 30 Dec 1992 07:00:42 GMT References: Message-ID: Approved: kls@ohare.Chicago.COM Organization: Chicago Software Works Sender: kls@ohare.Chicago.COM X-Submission-Message-ID: <1992Dec30.070042.13889@ohare.Chicago.COM> Date: 29 Dec 92 23:02:31 PST In article drinkard@bcstec.ca.boeing.com (Terrell D. Drinkard) writes: >I don't think I'm giving anything away by saying that the folding wingtip >option is still being studied, primarily by the New Large Airplane >Division. They apparently don't think they can get away with a 'damn the >infrastructure' attitude. :-) It's also still being offered on the 777, as far as I know. My understanding was the part of United's reason for not ordering the option was the weight penalty -- 3,900 lbs., or 2,000 for just the hinges and stuff without the actuation mechanism, which allows easy addition later on. United preferred the weight savings and even at that went with a higher MGTOW version in order to operate the 777 on Chicago to Hawaii routes, which are right at the range limits of the A-market 777. -- Karl Swartz |INet kls@ditka.chicago.com 1-415/854-3409 |UUCP uunet!decwrl!ditka!kls |Snail 2144 Sand Hill Rd., Menlo Park CA 94025, USA Send sci.aeronautics.airliners submissions to airliners@chicago.com From kls Wed Dec 30 11:20:04 1992 Newsgroups: sci.aeronautics.airliners Path: news From: barnett@convex.com (Paul Barnett) Subject: Re: Tire burn-out during landings X-Submission-Date: Wed, 30 Dec 1992 15:37:40 GMT References: Message-ID: Approved: kls@ohare.Chicago.COM Organization: Engineering, CONVEX Computer Corp., Richardson, Tx., USA Sender: kls@ohare.Chicago.COM X-Submission-Message-Id: Date: 30 Dec 92 11:20:04 PST In yarvin-norman@CS.YALE.EDU (Norman Yarvin) writes: >In any case, the practicality of preventing tires from disintegrating >depends on how fast tires presently disintegrate. How much matter really is >there in that cloud of smoke? Perhaps a gram per cubic meter of smoke? And >how much tire is left on the runway? Do they have to go out and scrape it >off now and then? (I imagine not.) Seems to me the loss of tire material >is negligible also. Compared, that is, with the other costs of running the >airplane. Actually, yes, they do go out and "scrape" it off now and then. One popular method is very high pressure water jets. Having said that, I will comment that the most convincing argument I have heard so far is that the additional weight added by any spin-up system would negate any reduction in tire wear. -- Paul Barnett MPP OS Development (214)-497-4846 Convex Computer Corp. Richardson, TX From kls Wed Dec 30 12:55:24 1992 Newsgroups: sci.aeronautics.airliners Path: news From: Tony Heatwole Subject: Re: Boeing 767 Cockpit Size X-Submission-Date: Wed, 30 Dec 1992 13:21 EST References: Message-ID: Approved: kls@ohare.Chicago.COM Sender: kls@ohare.Chicago.COM X-Submission-Message-Id: Date: 30 Dec 92 12:55:24 PST I have a reprint of the Harvard Business School case study on the Boeing 767 (#9-688-040, Rev. 2/89). It's a fascinating look at technology, manufacturing, and the culture of the Boeing Corporation. With regard to the 767 crew size: "In August 1981, eleven months before the first scheduled delivery of Boeing's new airplane, the 767, Dean Thornton, program's vice president - general manager, faced a critical decision. For several years, Boeing had lobbied the FAA for permission to build wide-bodied aircraft with two-, rather than three person cockpits. Permission had been granted late in July. Unfortunately, the 767 had originally been designed with a three-person cockpit, and 30 of those planes were already in various stages of production. " . . . Engineers concluded that the thirty-first 767 was still far enough from completion that it, and all subsequent planes, could be built with two-person cockpits without modification. Thirty planes, however, were in relatively advanced stages of production. Some were nearly ready to to be rolled out and flown; others had complete cockpits but were not yet tested; others had bare cockpits without any electronics installed. But since all thirty were being built according to the plane's original, three-person cockpit design, all would require some modification. " . . . Customers were notified of the additional cost and delivery delay they could expect on these thirty planes. The impact was not large: a small percentage increase in costs and an average delay of one month from promised delivery dates. All but one airline chose to have their planes built with two-person cockpits." So, the interesting question is, what was the *one* airline, and what has become of their 3-person cockpit 767s? Were these planes later converted to 2-person cockpits? I don't know the answers, but I'm curious. Interestingly, Boeing decided to complete the first 30 767s as originally designed, for 3-person cockpits. Then, they converted the 30 (less the set to be delivered as 3-person cockpit) to 2-person operation as a batch. This avoided the safety and manufacturing problems of trying to convert a number of planes in different stages of manufacture. Tony Heatwole Gaithersburg, MD heatwole@hns.com From kls Wed Dec 30 13:27:32 1992 Newsgroups: sci.aeronautics.airliners Path: news From: kls@ohare.Chicago.COM (Karl Swartz) Subject: Re: Boeing 767 Cockpit Size X-Submission-Date: Wed, 30 Dec 1992 21:24:22 GMT References: Message-ID: Approved: kls@ohare.Chicago.COM Organization: Chicago Software Works Sender: kls@ohare.Chicago.COM X-Submission-Message-ID: <1992Dec30.212422.15659@ohare.Chicago.COM> Date: 30 Dec 92 13:27:32 PST In article Tony Heatwole writes: > "In August 1981, eleven months before the first scheduled > delivery of Boeing's new airplane, the 767, Dean Thornton, > program's vice president - general manager, faced a critical > decision ... > " . . . Customers were notified of the additional cost and > delivery delay they could expect on these thirty planes. The > impact was not large: a small percentage increase in costs and > an average delay of one month from promised delivery dates. > All but one airline chose to have their planes built with > two-person cockpits." > >So, the interesting question is, what was the *one* airline, and >what has become of their 3-person cockpit 767s? Were these >planes later converted to 2-person cockpits? I don't know the >answers, but I'm curious. Here are the first thirty-one 767s: ln sn model first flt customer reg'n -- ----- ------- --------- -------- ----- 1 22233 767-200 8/26/81 Boeing N767BA 2 21862 767-222 11/ 4/81 United N601UA 3 21863 767-222 11/24/81 United N602UA 4 21864 767-222 12/19/81 United N603UA 5 21865 767-222 1/18/82 United N604UA 6 22213 767-232 2/19/82 Delta N101DA 7 21866 767-222 3/25/82 United N605UA 8 22307 767-223 10/ 6/82 American N301AA 9 21867 767-222 7/20/82 United N606UA 10 21868 767-222 8/13/82 United N607UA 11 21869 767-222 7/19/82 United N608UA 12 22214 767-232 8/27/82 Delta N102DA 13 21870 767-222 9/17/82 United N609UA 14 22564 767-231 10/15/82 TWA N601TW 15 21871 767-222 10/30/82 United N610UA 16 22517 767-233 10/ 9/82 Air Canada C-GAUB 17 22215 767-232 9/25/82 Delta N103DA 18 22681 767-209 11/23/82 China Airlines B-1836 19 22308 767-223 11/ 1/82 American N302AA 20 21872 767-222 1/27/83 United N611UA 21 22565 767-231 11/13/82 TWA N602TW 22 22518 767-233 11/ 9/82 Air Canada C-GAUE 23 22309 767-223 11/16/82 American N303AA 24 22692 767-277 5/ 4/83 Ansett VH-RMD 25 22310 767-223 1/18/83 American N304AA 26 22216 767-232 11/24/82 Delta N104DA 27 22217 767-232 12/17/82 Delta N105DA 28 22693 767-277 5/20/83 Ansett VH-RME 29 22566 767-231 12/14/82 TWA N603TW 30 22567 767-231 1/28/83 TWA N604TW 31 22218 767-232 11/10/82 Delta N106DA I know United's are two-man and would bet the same for American, Delta, and TWA. Probably Air Canada. That leaves China Airlines and Ansett. I suspect the latter as I recall hearing about some Australian airline having three-man 767s. Any Aussie friends know for sure? Do their later 767s have three-man crews as well? BTW, I believe all thirty-one of these are still with their original owners with the possibly exception of TWA's -- some TWA 767s were sold earlier this year though I'm not sure if they were the oldest or newest ones. -- Karl Swartz |INet kls@ditka.chicago.com 1-415/854-3409 |UUCP uunet!decwrl!ditka!kls |Snail 2144 Sand Hill Rd., Menlo Park CA 94025, USA Send sci.aeronautics.airliners submissions to airliners@chicago.com